You are on page 1of 96

ji.

;ljI'
l. ,.l,

ANSWER BOOK FOR MORTIMER'S"

F T F T H

T I'"O I\

,,,1i: : ,,r; '


,:l'

,.i, , ' r li:: ' '


Illi:

', .

Lawrenc" Epstein M.

F'

Contents

Chapter chapter Chapter Chapter Chapter Chapter Chapter Cl.rpter Chapter Chapter Chapter Chapter Chapter Chapter Chapter Chapter Chapter Chapter Chapter

1 2 3
q,

6 B 9 lo 11 I2 13 I4 t5 16 L7 1B 19

Chapter 20 Chapter 21 Chapter 22


@ lgg by Wadsworth, Inc. All rights reserved. No part of this book may be reproduced, stored in a retrieval system, or transcribed, in any form or by any means, elecbronic, photocopylng, mechanical, recording, or otherwise without permission the prior of the publisher, Wadsworth publishing Company, Belmont, California 94OO2, a Division of Wadsworth,

Chapter Chapter Chapter Chapter Chapter

23 24 25 26 27

Introduction Stoichiometry Thermochemistry Atomic Structure Bond of Atoms and the Ionic Properties Bond The Covalent Orbitals Geometry; Molecular lutolecular Gases and So1ids Liduids Solutions Reactions in Aqueous Solution Chemical Kinetics Eguilibrium Chemical of Acids and Bases Theories Part I Ionic Equilibrium, Part Il Equilibrium, Ionic of Chemi-cal Thermodynamics Elements stry Electrochemi Hydrogen and Part I: The Nonmetals, the Halogens Part II: The Group VI A The Norunetals, Elements Part III: The Group V A The Nonmetals, Elements Part IV: Carbon, The Nonmetals, Boron, and the Noble Gases Silicon, I{etals and Metallurgy Complex Compounds Nuclear Chemistry Organic Chemistry Biochemistry

I 3
ti

l1 t7 24 31 42 49 6L 66 74 80 90 93 95 99 107 LL2

119 L27 131 T4L 150 155 l-73

I S B N0 - 5 3 r { - 0 e ? e 1 - b
Printed in the United States of America

L 2 3 4 5 6 ' t B 9 10---87

86 85 84 83

r'll/\['TER I I NI'I'IiODUCTION

(a) Sections 1.1 and 1.2, (b) Section L.2, See: (c) Section L.2, (d) Section I.2, (e) Section 1.1 (a) (e) (i) (b) antimony, (c) aluminum, (d) gold, strontium, (f) silicon, (g) mercury, (h) helium, silver, (1) cadnrium sodim, (j) neon, (k) calcium, (d) K, (e) Cu, (f) Co, (9) Fe, (k) Mg, (l) Mn, (m) Li,

t.2

(a) Sn, (b) Ti, (c) P, (h) I, (i) Cl , (j) cr, (n) Pb (a) 4, (h) 4
l.i

(b) I?,

(c) I,

(d) 5,

(e) 4,

(f)

1?,

(g) 4,

(a) 137.0, (b) 10.00, (f) 0.00210 (a) 5.94 x I0


- a

(c) 0.900,

(d) 5.0,
- q

(e) ]-I2, -,

l .( )

',

(b) 6.25 x lO-, (f)

3.0 x l-O

(d) O.096, (e) 13.6 , | . t | .rt (a) (a) (a) lOs cm, Io3

2.5 x lO2 (c) 107 ns, (d) 1O-r 6 rtn

(b) 10-6 kg, (b) (b)

lj-ter,

1o-3 m3 (c) 0.099 nm, 99 pm

|.,) .Io |.r,L

1O-1 nm,

102 pm,

1.5 m l-.21 km 1.17 x 102 day

| " t . 2.81 x lo3 hr,


l . t l ,

0.954 m3 9.595 trip m1 flour, and 4.93 mI vanj-11a,

l " l / l

l ' r

236.6 ml sugar, 59.2 m1 butter, 394.3 1I8.3 ml milk, 7.39 ml baking powder, l-.23 mI salt

I.16
1 1 ?

(a) 9s" increase' (a) 5B%, (b) (a) 2oo. 9 22 K

(b) 6% increase'

(c)

l0g" increase

r ' l l / \l " f E R 2 J ; , I ' 0I C I l I O M E T R Y

I.IB
1 1 0

PL, (b) r'25

x 103 g alloY l,,rl_tgnJ=Theory, ,l . l. Atomic W

I . 0 5 x l O 3 g Z n

See Secti-on 2.I. See Secfion 2-I.

4.0 x l0
*l .21

- 1

-a 3 O Og C u l e f t over' 60 g Ni left over


)..2. ,l.l

g 2OOO alloy; 5L.445 cm/s

A relative atomic weight is actually a ratio: the average mass of the atoms of an element compared to the mass of an atom of a standard. Since CH+ is 75.0e" by mass carbon and 25.0% by mass hydrogen, the compound consists of carbon to hydrogen in a mass ratio of 3 to 1. One C atom, however, is combined with four H atoms ii-Eu+. One C atom, is 3 times the mass therefore, must have a mass that of fo_ur H atoms. If the H atom is assigned a mass of The 1.OO, four H atoms would have a mass of 4.0O. relative mass of the C atom would be 3 times 4.OO, or 12.00. See Table 2.1.

t?

A9 km/hr 24.7 hr
(a) x lO7 m' 464-5 m/s, (b) 4'61 -7 ' m ' ( b ) O' I 3 1 ] r n (a) f .31 x 10 (c) 6 . 3 8 x 1 0 6 m

1A

L.

Z2

L.

ZO

a1

O . O 5 7 Oc m 3 cm3 rntense (a) 2.85 cm3r (b) 4'50 anto graphite to transform t.qoit.a 6 . 4 x 1 0 1 2 g A u 1.03 x 103 cm (a) o . B 5 8 9 , / c m 3, (b) f loat

,Q

pressure is diamond.

to

'l'1,,,@ .l.( (a) (b) (c) 24.7 mol- Hz, L.4g x IO25 molecules H2 H2O H2SOa

r1.30 kl.3l

2.78 mol HzOt L.67 x 102+ molecules

kL.32 3.380 Mm
tI. JJ

0.510 mol H2SOa 3.O7 x 1023 molecules | (b)

4.a22 x Lo27 g

( a ) 2 . 9 8 x 102 s atoms, ( c ) 2 . 1 5 x fo24


t {

5.01 x 1024 atoms,

atoms

4.480 x t_0 The atomic

g A I

weight

is

58.93

(the

e]ement Ir

is

Co)

(a) 4.6135 mo1 Pt,

O . 5 2 O 2 4 mol

( b ) 2 . 7 7 8 3 x 1024 atoms Pt,

3.\32g

x l-023 atoms Ir

rll_.29 *2.29 is over l'-r.lrcentage : . 3 0 ')..3L Composition Ni in Ni (co) q BaCO3 l! = 3; the formula is CuSOr . 3H2O

2.LL

(a) 0.14393 (b) 1.3335

mol Au'

8 '6676 x 1022 atoms Au

x 1020 atoms Au extend the 6'022 x 1018 km' which

VCl3

t 2.!2

They would

4 x 1O1o times

distance' 34.42 69.592


t

*2.L3

2.22 aomsCv

Ba in

2.32 Formulas 2.L4


' (c) VrS+ r (d) NaePeOz'+ ( a ) H g B s S o ( b ) N a z S z o ' + , (e) C+Ha ( d ) N e S e F sI (a) CoS' (b) B z o H r e r ( c ) S+N2, ( e ) P s N s C I ro

49.762 Zr in ZrSiOa

;.33 629 g Zn '2.34 2.00 kg Cu

2.L5

i . _ . l l _0 . 6 3 3 4 9 X e 0 . 3 6 6 6 s F ,t.36 2.9A9 9 Li, 2.011 9 N

2.L6 2.17 2.IB 2-Lg 2.20 2.2L 2.22 2.23 2.24

f,ien'* C1H12ON C2H6N ca5P3013H CeHsOa C e H l N O a CeOzHs C7H503SN H' (a) O.2lO mol C' 0'350 mol (c) 2'87 g CsH5 (b) CsHsr or c a s ( P o r + )g ( o H )

2-37

A 2 . 8 % C , 1 7 . 4 e "H o

* 2 . : 1 9 8 3 . 9 e "c , l 2 . O B H , 4 . l z r,)..39 a4.64% Fe2o3 in ore * : 14 0 . 1.554 S in oil

r'!!.rcmlgaI *Equations ').-!! (a) VzOs + 2Hz + VzOs + 2H2O (b) 282iu^3 7C -> BaC + 6CO + (c) 4Bi + 3O2 -> 2Bizos (d) CaCz + 2H2O + Ca (OH)2 * H2C2 ( e ) B a ( N O g ) z ' + H z S O ++ B a S O q+ 2 H N O 3 .'..42 ( a ) 3 N O 2+ H 2 O + 2 H N O 3+ N O ( b ) A l 2 S 3 + 6 H 2 O - > 2 A L( O H )3 + 3 H 2 S (c) 3SiC1a + Si -> 2Si2C15 (d) ( N H ' + ) z C r e O z 4 H z O* C r 2 O 3 +

2.25

H' 0'0900 mol S (a) 0.36o mol c' 0'360 mol g C+H+S (b) Cr+H+S, G) 7 '57 N

0.0555 mol tc2.26 (a) 0.389 mol C' 0'389 mol H, 0.777 g N (b) 4.67 I c' 0'393 I H, (e) CzHzNOz ( c ) L ' 7 7 g , ( d ) 0 ' 1 1 1 mol O,

2.27

x = 6; the formula is

CoCl2

'

6H2O

( e ) C a 3 N 2 + 6 H e O + 3 C a ( O H ) ' z+ 2 N H g

2.43

( a ) 2 C s H rs + 2 5 0 2 + l 6 C O z + I B H z O -> 2COz+ 3Hzo ( b ) C z H e O+ 3 O z ethyt alcohol requires Note that the combustion of of of fuel thn the combustion less oxvge"';t;-;;i" oclane does'
Equations

.l.61 .,..62 ,'..63 '.'..64 .l .65

107. O g KIO3 15.0 g NaOH

16.0 ml H3POa sol t n 75.0 ml AgNO3 sol'n 5. O0 ml KI4nO+so1 'n g CaO

Problems 2.44 2.45 2.46 2.47 2.4A 2.49 2.5o 2.5L 2.52 2.53 2.54 2.55 2.56 r'2.57 *2.58 *2.59

Based on Chemical 138.1 I H3Poa

.)..66 0.842 N2o; (b) 1'31 I NH3 :.67 . f. 6 8

(a) 6-00 g NaNH2' 3'38 I

2.86 g 12 (a) O.464 9. Na2CO3 , (b) 37 .Le" Na2CO3

3'L7 q Crzog 5.32 g KNo2 2'LL g KNO3 3.26 g Hr (a) 2-O7'l g As, (b) f5'5% Asao5 ' (c) 11 '7e" As

73.72 Na2SO3 4-99 g NH4scN 1.OB g F2 L.LO7 9 BzHo 1.29 I SFr

4.33 g oP(NHe)e B0'8% (a) 0.2956 9 Ti' (a) 1'94 $ NaN3' (b) 3'BO7 g TiCl3' (b) 61'9% Yield (c) 78'80% vield

45.0% Bao in

mixture rn-Lxture

62-Os" CaCO3 in L5-2% CaCO3

Reactions 2.60

in

Solution

22L g H2Soa

CHAPTER 3 TIIERMOCHEMISTRY

l . r {

r0 _12

-1081.6 kJ -7L.4 kJ

3.31

-2727.8 kJ

,'3.33 -622 kJ

Heat Measurements, 3.1 3.3 3.5 3.7 3.9 3.11 1.13 3.15 37"C -400c L.36 kJ/oC 139 kJ L44 J 26.740C 873 kJ 2.25 kJ/oc

Calorimetry 3.2 3.4 3.6 3.8 3.10 3.L2 3.14 3.16


18.8 kJ "c) 2.46 J/ (9 23-47"C 92.2 g -L7.78"C

l,;rr_f halp_iss

of

F_ormation

f-.34 (a) Ag(") +


(bI
, l

+clz

(g) + Aecl(s)

AS = -r27 kr
A+ = +33.8 kJ

JNz

(g) + oz(s) + Noz(g) n 1o1'|-l r i*rtgl

(c) ca(s) +.c(sraphite)

+ c a c o 3( s ) A+ = -L2O6.e kJ

1.35

(a)

1 + c(graphire) TuzG)

+ HcN(g) A--{ : = + 1 3 0 . 5 k J H

(b) c(sraphite) 2.75 x ro3 kJ 0.907 g CtzzzOtt J-,19 -1125. 2 kJ

+ 25(s) -> cs2 (f )


*
?

A+

= +87.86

( c ) Nz (g) + 2HzG)

ior(Sl

+ N H q N o s( s ) A"'t : = - 3 6 5 . 1 H kJ

Thermochemj-cal 3.17 (a) (d)

Equations (b) exothermic' (c) endothermic' t.3B l:i9 -I2I2.3 kJ

3.37 3.39

+96. B kJ -847 kJ

endothermic, exothermic
1 E

(a) CH3oH(1)+;or(s) (b) -764.L kJ

+ co2(q) + 2Hzo(r)

3 . 1 8 c 6 H 6( 1 ) * 7 o , 3.19 3 .2 0

( ( g ) + 6 C O zg ) + 3 H 2 o ( l )
f

Ag = -Z6a KJ
r.4I

c z H s o H( 1 ) + 3 O z( g ) + 2 c o z ( S )
L9.42 kJ
3. L

3Hzo(1) AH = -1368 69.r kJ

( a ) c 5 H 5( 1 ) r (b) -3267.7 kJ

+or(g)

+ 6coz (g) + 3H2o(l)

3 . 2 2 +76.3 kJ

3 . 2 3 383 kJ, 45r kJ

t-.42

( a ) N 2 H a( 1 ) + 0 2 ( g ) + N z ( g ) + 2 H z o ( L ) (b) +50.6 kJ/mol+ -' co2 (g) + Nz (s) + 2H2o(1)

La\^/ of Hess 3.24 3.26 3.28 + 5 0 .0 k J -300.1 kJ - 1 3 7 6 . Ok J


5. )

+81.5 kJ -149.9 kJ -319.5 kJ

\.43

( a ) c o ( N H e )e t s l (b) -333 kJ,/mo1

*oz(s)

3.27
3. Y

1.44 t.46

-163.0 -62.8

kJ,/mol kJ,/mol

3.45

-35I.5

kJlmol

r'llAl'TER 4

Bond Energres 3.47 3.48 3.50 3.52 3.54 3.56 3.58 3.59 -92 kJi in Table +96 kJ -206 kJ 153 kJ -23 kJ -330 kJ (a) -150 kJ (a) -L924 kJ
J . I

A---f : o f H

Hcl (g)

is

*92.3O kJlmol.

A'I'0MIC

STRUCTURE

3.49 3.51 3.53 3.55 3.57

+112kJ
Nu_

+132kJ
4.L

-121 kJ -120 kJ -1288 kJ


4.2

Both charge and mass determine the degree of path of an electron from a straight-line de)fl-ection in an electric or magnetj-c field. It is impossible factor either to investigate separately using Thomsonrs_method. (a) H-, (b) lighter higher x l0r+ mass charge g,/cm3

(b) -1s9 kJ (b) -1915 kJ

le2+,

*4.3

(a) I.82 (b) I-32

x 1Ol8 g

4.4 4.5

See Table 4.2 Since Z = 29 for Cu and Z = 79 for Au, the positive charge-of the Au nucleus is higher than the positive charge of the Cu nucl-eus. Consequently, more wider^rhen the Au foi-l angle deflections \^rere observed was used. 2.7 2.2 (a) (b) cm x lo-r2a 56 electrons; ,n B - : :o i
tJJ

r4_:9 n4:l 4.8

nucl-eus:

5 6 protons,

82 neutrons

4.9

(a)

78 electrons;
A,

nucleus:

78 protons,

1I7 neutrons

(b) I^zn JU

10

11

4.LO

Symbol P Ti Rb

Z 15 22 37 80 58 26

A 31 4A 85 202 140 56

Protons t5 22 37 80 58 26 35 Protons 19 25 40 a2 54 34 4A

Neutrons I6 26

Electrons l5 22 37 80

lllectromagnetic 4.21, (a) (c)

R radiation, (b) blue light,

infrared microwaves

!?
L22
z

4.22 4.23

iir

1.13 x lols /s (a) 4.29 x Lora /s, (d) 7.50 x LoL+/s, 2.84 x lo-re J

H9
UE

g
23
36
Electrons I9

- 3 +
H'

_U J
44 Neutrons 22 30 50 L26 78 46 66

4.g7 x 1o-re J L.99 x 1o-r3 J l.gg x Lo-22 J

"t4.11 Slmbol K Mn Zr Pb Xe
q

3s
Z 19 25 40 a2 54
2-

12
A 4L 55 90 204 L32 4 q 1r4

4.24

(a) 3.oo x ro2o /s, (b) 3.oo'x lol L /s,

4.25

(a) 333 m, 5.97 x 10-28 ,l (b) 250 nm, 7.96 x 10-16 J

2!
4-26

* 54 36 46

251 photons 43 s L.lA x :.ors ,/s, 263 nm J q - l -(b) 3 . 5 9 x 1 0 J (c) 5.41 x Lor+ 7s 555 nm (a) 4.97 x 10
- l q --

4.27 *4.28 *4.29

3 48

I I

cd2+

Isotopes'

Atonic

Weights

jil i
I

4.L3

(a) A, E; B, D; C, F,

(b) A, Bi Cr D; E, F; ^!9mag_gpegtra 4.31 97.24 nm 4.32 1875 nm

54n- 5or, 50.- _ 53"o _ 53v , ^ ,), 5 4 . ^ (c 26r'e, 24utt 23u' 24-t' 26'-' 23' 4.L4 The unit based or, 12", the current standard'

4.Ls oo.tszsr\v and o.zstllv and :z-soalflne 4.16 oz.otzLlTuxe 4.ti


4.18 4.I9

4 . 3 3n = 6 . \ = 2 4.34 (a) ultraviolet, visibte, visible (b) n = 2 '+ a = f rr = 4 + g = 2, !_ = 3 + n = 2 to the transition to the transition n = @ to n = 5. n = 6 to n = 5.

s 2 . 5 % 1 L a n d z . s a l u
69.72 u 20.18 u

* 4 . 3 5 2.279 Um corresponds 7.459 Um corresponds *4-36

(a) 3.289 x LOrs /s, (b) 2.L':.g x 10-18 J (c) 1.3:-2 x 103 k.r/mol

L2

I3

rr4.37

(a) (c)

2.g6L x 1016 /s,

(b) 1O.BO nm,

4.50

72.93 nm, 54.02 nm (3.28g x LOts /s)z-2-, (b) 1'316 x 1016 '/s 4.51

*4 .38

(a) K =

(a) = a (d) (f) (g)

(b) impossible 18 (entire (when n = 3 shell-), 3, 1 cannot equal 3), (c) I0 (3d subshell), (e) 2 (3s subshelt), 2 (a 3d orbital), (when I = 0, ml cannot equal +2). impossible 6 (3p subshell).

Periodic 4.39 4.4O

LlLw See Section 4.10

of elements are that the properties M e n d e l e e v stated atomic weight' of lncreasing functions periodic are of elements the properties MoseleY showed that of atomic number' functions periodic to be produced by electron X r a y s a r e believed the atom (to deep within to levels transitions Section 4'10' = 2 level). See o r n n = 1 the

(a) 2 (4s subshell). (b) impossible (when I = 0, m1 cdonot equal +3), (c) 2 (a 4f orbital), () 10 (4d subshell), (e) impossible (when n = 4, (f) 14 (4f subshell_), (S) 32 (the I cannot equal 4), entiren=4shell). (a) 9, (a) 24, (b) 17, (b) 12 , (c) (c) 8 4

4.52 4 .53

4.41
I

I i l-ectronic

Confiqurations

i
I
i l

4.54 *4.42 z=2a-o,2BNi *4.43 26-" 0.227 rw

ls 1'

2s 1L

2p 1L 1' 1t

3s lt

3p

3d

4s 1'

1t 1L 11, 1t 1 1 1 1

r"'
ouantum Numbers 4.44 4.45 *4.46 4.47 4.48
See Section See Table 4-I2 4-4
-36

zz' zg' :g' t'


4.13 (b) 2 5MN, (c) (c) 77Tr

396 4='

4.55 4.:9_ 4.57

See Section (a) rzcJ- t

rli
I

2oca, 63EUr

(d) (d)

3oZnl 7tTr,

(e) (e)

35Kr gsBa

(a) s+Xe,
TCL t

(b) +BCdr 63Eur

(a) 0.O243 r:m., (b) 3.51 x 1 0 See Figures See Section 4.15, 4.11

4.58

25Mn

4.L7 t and 4.IB

!.32
m -.-s
L1 /) -1 /) +1 /,
l 1 l m' n

The notations (a) 0, b, c, (b) 2,

can be checked by referring (c) 6, (d, 2, (e) 1,

to (f) 0

Table

4.8.

ii

4.49 n 1 1 2 2
r

m 0 0 0 0
'l

l 0 0 0 0
+1

m +'l /)
+-l /)

!.So
4.6I 4.62

d , a n d e are

paramagnetic to (f) Table 0, 4.8.

The notations (a) 2t &, b, c, (b) 4, d,

can be checked by referring (c) 3, (d) 4, (e) 0,

Z a

L | +l

(g) 2

-L/2
+1 /)

2
. Z I L

L
-'l

--t /2 -1 /)

and g are paramagnetic

L4

15

4.63

(a) 4zAgrz rs2 2s2 z{


c -l-

zt
:"'

?"

0 3d1o 4s2 4d6 4dr

CTIAPTER 5 I)ROPERTIES OF ATOMS AND THE IONIC BOND

(b)

s 2 P b z +z l s l 4fL4

2sr 5s2

t^6
(n"

o 4s2 496 4dr 0 gu 3ql o 6s2 Atomic Radii !{ithin a giroup the atomic size increases with increasing atomic number because the valence shell quantum number n, increases. Recall 5rincipal that in the Bohr atom, x = n2 x .059 nm. Ba ) Sr for example. Within a period the atomic size for the main grop elements decreases with increasing atomic nunber because the valence quanturn number is shell constant whilst the effective nuclear charge increases p < Si. and draws the electrons closer in. Within a period, the main group metals are the non-metals, as explained in 5.1 above. the transition metals are smaller, because d shell is filled the nonmetal-s add to the quantum level. larger than However after the next

5dl

(c) zrsc3+, ls2


(d) z+Cr3+: (e) 16S2 , ls2 lq2

zZ'
zt' zZ'

zg.u gt 2p6 3g'

3q6 396
3d3

5.1

2t

tz'

3g6
3dr o 4s2 gt dt o

(f) 53r :
I

tt zt 296 3x2 39: s"' sg'


(c) o' (d) 3'

5.2

4.64

(a) O,
only d is

(b) o,

(e) 0'

(f)

0,

Lii
i

Paramagnetic 4.13 and 4.14 t+2Mot 43Tcr 4+R 45Rh 46Pd and 79Au' subshell: Half-filled Cu, Pd, A9' subshell: Au' 5.3

4.65 4.66

See Sections 2 E C tt

zgCur qlNb

47Ag 54Gdr 78Pt, Cr, 4.67 Mo, Gd. Filled

non(b) representative' metal, (a) representive, (d) representative' metal, (c) transition, metal, (f) noble gas' metal, (e) innertransition, metal, nornetaI

The initial effect of entering the transition series is'a shrinl<age because of the increase in nuclear charge, while obtaining minimal shielding. The higher the I quantum number, the less effective the shielding. However as the d shell approaches completion it becomes an effectie shield between the next quantum shell and the nucleus. (a) Si > S (b) Sn > Si (c) ca ) Si, because Ga ) Ge (d) Al > Si and Ge ) Si (e) M9 > Si (f) s_ ) Cl , and Cl > F so Si > F (g) Si > C. All above in accord with rules in problem 5.1. Also following above rules: (a) po ) 1s, (b) (c) Ba > Sr (d) Sr > Sb (both are main group (e) fn > Sn ) Ge, element.s), (f) pb > Bi (S) K>Ca>Mg. 232 pm p > S,

5.4

5.5

1i.6

16

T7

4.63

(a) +zAg*, Lt' (b) s2pb2+2, t"l

z"' z"l

zgt 3"'

pt gqto 4"'

496 4dr o ap6 aar o

| i l / \ 1 , ' l ' L l R5 T'IT{)I'IIRTIES OF ATOMS AND THE IONIC BOND

2bt 39' p6 al o 42' sp6 sato os2

ltu s"t
a+

,uic Radii
(c) 2 1Sc"-: 1+ Is2

zt'
2* 2sz zZ' 5p6 zg'

2t

:" '

3p6

r,. l.
t 3

(d) z,*cr"-, (e) r652 (f) E3I : :

1s2 ls2 t"'

2p-G :"' 296 "t "t gt

?n6

e^6

3dr

o 4s2 4po 4dl o

srt
4.64 (a) o,
only 4.65 4.66 d is

Vithin a group the atomic size increases with increasing atomic number because the valence 'Recall shell principal quanturn number n, increases. that in the Bohr atom, x = n2 x .059 n{n. Ba > Sr for example. Within a period the atomic size for the main group elements decreases with increasing atomic nrnber because the valence quantun number is shell constant whilst the effective nuclear charge increases p < Si. and draws the electrons closer in. Vithin a period, the main group metals are the non-metals, as explained in 5.1 above. the transition metals are smaller, because d shell is filled the nonmetals add to the quantun leve1. larger than However after the next

',.2

(b) 0,

(c) O.

(d) 3,

(e) o'

(f) 0,

paramagnetic 4.13 and 4.14 ',.3

See Sections

2t+Cx, 29Cu, 4Nb 42Mo 43Tc 44Ru 45Rh, 45,Pd 47Ag 54Gdt Cr, Mo, Gd. 78PEt and 7eAu. Filled subshell: Half-filled Cu Pdr Ag, subshell: Au.

4.67

(b) representative, non(a) representtive' metal, (d) representative' (c) transition, metal, metal, (e) innertransition, meta1, (f) noble gas' metal, norunetal

The initial,effect of entering the transition series is'a shrinkage because of the increase in nuclear charge, while obtaining minimal shielding. the higher the 1 quantur number, the less effective the shielding. Hor^rever as the d shell approaches completion it becomes an effectie shield between the next quantum shell and the nucleus. (a) Si > S (b) Sn > Si (c) Ga ) Si, because Ga ) Ge (d) A1 > Si (e) Mg > Si and Ge ) Si (f) si ) cl, (g) Si > C. and Cl > F so Si > F A11 above in accord with rules in problem 5.1. Also following (a) po > Te, above rules: (b) (c) Ba > Sr (d) Sr > Sb (both are main group (e) In ) Sn ) Ge, elements), (f) pb > Bi (S) r>Ca>Mg. 232 pm p > S,

'.4

',.5

';.6

L6

L7

Ionization 5.7

EnergY

14

a trend just energy follows ionization The first There are anomalies to that of atomic size. opposite in however as explained two periods in the first problem 5.10. Because metal atoms are ionization metals their coulomb's law is E = generally energies than larger are 1ower. to non-

5.8

5.9

ql-92.

Within a period there is a general increase in the energy liberated by electron attachment because the atoms are getting smaller and the attached electron can get cl_oser to the nucleus. The same anomalies discussed in 5.10 occur, and are relatively important because we are concerned with energes much less than ioni-zation energies. Thus beryllium (which must attach a p electron) is so far out of line that tne attachment reaction is endothermj"c. (positive sign in table 5.2. ) Down a group differences are relatively small and unpredictable. Attachment energy is expected to be less exothermic for a larger atom but this effect is offset by the reduced electron-electron repulsion in the larger atom.

consider

9r

be -1,

and q2 the charge on the the charge on the electron, Since removing the electron. behind after ion left and +1 for the q2 = +2 for the second ionization, energy must be at l-east the second ionization first, r may be the same or Note that the first. twice ^ ^ ^ ^ . i h ] . , rs^ S S f O r t h e S e c o n d i o n i z a t i o n , but never },UDJTU!)' is the first electron greater since the outermost removed. 5.10 to the trend The sequences Be-B, and N-o, 90 contrary Be ) B in size the period, because although across a in higher removed from B is a p electron, electron The removed from Be. energy than the s electron from N, is one that removed from O, unlike electron p orbital and is subject a single occupying of a pair which makes it easier repulsion to electron-electron size of o). the smaller to remove (again despite the usual trends Following (d) Sb > Bi (c) Sb > Sn ( g ) A r > S ( f ) S > S e (b) (a) Sr > Rb (e) Se > Te > Sn Sn > Sr

r,.15

The positive sign indicates that energy must be expended to forcibly attach an extra electron to the atom. The attachment reaction is endothermic rather than exothermic which is more conmon. (a) s > CI in size; electron comes in closer to C1 and more enefgy is liberated. (b) p must put its extra electron into an already occupied orbital and overcome electron-electron repulsion which detracts from the exotherm. (c) As noted in 5.14 Be must add a p electron. which is much further out than the s electron added to Li. A1l negative ions resist the attachment of an additional (like charges repel each other). electron One must expend energy to force a second electron to attach; reaction is endothermic. (a) Na is largest. (b) Ar has the highest firsc ionization (c) Na is the most reactive energy. metal because it has the lo\^est ionizati.on energy. (d) Cl is the most reactive (e) ar is nonmetal. least reactive having all its valence orbitals filled.. (f) The metals are the three elements Na, Mg, AI.

',.16

5.11

'-L7

5.I2

the usual trends except O vs- N which All follow (b) Ne > Ar (a) Ne > o in 5.10. explained (e) l > o (anomalous) (d) F > Cl > S (c) o > S (g) Mg > Ca (f) M9 > Na

is ,-lB

Elect]:on 5.13

AffinitY of 1680 kJ of energy to the expenditure It requires from a mole of gaseous mole of electrons remove a When a fluorine atoms to form a mole of F+ ions. joins a mole of gaseous fluorine mole of electrons atoms to form a mole of F- ions, 322 kJ of energy is liberated.

IB

19

Lattice 5.19

En-ergy, -824 kJ mole

Bgrn-Haber

Cycl_e 5:2O '2]152 kJ mole-r

, .rlt

(a) (b) (c)

all

paired,

diamagnetic electrons, highly pararnagnetic

4 unpaired three

unpaired

electrons, unpaired,

paramagnetic paramagnetic

5.2r

-3010 kj mole -324 kJ mole-l K CI < K2O < CaO. Ereater charge

5.22

-603 kJ mole-I

(d) (e) (f)

thro electrons all all paired, paired,

diamagnetic diamagnetic

5.23 5.24

Choose K2O over

K C1 because of Choose Cao size

,.2.()

and smallet'stze of greater

of'o2-.

over K2O because - ^ 2 + 5.25

charge

and smaller

A simplified way of writing configurations of large atoms is to use a noble gas symbol for the'configuration of most of the electrons, as shown in the arls\/ers below. (a) (b) (c) s2cu+ cu2+ 1s22s22p63s23p6 or simply: or: [Ar] tArl 3dr 0

of greatest the guantity energy is usually The lattice for enough to compensate and mast be large magnitude the to ionize amount of energy required the large the next most important rnetal atom, which is usually
a'rn+i +rz Yssest.

Ls22s22p63"23p53dr 0 tArl [Ar] [Ne] lxe] . q5d10 4f r (Not" rhe 4ft45dI o6p2 3d3 diamagnetic diamagnetic has an unpaired diamagnetic diamagnetic diamagnetic 3de

(d) Sc5' (e) F (t)

sg2+

txel 5p6

slmbol includes

The lonic 5.26

Bondr

T)Pes- of NaCI M9CI AIC13

Iols
Na3N Mg gNe

Na2O Mgo AlzO


N L

(9) pb2+ (h) . lO (a) (b) (c) (d) (e) (f) (g) (h) cr3+ all all

[xe] [Ar]

paired, paired,

A1N

5.27

(a) ltgt' (b) cr2+ (c)


4r

rs2s2 2p6 rs22s22p63r23p63da rs22s22p63s23p63d7 Ls22s22p63s23p63d104=24p64d9 3 Ls22s22p6 =23p63dr04"24pt4dr r s ? 2 s 2 2 p 63 s 2 3 p 6 3 d r Same as xe .11 0

one d orbital paramagnetic all all all paired, paired, paired,

electron,

coz*

(d) pd2+ (e) Ag+ (f) r

the p electrons are in unpaired, paramagnetic

separate

orbitals,

t\^o

04"24p64d105=25p6

three unpaired d electrons orbitals, paramagnetic [Kr] j.soelectronic G^3+, cu+, with ce+,

occupying

separate

(a) (b)

Se2 , Br-,

pJc+, sr2+,

y3+

lzn?+)r

(uncommon)

20

2L

(c)

lznJ :

Gar, Ge+2 F , N e , N a t , M g 2 + ,A 1 3 + , N 3 Ki, s"tf, Ar, cl , s2-, Pt-, lin*

'. rr, t,. t"l

(a) Cs , (a) Br ,

(b) S(b) Cs,

(c) S- , (c) 02-,

(d) cr2+,

(e) Ag,

(f)

As

(d) [o2-], (e) [c.2+], 5.32 (a) [ca2+] (b) [as+] (c) tn+l (d) tHsl (e) Ixel
5,33 s2 iorr= are

(d) au+, (e) T1+, (f)

rn+

rrr3+, srru+, Ag+ cd2+, rn3+, srn* Kr, Br, s.2-, st'*, Yt*

rrrr,,r.rclature of ,. itJ

Ionic

_Csmpounds (b) Ca3 (Poh) 2 (d) Mg (c1o3 ) 2 (f) (b) ZnCO3


/-r r Pr

(a) CrzOg (c) (e) Ag2Cr2O7 li (Nog ) F e 2 ( S O +) e Ba (oH) 2 PbCrOr

r1+, Pbz+ re2-,


rare,

r-,

cr*,

Ba2+, L.3+
, . l'.) (a) (c) (e) ,.40 "2 iott are possible (d) Au2S3 (f) sulfate NHaC2H302

no negative

The sc Lit and Be2t are also rs2. except H-rs2. Only others are 1 electrongroup never loses just the dl0s2 li"t"d below. To summarir" s2: H-, Li+,

(a) manganese (II) (b) (c) (d) (e) (f)

most nonmetal t2p6 very comrnon, including 8.2+; metal ions of main ions and the positive negative groups A13+; rr rrr and III. include Examples o2-r'F-, all the zn group, sn++, Na+, Mg'*, ".q. zn'n, also ,.41

magnesium phosphate lead (II) carbonate (II) chloride

mercury

10 ions

sodium peroxide aluminum calcium cobalt tin (II) sulfate perchlorate (II) nitrate

and main group rrr, cu+, Ag+, Au+.

e.g.

G.3+; also i-ons aren't srr'+,

Pbt4;

Negative Tl*,

possible. sb3*, (a) (b)

dr 0s2 ions ano 5.34 A1'*, Pbi+,


L

are rrr*,

Pb2+, A"3*,

rJl(c) G.'*,
' +

fluoride permanganate phosphate iodide, or dimercury diiodide

S"3+, N' -, are

K+, Ba2+, ate dlos2 lons.

S2p6 iorrs.

(d) (e) (f)

potassium iron (II1)

T1+, Bi3+, cd",

cu*, 5.35

are dlo energy

iott. is requ.ired to form'Cu2+ compared

mercury

(I)

Additional

is not extreme because the second to cu+, but this is from the 3d she11 which is close to the electron energy is more The additional in energy. 4s shell energy lattice than compensated for by the greater the second rn the case of N-tf, in the case of crr2*. fat below the must come from the 2p shell, electron is so great and the energy requirement 3s in energy, for by the additional it cannot be compensated that lattice energy of the hypothetical Na2+ ion.

23

, , . ' ,

( a ) H - - - H t "

( b ) : C l - o - C l :

(c)
covalent 6.1 compounds, Hz, Nz, Lewis structures f2r (S2 known at high ( d ) : N = C - C = N :
I

4..

v , : O - C lI G V
I

C-2, Fz,

CLz, Br2

- 0.9
H

temperatures) 6.2 Ba Br2 must be ionic table), formula containing is always (opposite sides of the periodic (e) H-

'YO o - o L I :

The simplest Ba2+ and Br- ions. given for ionic compounds.

( a ) r - 3

(b)

:Cl :

6.3

(a) ,-c=tt, because it


structure charge at

A A :Fg=C-:KJ formal

I t

structure charge.

I is better In addition
F (c) H t . . H - C = O :

minimizes

2 is poor because N gained a negative the expense of the more electro-negative

'q],
(d)
/-1 tz-t .. :N -

t
,r,

"

(b) "r..@21

';'O "\@@z:'
H

t ^ I l2+l SV-

F:

"/*-*\o'ro

./

N=N

\.r:r .b.'v

Structure

\-/

(e)
. .

,it
l

,i,
1
l

is better because there is less formal charge 2 is very poor In addition structure separation. because it requires the same sign of charge on
adjacent 6 . 4 atoms. N : ( b ) H (d)H-

t . . , . {- 3 - 3 - I , (a) . . 4 :o: V I
I

(b)

( a ) H - C f (c) H

O , "o - J @ - ,O l
:9:A \-/,
(c) .. /a\ :o: v | ..a Q,V 1,4,c \-/,

" - l@- ,o
,9, O
(d)

I
H - S i - H

"

( e ) H - - r

I
H

(e)

O c\: o.. V :

I 'Y'

-O'" o'T
'9'o

,, O | '. ':\

,\:-j@-;,o " \7"


'Y'O
24

l ^

6.8

(a)

H - N - N - H t l
t U l E

( b ) H - N = N - H

pr. r rr rlId,fIC

t,. l.t

(c)

.F "

NI I

\T

"

F.

(a)O ,

Resonance is invoked in cases where a pair of electrons is not localized to the bonding region of a particular pair of atoms, so that the f.ewis structure scheme is inadequate. Such a molecuLe is a resonance hybrid, for which the best we can do to show its structure is to draw various resonance forms. The structures H-C=N: and H-N=C: represent trto different molecul_es with different skeletons, i.e., different sequences of attachments of atoms. In resonance forms the positions of the atoms are identical, only the electrons are shifted. If atom one atom may bear
+->

(e)

.nl

6.s

t.l O,c = o,O


(b) carbon
.'a'

:l= o:O

Orc = N,
either

:N= N:
nitrogen

nitrogen

carbon

6 .10

(a)

O.:;. '\
N\7 F:

has a formal charge of 1r, a formal_ positive eharge.


:NIp_N_H

no ad.jacent

\ ^ ,./

** ,/

* @ -I ,
(c)

. .. r a : N = @ .H p=NH--=,

@..o
is,-sufficient, but a possible whicrr

o..o..
(b),=-I,

this

structure

resonance srructure

is H -@A

= - i;,O;

o'e
o ( d ) , = 3 @- F ,
Compound (a) requires
6 . 1 1 ( a ) H - N - C = N :
la'l .Fr -

- @t \ . .
F:

however cannot be as important structure

as the uncharged

n -O

- s @ = [ < - -H - = i i @ - , O +
important, *-*

Borh
possible.

NT =

TrI -

F.

-o A :g-P=N: v
struc

structures

..e ,9=n'=, O

no uncharged structure

us to show resonance
(b)
. -

:CI: : C l- C = O :
l

:N=s:6
A

, ! - ,O @ , s = , l l * ' l l
O 'l!-F,
A

I I

(c)H-il-=,

+ O,s-,O ill <


t, l/

+
-

(d).. G) ..oo.. o ..oo.. o


: O = N - O : : O - N - O :

: O - N = O :

:O=N-CI : . Y ,{+

o ',,:'o c\ ..(9
. tT_c

- > l l l

, S-m,

:O-N-Cl

,9, ., '
\J

"

"

O:

<

"

,9t

1
A \,'
+->

I 'Y'O

ll

( " )q H - c : N : + - + = c = , O ,
Resonance is indicated by double-headed
o

arrow,

27

6.18

: "F - N - N = O : l I :F:

This structure

is

sufficient.

r,..!l r,..r')

IF

> BrF > C1F > ICI

> IBr

BrCl

:F "

. . g / . .
,Ir

. . tr./ a N = 5 - 6, I I
A

A possible resonance which however cannot

structure, be as

.. \9 6 . L EH - N = N = N : 4 . . 6.2o \'l :o-c=N:

important structure.
r + - >

as the uncharged

H - N - N f

o..@
The left

N :

A < - + :o=c=N: \./

form,

whi-ch has
,. lO

Electron affinity is the energy of electron attacrunent and can be measured precisely, either directly or indirectly (Born-Haber cycle). Electronegativity is a dimensionless numerical rating of an element's tendency to draw electrons to itself. It is an approximation dependent somewhat on the judgement of the person who worked. out the rating scheme. originally (Mulliken scale) it was based on an averase of ionization energy and electron affinity; the pauling currently popular scale is based on bond energy. In any case it is electronegativity whj_ch assesses the total character of the atom and thus provides us *IE-T guide to judging how polar a bond will be. CS < OCl < CCI < OC = SO < InI < CaS < AlO < Cacl Use A for a) b) c) d ) e) NaBr, N Br, PBr P S Pbr electronegativity < A1S < BeI < SiO

the negative charge on the more electronegative atom, wl-l make a greater to lhe contrj-bution form. resonance hybrid than the right-hand

6 . z L O , - = , < - > , = - , O
6 . 2 2H - C = O : # H - C

difference

bel_ow.

I I
:O: ,a " 6.23

I t \:,/
O:

- ,O
Other less are shown

A = o-.8 A = 0 . 4 A = 0 . 4

:O=C=C=C=O: important below.

This form i-s sufficient. forms wth formal charge

IOnlc Covalent, nearly 0B ionic Covalent, perhaps 10% ionic Covalent, slightly ionic Suggests covalent. However pbl2 is salt 1ike, 4OOoC, suggesting ionic. A is a poor

F \ D u

g ) B H

A = 0 A - .2

criterion Covalent, Covalent,


character Covalentr lonic Ionic Covalent Covalent Covalent Covalent Ionic Covalent Covalent Covalent

in this case. nearly O? ionic very lit.tle ionic


perhaps 17? ionic like HCl

O , - "= "- c = o ,@ *-t @:o =c-c=c- , O


Bonds of 6.24 Inermediate (a) CuClz, (f ) AI2S3 6 .25 ( a ) L i g Pr (f) 6.26 6.27 Fe2S3, Characterr (c) Electronegativity Lj-I, (d) PbCl*, (e) CdIz'

h ) B Br A = 1.0 i) BaBr A=2.I


t,.12 a) b) c) d)
p \

(b) MgSe, (g) SnIz (b) BeBrz, (9) In2S

(c)

Au203,

(d) SnBrz,

(e) AgzS,

fl

s)
h) i)

Bond is Bond is

5.86% ionic 15.3% ionic

CaO C O C10 C Cl C Mg CsO CS C I C H

L=2.4 A = 0.8 A = O.2 A = 0-6 A = 1.3 L=2.6 A=0 A = 0.1 A = 0.4

perhaps 108 ionic very low polarity slightly ionic very polar verylowpolarity very 1ow polarity very low polarity

29

.e

6.33

( a ) C H m o r e polar (c) (e)

( b ) OH more polar ( d ) OH more polar (f)


e NH e

<+

t , l l A l " l ' l l l {7

F S m o r e polar FS more polar

<i-->

,llr 'tll,AR GEOMETRy; MOLECULAR ORBITALS

more polar

Nomenclature 6.34 6.35 6.36 BfFs'

of

Covalent

-Binary_Compounds SsNe

@
NO contai.ns an odd number of electrons so one atom must be content with only seven electrons. In p C15 there must be l0 electrons around p to form the fve bonds to the chlorine atoms. In N the octet cannot be exceeded because for n = 2 there can be only four orbitals. and. at most eight electrons; whereas for P, where n = 3, there can be as many as nine orbitals and 18 electrons (although rarely more than 12). AB2 linear Ab3 planar AB2E bent triangle

S2CL2, PqN+ TeF5,

P r + O o 'B F 3 C l 2 O 3 X e F 4 , I z O z pentachloride a) phosphorus pentoxide, b) iodine or diiodine c) silicon tetrafluoride d) sulfur trioxide e) tetrasulfur tetranitride a) sulfur hexafluoride b) tetraphosphorus trisulfide c) d.ichlorine heptoxide, or d) silicon dioxide tetrafluoride e) dinitrogen

pentoxide

6.37

chlorine

heptoxide

ABa tetrahedron AB3E trj.angular AB2E2 bent AB5 trigonal bipyramid figure that may be a ,,see_saw,,, pyramid

AB4E an irregular four_sided called a "saw-horse" or AB3E2 T-shaped AB6, octahedral AB5E square AB4E2 square (a) (c) pyramid planar

AB+ tetrahedron, AB2 linear, (f) (d)

(b) AB5E square AB2E bent, (e)

pyramid,

aesn triangular (9) ABaE

pyramid, distorted T-shaped,

AB3E triangular or

pyramid.

tetrahed.ron (i)

"sa\-horse,,, bipyramj"d

(h) AB3E2

ABs trigonal

30

31

7.4

See previous (a) BHa sp3

problems

for

Lewis d2upt sp3 (i)

structures (c) (f) Beclz sp

( a ) sbF4

ABaE distorted horse " + . ABa tetrahedron AB3E trigonal ABaE2 square AB3E trlgonal pyramid planar pyramid tetrahedron, or

"sa$/-

(b) XeFsf (e) (h) sncl3

(d) sbF2? sp2 (g) TeFa dsp3

AsH3 sp3 dsp3

( b ) AsCla + ( c ) qa'^ (d) XeF4 (e) (f) CdBr2


4

rF3 dsp3

siF5

7.5

( a ) TlBra + (b) XeF3' ( c ) SClz ( d ) AsF2( e ) GaI3 (f) C1F4-

ABa tetrahedron AB3E2 T-shaped AB2E2 bent AB2E bent AB3 trigonal ABrEz sguare ABaE distorted AB5E square AB5E square sp3 (b) xer3+ (e) Gars Planar Planar tetrahedron pYramid pyramid dsp3 (f) (c) sc12 sp3 or "saw-hor

AB2 straight ABr*E distorted horse" AB2E3 linear 486 octahedron + ^ (b) AsC14- sp3 ( c ) s e F s ' s p ' (e) GeF3 sp3 (f) CdBrz sp (h) IBr2- dsp3 ( i ) s i F 6 z - d 2 s p 3 tetrahedron or ,'saw-

( s ) IJJ- I
(h) (i)

I.Br2 " . i o . -2 -

( s ) PBra
( h ) TeF5 (i) sbFs2-

(a) (d)

(s)
/. tI
(a)

sDI4 dspxeFa d2sp3 Bira- dsp3

7.6

(a) TlBra-

C12O :Cl-O'
? . . ;

il,
A S -

(d) AsF2+ sp2

sp2

crFa (i)

dsp3 sbFs2- d2"p3

(s) pBra7.7
\4,1

(b) A s O 3 -

dsp3

(h) teF5-

:Ov

d2sp3

Bicls+

AB5E square AB5E square AB2E2 bent AB2 straight AB3 trigonal AB2E bent

Pyramid PYramid (c) OeSClz

,,O
4 . . V:O: A, v l *

AB2IJ2, sp3, bent . . 4 O: v AB3E sp" trigonal pyrardd

( b ) SeF5 ( c ) C1F2 (d)

InC12-

@ - ii,

ABa sp3 rerrahedron

( e ) BeF3 GeF2

Planar

.r'l .

( s ) AsFa
(h) xeFz (i) AlH+

tetrahedron, ABaE d.istorted " savt-horse " AB2E3 linear ABq tetrahedron

or

(d) oscl2

t = 3 - ii, I

AB3E sp3 rrigonat

pyramid

'9]'
(e) oCclz
:O = C - Cl:
I

AB3 sp2 trigonal

planar

I
(a) eicrs2d2sp3 (b) sers dt"pt (f) (i) (c) crFe+ sp3 GeF2 sp2 AlHa sp3
t.t-2 (a) (f)
. ^ 1 .

(d) InCl2+ sp

(e) BeF3- sp2

(9) AsFa- d,sp3 (h) xer2 dsp3

csz

t3 = C = 3r

AB2 sp

linear

:o:
tl . . t l - P - C l : : C 1 I " AB4 sp3 tetrahedron

I
32
:Cl:

(b) , = sl - ii,
( c ) v : o - C = N : tn.t Not. considered. the form:

O,,
AB2E sp2 bent AB2 sp linear

, 6O ,

( a ) o 3 c 1 o c 1 oc l z o z ) O , A- " l @ (3

4 . .

o,l,

- - " l @ - , u "

,l'o
of

onfy one resonance form need be be deduced will The same results , -

from

\Y ,a\ .. ( d ) ' ' ' : o - N - F :

/
II

c =,O
AB3 sp2 trigonal Planar

Each Cl is tetrahedra

sp3 (ABa) so the molecule consists joined (central at a corner O).

two

" l l

o:
(e) C10g

(b) o2NoNoz (NzOs )

o,, \o
/

:
-

,O
(and resonance forms)

. . @ o - N
\

\.. v

:O -

ii @ - , O "
| ,, O

AB3Esp3 trieonal
^,,rm; .,rlqsu A

Each N is sp2 (AB3) so the molecule consists of two (with N at centers) triangles joined at a corner oxygen. Free rotation is allowed. (c) HONO(HNOz) H - - = r N is sp2. Nonlinear

sp3 bent

"- lo-a,o

AB4 sp3 tetrahedron

mol-ecule with ONO angle of about of alput 1O9.5o. Freely r+Lating not coplanar.
( d ) F N N F : F - N = N - F : are about I20". N = N orra N ' s .." i"

12Oo and NOH angle so .four atoms are

,,o
- ,O

"

"p2

so NNF angles is or

(b)

Seo2

g"

A B 2 Es p 2 b e n t
A B 2 E 2s p 3 b e n t

coplanar opposite

with either (trans. ) .

both

F's

rigid so molecule on same sj-de (cis)

(c) cIOe

..4 .. , = ii - ,\7

(e) clsscl

,qi - : - : - qi'

s's are sp3. Molecule


at all joints. Aft

(d) so3

2-

O,

"

- 3O-

,j,o
..
l r

"

,O

A B 3 Es p 3 r r i e o n a l
pyramid

is nonl-inear and freely rotating angles are about 109.50.

(a) oXeFa
(e) H C N (f) H-C=N: AB sPlinear

/?l.. 1Cr - A \J XeO3 v:O" - Xe \-/ - , " t


:O:V

or

t = ii: = " l l
l l

.:

', tl l rii - iie - ii, " / t\ ,/

Xe is d,=p, 1or sp3d2),


AB5E square pyramid

'I'

,'f,.

O:

(probably pair)

most

stabl-e

with

O opposite

the

unshared

AB3E sp3 trigonal

Pyramid

( b ) ( H O )s I O , acid.

also

written

Hs106 and call_ed periodic

Lewis Note that the use of the alternative (d-p 1T bonding) with expanded octet structure of the s in the ileduction makes no difference 34

35

"

, fl
l

k1,', H I is d2sp3 AB5 octahedron around I with O at corners. freely from H atoms rotate 5 corners.

,rl.r Orbitalsr (ols Lz and O*ls

plT-dT Bonding are Bee shown only for Li2 82 to C2)


w2

H - O - I - O .. ,/t\
. t a l ' .

/
H

//' :u:
I H
l

\
\

.. n

+
1t2p

+
1t2p

++
tt2p

++
Tr2p

.
tl

++
F: " : S ls dsp", ABs, trigonal (probably most bipyramid stable with O in axial position) .

(c)

OSF4

:F

- s -

++
o*2s

o*2s

++
o*2s

++
o2s

t+
o2s

t+
o2s

++
o2s

( d ) ( H O )' + X e O z n

++
_ ^.

/4, ,, -/ " -\. . ,,\'' 9' n" o"' n

., _

",i

?,

'n/"

o*1s t* ols
ors B . O .= 1 , diamag. N2

++
o*1s

t+
o*1s

t+
o*Is

t+
ols B . O .= 0 , diamag.

t+
ols B . O . = 1r paramag.

f+
B.O.=2, diamag.
F.^

Xe is d"p3, A86 octahedral arrangement of Xe, 4 dangling Hrs freely rotating. central 7.16

and thus The Cl bonding pai-rs are more voluminous, are more Iike1y to be equatorial where there are o pairs at 90o, rather where 2 neighboring than axial pairs at 90o. One could there are 3 neighboring argue that having them both axial would minimize repulsions but the between the C1 bonding pairs, of 90o neighbors is more important. consideration likeIy arrangement is to have the Cl bondq The least (one axial at 90o to each other Cl and one equator
a'] \

o-rp
,' *Zp tT-rp

oup t f _ fi*2p r*2p

o-rp f+ r*2p f+
ffrp

t+
1T* 2p

;-r

t+

++
lfrp

f+
Tf2p

++
T2p

t+
T2P

t+
7.L7
The unshared pair drives the X's close . be in an effect which will together, I most pronounced for X = Fr because the X: P - F bonding pairs occupy the least least resistance to being pushed volume and offer together.
P I 2t: .

:X -

o2p

++
62p

t+
o2p

t+
o*2s

++
6*2s

++
6*2s

++
o2s B.O.=3r diamag.

++
B.O.=2, paramag.

++
62s B . O . = f , diamag.

36

37

7.I9

(a) Hz

(DJ

+
il2

(c)

HHe

(d) Hez

(e) Hez+

w2

+
o-1t

+ t
o

+ +

+
oJ(

*
,lTt

t+.
-1 ^ v ! >

+
!

02

1+
T* _L 2 firt

fI t

BO= 7.2A

t+
o o
'II 2 isoelectronic

+ Oz'

w2

oz2-

t+
1T
Ork O*
d *

++

t+
TT

++
1T

o?t

t
1T.-

+
TiT'IT

+
1T*-

+
F-nf-

t
tT.-

f+
,lTt

t+
TfJ<

(b) B O = ( c ) C2 is

3
(if we disregard the order of

1I

++ r + f + f + + + + + J + ,TT
1T 1T 11 1T

++
1I

o and n orbitals) N2 and CO are the

with N2, NO+, CN, and CO. neutral isoelectronic molecules.

++
o BO=2 paramag. 7.2L N2

t*

-;-

^-t,

t+
1 diamag.

co
dtk

co'

co

No

2 parama9.
N2

L2

paramag.

Oz

Qz

tT-

tT-

r+
t
1T.
T:t

++
t+
U

+*
t+

parama9.

t+

t+

f+

++
t+
z

++

t+
1I

,IT

t+

t+ ++
z
^ 1

B o = 3 + NO

paramag.

paramag.

++

t+
z
a

t+
l a2

NO

B O = 3
Since shorter distance than

N2 is B-o. \,rith inversely varies . | . + 02. than is shorter N2' and 02'

++ +l
B O = 3

++

++ ++

t+

2 paramag. 39

3B

7.24

4..

\J:O

N -

O : < - - +: O = N - O : V

..a

The resonance the two between

show the negative charge shared between oxygen atoms and the bonds intermediate and double bonds.

which j-ndicate some multiple bondinS (d_p pi bonding) thus shortening the bond length. Note that since these four forms put negative formal charge on adjacent atoms they must be very minor contributions. a 7 , -'I PHs Expect bond length found experimentally. = 1OO + 32 = I42 pm, jusr as

Alternatively we can say there is a sigrna bond between N and each O and that 2 electrons are in delocalized TI bond connecting all three atoms.

7.2s O,

- O = : + - +, = @ - , O
and a double bond.

" o , r ui s b e r w e e n

a single

O,6 - S@= : +-+,= B@!i,g


caser however there is a preferable has no formal charges allowable for sulfur, use its d orbitals to possible for oxygen. contribution from the

PFs Expect bond length = 1l_O + 64 = I74, than found. The actual bond is shortened bonding using d orbitals on p (3rd period) suggested by three resonance forms like
:F

much greater
Lr<r zl -n ^i V J V } / } J

AS

s i m i r a rr o o z
structure whi

:O = S = O . This is a 3rd period element, which exceed the octet rule, not Furthermore, a slight minor resonance forms:

- i l o =F o t "

, = O =o , @ - * O : o= O ' s= ,
suggests that the bonding is even stronger than bond, so that it is even shorter than the d.ouble distance. There are also contributions from the forms:

O , a - ' s = o , O . . - * O=o - , O :B
which average to the double bond. by sulfur, when the octet rule is attributed to d-p pi bonding. 7.26 In addition to this structure, Multiple exceeded bonding is

,,V

," O -s L- ,O Ii' t9r silicon ru1e. period a third element may exceed the octet There are four minor resonance forms such as

:o:

O . 6 - , J O -a , O
t

il

r A

"

40

4T

C}IAPTER B
lsJJ

(a) (c)

6.55 atm. 4 B 5 Ko r 2 L 2 a C

(b) 3.45 atm.

r J ' 1,., l

cas

La\^I

Sj-mpl_e Gas Laws r t .L 2 8.1 (a) At varies (b) At varies (c) At varies constant inversely constant directly constant directly temperature the volume pressure. with its pressure with the of a gas u.l4 lt.L6

2O5 m1. 619K or 346aC


lst 3rd atm line line 22.4 l-Eer, 1.633 mole,

8.13

9,51 liter 0.496 atm.

the volume of a gas sample (Kelvin) absolute temperat

2nd Ij.ne 4th line

44.8 J,l-Ler, 27.5K, 5th line

IL.76

4.2

t"lL"""2,""V ,rt,L"
( a ) 7 5 0 m 1. (c) 15Oo ml . (b) 150 ml. G) L2.0 atm. (c) 4.O0 atm. (b) 0.120 atm. 1000 balloons 7.46 x 10-6 acm (a) 4.04 liter (c) 117.2K or -156"C
(b) 352Kor 79"C

volume the pressure of a gas sample (Kelvn) with the absolute temperat

t t. l ' l rl. L9

5 .97 liter .4'74 arJm.


-t

8.18 L.34s "+4.20 4.22


-l

Il_er
11ao^ - I

390K^ r 80.99

il. .,ll- 34.19 il . .13 58.09

mole mole

mol.e

8.3

1'"{-.l e 1 ' ri r r c i p
tt. .).4 30. O0 liters 3O.O liters l|. :i5 10. O liters CHa required, NH3 required, HCN 40.O liters ; - 2 . s liter N2 No(9) N2 45.0 liters 90.0 liter" 02 required, HrO(g) produced

A.4

8.5 8.6 8.7

t t . . ' . 6 4 N H 3 ( 9 ) + 5 O 2( S ) - r 4 N o ( 9 ) + 6Hzo(g) tt-.).J 4NHg(g) + 302 (g) + 2Nz (g) + 6H2o(g) r..lB (b) 2lBK or -55oC ;r..1 9 0.50 liter Cl2 remain NH3 1.50 liter

B.B

(a) 171 m1. (c) 3B2Kor 109"C

9.OO liter (a) (c) It, l0

3NO -> N2O + NO2 P",^


I\U2 N2U

(b) 83.4 mI total

8.9 8.10

0.916 m1. (a) 2.34 atm. (c) 223.5K or -49.5"C


(b) 1490K or L2L7"C

= p.. ^ = O.5OO atm.

(a) 4NH3(9) + 3F2(g) + NFs(g) + 3NHaF(s) (b) 1000 ml NH3, 75O ml F2 .

42

8.31 8.33

5.7L g liter-r (a) 1.20 x lora (b) 3.23 liter

8.32

58.0 g mole-l

rl.,t.t il.,t4 l t . , ' ' i

.506 liters 39.6? A1

8.43

I.694

liter

x 1036 molecules (b) L.25 x 10-6 moles So2 (d) 2.8 x 10-6g. 273K

8.34

(a) B.O x 1O-s S I (c) 2.8 x 10 atm (.)

(a) .125 moles hydrocarbons; . g 1 2 5 m o l e s oxygen; .5OO moles COz; .625 moles water (b) (c) 2 (hydrocarbon) + 1302 + BCO2 + ]0HzO

8.35

6 n"O = 8.OO x 10 atm at x 1Or7 molecules

2CaH1s + 1302 -+ BCOz + IOH2O .0175 mole, .00250 mole . O 4 O Om o l e (c) CzHre

(b) 2.15

t.46

(a) (b)

(c) B.O0 x 1O-+%

Stoichiometry 8.36

and Gas Volumes + 3 N e O ( g ) + N H 3( 1 ) + N a N s ( s ) + 2 N e( g ) + 3NaoH(s)

I,a!L of Partial tt-47 n*ro = .I94 atm., ll.48 tt.49


because one way.

pressures n*, = .306 atm. "Nu = .0625 atm. Xco, = .750 (c) 2.459 Coi 1I.559 CO2 B.5l 8.53 O . O 9 1 l liter .0469
4Ult

(a) 4Na(s)

(b) 12400 ml Nzo (c) B27o ml.

"".

= .1875 atm.,

(") ""o = .25O, (b) .350 mole

Note: Answer in text is O.K. but different (a) can be balanced in more than equation 8.37

(a) 2HCN(q) + No2(g) + Czlz(9) + No(g) + Hzo(g) (b) ml (c) HCN = 5600 ml. r m1. NO2 = 2BOO ml.

rr-')0 O.lBB liter tt.'i2 tt.';4 .727 atm

28OOml.

(a) L.379 moles of mixed gases


(b) 1.00x = moles 2x = moles of N2Oa remaining

B. 38

( a ) c a H z ( s ) + 2 H 2 o( 1 ) + c a ( o H ) z ( s ) + 2 H z G ) (b) 4.7o g

NO2 formed .75g moles of = .45O NO2

8.39

(a) ca (s)

+ 2 H 2 o( 1 ) + H ( 9 ) z + c a ( o H ) 2 ( s ) problem 38 because CaH2 alre

(b) 8.95 9; more than has H2 "in it. " B.40

.621 moles of formed. (c) X*o, and (d)

N2Oa remaining. .450 atm

X", ^

.550 atm

(a) Al+Cg (s) + I2H2o (1) + 3cs+ (g) + 4Al (oH) (s) (b) 1.OO liter

= '550'

B.4t

(a) Laz (Cz) s + 6HzO(1) -> 3CzHz@) + (b) 0.361 liter

2La(OH) (s)

'l"tic 11.55

fneory

of volume to the of the voh:ne molecules in which


ls the

(a) The actual smal1 compared confined.

negligibly gas is

44

45

(b) Molecules move in straight lines untiL collide with other molecules or the walls. collisions are elastic, i.e., total kinetic is conserved.

they A]-l energy to na

Relative number o molecule

igh

(c) The kinetic energy of a gas is proportional the absolute temperature and independent of the of the gas. (d) Attractive forces between molecules are

(b)

energ"r

neglig IOOK speed = 279 m sec 5O0K speed = 624 m sec


-l -l

8.56

Pressure is proportional to the rate at which molecules impact on the wal1s. If volume is re
by half then there will be twj_ce as many molecules per'unit volume and rate of impacts will be doubled so the pressure will be doubled, as predicted by Boyle's Law. When the temperature is of the molecules increases doubled bV . the'average speed The increased sp,

) )

l5B7K CO rate t6 .0 = L.25 x COz rate

affects the pressure two \^rays; (a) more frequent impacts, and (b) more momentumtransferred per so the overall effect is E x /l = doubling of pressure, as predicted by Anontonrs law.
Charles' Iaw can best be deduced by first considering the volume constant. Then a d.oubling of temperature doubles (as above) and in the pressure order to restore pressure the original the volume would have to be doubled (in accordance with Boylel law) "

| "

(a) 1.185 I lirer-r .676 g liter .9o2g liter-r 30.0 -

(b) 58.0

I Gases {'b Two main reasons arei (a) finite size of the molecules, and (b) attractions between molecules. (a) Hz cl-oser to ideal greater intermolecular because i{I attraction. is larger and has

8.57

t,l

Relative number o molecule

low

high

(a)

sPeed

(b) The gas at IOOoC will follow the ideal gas laws better than at 10OK because the energy of inter_ molecular attractions will be much less significant compared to the thermal energy (kinetic energy) ac the higher temperature. (c) More ideal at I molecules are closer effective. atm. because at and attractive lO atm. the forces are more

ro

47

(d)

The gas with

T.

= 1O0K is

more ideal

because

the

(.IIAPTER 9 t,TQUIDS AND SoLIDS

low critical temperature is an indication of weak lntermelecular forces, i.e., only a small amount of thermal energy is required to prevent coalescence of the molecules phase. into a liquid 8.68 (a) .514 literi .O32L liter (.314) is IesB t attractions are

lrrtermolecular

Attractive

Forces and completely nontemporary dipoles caused density on a time scale forces

(b) At 50.0 atm the actual volume (.514) because intermolecular ideal the dominant deviation.

London forces are universal directional and are due to by fluctuations in electron of o n l y . 1 0 - 1 s seconds.

At 80O atm the actual volume (.O42L) i-s greater (.0321) because excluded volume is the than ideal dominant deviation. (c) At 8.69 Expect 5O atm, 611; Clz "a" to have at B0O atm, L.3L2 Tc because greatest it has the inlermolec 't.2

highest indicating

are due to permanent dipoles "-,r." by unlompensated. polar bonds. The negati.ve end ot oie *oi..rrfu attracts the positive end of the other. Many molecules have no dipole moment and such forces ale absent. Usually London forces are dominant (see table 9.1, p . 2it6). ( a ) O F z m u s t b e b e n t (two unshared pairs on O atom) so the bond dipoles add vectorially. BeF2 is straiqht so the bond dj_poles cancel each other (b) PF3 is a pyramidarmolecule. rf you consider the unshared pair on p to be pointed up\^rard each of the PF bond dipoles has a downward component, which add together. BF3 is planar and the btns are at 12Oo so

Dipole-dipole

highest attractio1. 8.70

vaIue,

(a) 22.41 atm. (c) The van is slightly der Waals less than

(b)

21.93 atm. the pr

equation shows that the ideal.

8.71

(a) 2.241 atm


(c) The van der is very slightly relatively than the pressure is

(b) 2.236 atm


!{aals equation shows that the pres less than ideal; but closer in problem 70, because in problem higher and behavior is less idea1. (b) 30.48 atm.

ll"-:::"t.anr

(i.e.,

rhe vecrorial

sum) of rhe rhree

8.72

(a) 30.62 atm. (c) The van is slightly compared Lo is behavior

der htaals equation shows that the presg less than idea1. The difference is 0.4 2.L42 in problem 70. At higher tempe closer to idea1.
t- J

(c) S',* is distorted tetrahedron with the S atom practically on an edge (a1so sometimes carred a ,,saw "see-sahrr" horser" a or "aispirenoir,, double wedged shaped) so that the SF bond ipoles cannor cancel each other. SnF4 is tetrahear-af wit, sn at the center and the resultant dipole moment is ,ero. (Consider one pair of SnF bonds and observe that its resultant must be equal and opposite to the resu.l-tant of the other two.)
HgCl2 r CuCI2, BF3, CHa pC15 XeF2,

??

He has lower intermolecular by an a value of only .0341

forces than H2 as shown compared Eo .244 for [I1r

SFG,

XeF,.

- t - o ma " " .1r (a) 1.77 x 10

(b) .o4772

The only arrangement which has zero dipole moment both Cl atoms axia1. If one or both are eguatorial there must be a dipole moment.

has

A.15

.l5B nm

48

49

Al I Lrond dipoles cancel in pF5; the axial contri_ but-ions are equal and opposite, and the equatorial are trigonal planar (as in BF3). pF3 is a trigonal pyramid with all bonds contributing, as discussed in 9.2 b. 9.6 Both the N - H and the N - F bonds are polar, but the negative end of the NF3 molecule is directed awalr from the unshared electron pair and is partially cancelled by it. The negative end of NH3 is in the direction of the unshared pair and the molecule dipo moment contains contributions of the same sj-gn from both the bond dipoles and the unshared pair. The case of phosphorus is different because p is much less electronegatj-ve than N. pH As a result bonds are practically non-polar and the dipole moment of PH3 is due only to rhe unshared pair. ;r' ;;;-"are more polar than NF bonds and make such a large contribution that the counter_effect of the unshired pair is relatively trivial . The argrment,s can be visualized in the figures below. NHs NFs
DH^

').9

As one goes to higher atomic numbers, the atoms (and consequently the molecules) get larger. Their oucer electrons are more diffuse and further from the nucleus. Such electrons are more easily subject to fluctuations and also more polari-zab1e ly ,r.igfrboring temporary dipoles. Thus London forces aie greater in the rarger morecures and the crystals can retain their structures to higher temperatures. Dipole*dipore forces exist only in those with a d.ipole moment: CH3C1 CH2CL2, and CHC13. Chlorine is a very large atom (compared. to the others) and London forces bet\reen Cl atoms in neighboring molecules probably dominate over other considerations. Thus expect that the boiling points will increase in the order written, with CHa far below the rest.

')'10

'l:lls Ilqgoge!_
, r 1 l

iond

zi" ,i\
9.7

1
i
p

,|.

/j\
o I t

HF ) NH3 because n' is so much more electronegative than N. Thus the H - F dipole moment is very large and causes the H bond energy to be very great. In both these cases the averagie nunber of H bonds per mol-ecule can be aL most l, because NH3 has just I unshared pair and H F h a s j u s t o n e H . Despite electronegativity considerations, exceeds the other two because the average H b:nds per molecule can be as high as 2. HFe can be thought of as being formed H2O number

of

Assune that the compound.s are all straight, and isoelectronic except for the S core. Then CS2 like co2 has zero dipore moment because the cs dipores are equal and opposite in direction. The o ltom is more electronegative than S and so the CO bond dipole is greater than the CS and gives a resul_tant equal to the difference, with the negative end toward O.-

by a hydrogen H i,

bond between HF and F , i.e., but in fact the is slmmetrical! HF2 ion (bette;

,F,O---written

FHF-;

9.13

( a )r - - n

o a

= o, @ rn. r,ewis structure Q" reveat-s formal c which act to produce a "dipole', in the opposite direction to that attributabte to the electro_
negativi-ty the result difference. The net of these contradictory sma1l dipole tendencies. moment i

I H

H bonding can occur both by (1) any H in NH3 joining an unshared pair on the O atom in H2O, or H2O joining the unshared pair on the

( 2 ) either N i n NHs. (b)


H I H - N -

H in

Any H in NHzOH can join an unshared pair on the O in H2O, or eifher H in H2O can join any of the three unsT-rared paj-rs in NH2OH shown above. ,

50

51

(c)

H I t . . H - C - O - H *

I
(d) H of either CH2O. 9.L4

The H on the O in CH3OH ( as H*) can join an unshared pair on the O in H2O or ei

t'lre Liquid '.I7

State

H in H2o can join an unshared pair in the O in CHTOH.


either H in CH2O can particL pate. But nonetheless the solubility of CH2O in H2O is greatly enhanced by n bondlng an unshared pair on the O in

H | .. C = o: H in H2O to

(a) :f the critical temperature is high Lhere musc be strong attractj.ons, sufficient to cause the molecules to coalesce into a liquid despite the high kinetic energy associated with high temperature. (b) At the surface a molecule has fewer neighbors than in the bulk. If intermolecular attractions are greac the molecule has a strong,inducement to move into the bulk phase and increase its number of nej.ghbors, Ieading to a high surface tension. Hign surface tension means a large tendency to reduce the surface_ to-volume ratio, i.e., to minimize the number of surface molecules rel_ative to bulk molecules. (c) rf one compares molecules of simirar size and shape, then a high viscosity indicates strong attractionsMolecules strongly attracted t each other will not move easily pasu one another and flow wilL be sluggish. Hor^/ever lh.." are many substances with littte intrinsic intermolecular attraction which are viscous because the molecules are very long and become entangled (e.g. petroleum tubricantsl. (d) e 1ow vapor pressure indicat.es Iarge inter_ molecular attractions which retard the escaping tendency. A molecule can escape only if it has enough energy to overcome the attractions of its neighbors.

One part of the explanation has nothing to do with H-bonding; it is that the HSO+- salts have much IeEa lattice energy to overcome than the SOa= salts because of the smaller anionic charge. H_bonding plays a role also. In both SO+2- and HSOa- the O atoms can all H bond to the H atoms in H2O, but the HSOa- has the additional advantage that its H arom (which is attached to an O) can H bond. to the O in Heo.

9.15

The boiling point data suggest that there is an additional force of attraction between the unlike molecules that doesntt exist in either pure liquid. electron density H bond formation
That additional attraction is an H bond between C and the O in acetone. The three Clrs pull enough

H bonding is of little importance because Cl is not strong donor, but the O in acetone is. Conversely none of the H atoms in acetone is "H bond active', sO H bonding is of no importance in pure acetone. 9.L6 Both molecules have permanent dipole moments and have similar london forces. H-bonding is present ln both and because of the 1arge H bond energy it is thr dominant force of attraction in both. The dif lies in that ethylene diamine has t\^/o _NH2 groups molecule hhich leads to twice as many average H b per moLecule and thus much more attractive force between molecules.

from C to render the H atom prone t with a strong donor. In pure CHC1

(e) Enthalpy of vaporization is a direct measure of intermolecular attractions ; the greater the attractions the more endothermi_c is the vaporization process.
(f) tne higher the boiling point the greater the attractj_ons must be. Boiling points vary in the opposi-te way from vapor pressure, i.e., lo", vapor pressure means high boiling point. (See part . ')-IB

The energy a molecule must have to escape from its neighbors is much greater than the average energy. The Maxwell-Boltzmann curve shows that at any one time a small fraction of the molecules have sufficient energly to escape. These are the only ones which escape, and so the average energy of the remainder must decrease. If average energy decreases, temPerature decreases.

52

53

, )

] q

An equilibrium state is one in which the quantitiel of substances in equili_brium undergo no change with rf a liquid is placed in a Jrosed container !1*.: fi11in9 it only partially, the space above will populated with molecules (of the liquid substance) the vapor phaseThat population remains constant with time as indicated by the constant pressure of the vapor, Actually the equilibrium state is a dynami-c condition in which many molecules leave the liquid phase per second and an equal number of molecules (not necessarily the same ones) return tO the liguid per second, thus maintaning the popula constant. The boiling point is that temperature at whi_ch the vapor pressure is the same as that imposed on the surface of the liquid. Vapor pressure is a strong function of temperature; if the pressure on the surface is increased, raising the temperature will increase the liquid,s vapor pressure untiL it the imposed pressure. Estimate the boiling points as closely as possibll from the graph; there wil_l be honest differences between observers. Approximate values are: 19o, 6tc 82".

100 10
P/Atu.
I.U

.L75 .100 .0013


L99 -L69 -L57 -L52

9.2O

r/'c
-oJ

Solid Xr is denser than liquid at l.O0 Atm. This foLlows from the positive slope of the liquid-solid line which is "normal.', An ,'abnormal" substance like r^rater is denser in the liquid state and corresponds to a negative slope. (a) Minus loC. The pressure is initially so low that the entire sample is vapor. Vfhen the pressure reaches about .006 atm the sample condenses to solid ice. Eventually at very high pressures (ru100 atm) the ice wj-l1 melt as you cross the almost vertj_cal solidliquid line. (b) 5OoC. When the pressure reaches ,r,.12 atm the vapor condenses to a liquid and undergoes no further phase change. At exceedingly high pressures (> IOTOOO atm) there are different forms of ice which are stable as hi-gh as 50o and more (not shown in fig. 9.9). (c) Minus condenses superhigh ice which '.25

9.2L

(a)

.010 atm: 7.1oC

(b)

.025 atm: 2L.30C

Phase Diagrams a )')

1 ? L

10.0 P/Atn. 1.0


'I

5ooc. At tu 4 x 1o-5 atm. the vapor to a solid, which never melts. Ho\,rever at pressures there are other exotic forms of become stable.
- 3-

.01 10
Note:

(a) t x 10 and remains

atm. At -10oC the so as it is heated. -lOoc At

sample

is

all

vapor ice) and and

20

30

The pressure scale is non_linear but approxfin order to-shov aff T1t:lyr e slogarirhmic teatu c1earlv.

40

T/x

(b) O.5 atm. At at tuOoC it melts. remains vapor.

it is solid about g2oc it

(ordinary vaporizes

54

55

(c) 1.1 atm. At 'r,0oC (just slightly it 9.26 vaporizes

IOoC it is ordinary ice and at less) it melts. At about lO3oC and remains vapor.

').29

(a) Minus 60oC. This is between the normal sublimation temperature and the triple polnt so at about 4 Atm. the gaseous sample condenses directly to solid COz and remains so. It can never become liquid because the solid-liquid line slopes away in the normal fashion. (b) ooc. This is far above the triple point but below the critical point so at a sufficiently high plessure (n 30 etm. ) the sample condenses to a liquid and remains so. (a) 2 atm. ("dry The solid -'72"C and remains gaseous. ice") sublimes at about

Yes, ice can be purified by sublimation in an apparatus in which the pressure is always less than the triple point pressure of about 0.006 atms. Likewise the condensj-ng surface would have to be below the sublimation temperature for the pressure (Given by the vapor pressure used. curve for iee-)

'l'ypes of 9.30

Crystalline

Solids table 9.4 in the first three

The answer appears in columns on t h e l e f t . (a) Si, a netvork

9.31

crystal metallic crystal

, covalent bonds. of

bond.s.

9.27

(b) Ba, a metal, (c) (d) Fz, molecular London forces. BaFz, j-onic,

non-polar

molecules,

(b) 6 atm. This is above the triple point so the solid will melt to liquid COz at about -56oC. Then at about -53oC the liquid will vaporize and remain vapor. 9.28 Consider the solid and 1iquid forms of X in equilibrium: X(s) i=+ X(1) If, as is usual, the solid is denser, then an incr in pressure will favor formation (Le of the solid principle). Chatelier's The sketch on the left be sho\^/s lhat this will be so only if the slope of the melting point li-ne is positive. The opposite situation, which prevails for water, is sketched on
+ha ri h+

electrostatic non-polar, po1ar, non-polar, non-polar, non-polar, bonds.

attractions. London forces.

( e ) B F 3 , molecular (f) e.32 P F 3 , molecular

London and dipole-dico1e. London London forces. forces.

( a ) o z , molecular

( b ) B r z , molecular
(c) (d) (e) (f) 9.33 (a) (b) (c) (d) (e) BrzO, molecular Ba, metal, BaBre,

London forces.

metallic

ionic,

electrostatic electrostatic

attractions. attractions. as well plus as stronqer dipole-

BaO, ionic,

BrF has stronger L o n d o n forces dipole-dipole forces. BrCl has stronger London f o r c e s d.ipole forces, which C12 I a c k s . Cssr is stronger

ionic and most likely high forces than BrCl which is compared to (London only).

melting; polar molecular. Br2 which is

Cs which is metallic, non-po1ar molecular

C diamond which requires the breaking of bonds. C12 non-polar mo.lecular requires coming only London forces.

covalent over-

56

57

9.34

(a) (b)

sr; it requires breaking metallic bonds whereas C12 requires overcoming only London forces.

9.40

SrClz; ionic crystal. SiCla i_s a non_po1ar mor-ecule requiring overcoming only r,ondon forces (c) SiBrr+. Both are non_po1ar molecular but SiBra must have greater London forces because bromine is a much bigger atom than chlorine. (d) SC14. These seem close, but SC14 is polar while SiCIr is not, so there is the additioial ipofe_ dipole force. SiC is merely London f,i. a covalent net\,rork sol_id while Siclk is a non_polar molecular solid bound or.rfy y forces.

M = 5 5 . 7 7 g mole-r (The element is undoubtedly for which t h e a c c u r a t e atomi.c weight is 55.85.) M = 4 0 . 1 0 g mole-r 316 pm (probably 9.43 Ca, M = 40.08) 389 pm

iron

9.4I 9.42 9.44


O /'1tr

N = 6 . 0 2 x 1023 atom mole-t Cube edge 124.5 pm = 2.I7 cm 9.46 9.48 I27.6 pm

(e)

9.47

314 pm

X-Ray _Diffraction 9 -49 9. 5l


o q,a

gf

Crystals

9.35

(a) (b) (c)

Metallic b o n d s ; v e r s u s H 2 o n l y London LiH. Ionic crystal r e.l_ectrostatic forces; H2 only London forces. LiH. It is difficult to versus metallic crystals.

force ver

348 pm 70.8 pm

9.50 9.52

222 pm 211 pm

choose between ionic Hoi^/ever LiH wi-th verv lattice ene n melting

f o r n = 1 0 = 14.9o;forn=210=30.9" f o r n = 1 r 0 = 16 . 1 "; f o r n = 2 , 0 = 3 3 . 8 "

sma11 ions, Li- and H , has a large and ranl<s high among ionic .ry=tui" poant. Li ranks l the the (d) (e) very 10w density electron interj-onic space.

9.54

Group because r) "?'.ililnrl:;:t:.ff.ulrllt"l


cl0ud which but plus fi1ls C12 has dipole_

toni-c Crvstals

e .s s

Clz. Both are non-polar molecular much greater London forces. tCt. It has greater London forces, dipole forces which H2 1acks.

( a ) The cesi.um chloride and o n e c a t i o n .


(b) 4L2 pm

unit (c) unit (c)

cell357 pn

contai-ns

one anion

e.56

( a ) The sodium chloride and f o u r c a t i o n s . (b) 555 pm

cel_l contains 278 pm

four

anions

Crystals 9.36
o ?o

1.453 g cm

3 9.37 3.598 g cm in po must crystallize

e.57 3 a simple 9.58

( a ) The zinc sulfide and f o u r c a t i o n s . (b) 541.6 pm


(a) (a) (a)

unit (c)

cell

contains

four

anions

_ r.uuu acom. cubic lattice. n = 4 atoms. cubic Lattice.

165. B pm

384.5 pm 594 pm 584 pm BaO, CaS, NaBr,

(b) 7.01 s crn tD, /.59 g cm (b) 4:82 s cm 3 AgC1, KC1 59

9.39

Au must crystallize

in

a face

centered

q ( o

9.60 9.61

1\IJ,

5B

Defect Structures
9.62 Dislocations. Planes of atoms are not perfectly parallel. for example where two crystals join; or plane of atoms extends an extra through a portion of a crystal. defects stoichiometry. in interstitial vacant. Impurity lattice 9.63 atoms. points, point arising Extra sites, from deviations from perfect anions or cations may be lodg or some lattice points may be frequently be found on for the proper ions. (b) 8.244 cm-3

r ' I I A P T E R1 O J;OLTIONS

'l'lre Solution

Process

The forces of attracLion are less in pure Br2 (1) than in pure Iz (s), because of the lower atomic nurnber of Br, corresponding to lower London forces. lo.2 (a) CH3OH, which which can hydrogen-bond separates into ions, to H2O. each highly H2O

These will substituting

(a) 0.5e" oxygen vacanciesr (c) 8.239 9 cm-3

(b) NaCl, hydrated.

(c) CH3F, which is more polar, with regard to intermolecular (b) (b) 7% cation vacancies ion vacancie

thus more like attractions.

9 .64 9.65

(a) 5 .64 (a) 58.394

L4e" chloride

Whether one melts or dissolves 12 the same intermolecular attractions must be overcome; furthermore the interaction of Iz and CCla molecules is slight and not very different from the Iz - 12 interaction in liquid iodine. However when ionic substances dissolve in water, very large hydratj_on energies are liberated to compensate partly or entirely for the energy required to melt the crystal. |().4 lo.5 Smal1 i-ons, 1a Fe3+, but this + (b) Li', (c) (d) highly charged ions. Also slightly smaller,

more highly charged. is a minor difference. much smaller.

F , smaller. SN2+, a bit smaller. of both smaller size and sreater

(e) e13+, charge. z +' , (f) MS

because

much smaller.

Water usually appears in hydrates as water of hydration of the positive ion. It may also occupy interstitial spaces in the crystal structure. Finally, it may be H-bonded, particularly to an oxyanion.

60

6I

10.7

First one must consider the enthalpy of solution neaf saturation. If it is negative, i.e., the solution process is exothermic, then raising the temperature }owers the solubility in accordance w"ith Le Chate principle. By shifting back to the left the reaction will absorb the heat one supplied in attempting to raise the temperature, -47 kJ mole-r
- /U5 KJ _ MOJ-E -l

10.22 10.24 I0. 25 10.26 t.o.27 |o.29

14.89 KMnO4 (a) 57.69 (a) 5.51 M (a) 3.t76 3.35 M 25.5 ml 0.316 M X solute (a) (b) = .0843, M

10.23

5O.69HBr

33.7

(b) 33.9ml (b) 6. 93 m (b) 3.60 m to.28 10.30 10.32 X solvent = 0.789 M B.lt ml

LO.B
IU.I-U

10.9

-15 kJ mole-l

This

applies to + + Na'(S) + Na'(aq) cr (S) + c1 (aq)

value

the

combined

processes: 10.31 10.33 process of other to energy. 10.34 O . 4 O OM .9157

which release energy; as well as the separating rrater molecules from each accommodate the ions, which required IO.II-7g2 kJ *o1.-l This value applies

2 N a 2 0 2( s ) .0733 M

+ 2Hzo(I)

+ 4NaoH(aq) + 02 (9)

Pressure
to the combined processes:

Boi 1i

Point
c.1 A

of

Solutions

n* (9 ) + nb+ (aq) F 10.12 10.13 (g) +F (aq) 2.L29 CO2 0.1809 N2O

.265 Atmos. .171 Atmos.


tQ. q _ mole -l

10.36
r-u.5t

0. 886
- t

.0481 moles, .00409 moles,

10.40

Y-

g moJ-e

(a) P_ = 1.117 Atmos.


A

(b).838 ConSentratj-on_of IO.14 Solutions tt).42 Temperature-independent measures are: percentage composition, mole fraction, and molality. Tempera dependent measures are molarity and normality. M=><rn (1-yl100) x(cH3oH) = .294 x(Hzo) = .706 (a) .434 Atmos. ideal (.434).

(b) Actual pressure (.400) is less than Negative deviation from Raoultrs Law.

10.15 10.16 10.17 10.18 LO.2O

(c) Heat must,be evolved when the liquids are mixed since intermolecular forces are greater than in the pure liquids. (d) Since the vapor pressure is lowered a higher temperature is required to boil and so, if an j_t must be a maximum boiling azeotrope is formed, azeotrope.

X ( C e H s o H )= . 2 L 7 ; X(urea) 13.3e" = .0323

X ( C 2 H 5 O H= . 7 8 3 ) IO.19 1O.2I 7I.4% 7.01q KOH

62

10.43

(a) .497 Anos. (b) Vapor pressure shows a very deviation from Raoultrs Law. strong positive

10.65 T=27.1Atmos. 27.f Atmos. on the salt water would equilibrium in the rate of flow of hrater between one side of the membrane and the other. pressure would be necessary _Additional to favor the flow from the salt side to the fresh side, the rate of flow increasing with increasing added pressure. A pressure of just maintain

(c) Heat must be absorbed upon preparing the solution since there is less force of attracti-on between molecules in the mixture than in the pure liquids. (d) Mixtures relatively boil easily. form a minimum boilinq azeotrope. These two could,

LO.44

(a) 9og mole-l

(b) .611 Atmos. 10.46 10.48 14.89 -11.8%C,/m

sglut-isJnq 10.66 10.67 10.68 i tf

o-f_ Electrolytes = 3 ions = 2.67 = -.40oC = -.242"C = 3.5% per molecule

10.45 333 9 LO"47 - 1 . 7 5 " c 10.49 L22.59 mole-r

l-0.50 I54.3 9 mole-l to 176 .200 Atmos. O.7g7 x lo-s

1 0 . 5 1 J-75.69/moIe round off

1.O.69 tf 10.70

ro.52 1a) PUz = .800 Atmosr nO,


(b) xwz = L.487 x lo-s,
(c) -.00236oC 10.53 10.55 10.57 2.77e"C/m 161.14C 69.g g mole-'

% ionization

*o.

10.54 10.56 10.58

27.0 g 62.A g mole 186 9 mole-l I

Osmotic

Pressure

10.59 10.6I 10.63 L0.64

2.9I Atmos. 6 7 , O O O Im o l e - 1 (a) .01055 Atmos. 1BOg mole-l

10.60

33.6 9

IiiiiO.62 334 g mole-l (b) 109 mm

64

65

(l t^t,,t'ilR f f REACTIONS IN AQUEOUS SOLUTION

Oxidation 11.5

Numbers Cl03

c15+;
r.l 1r.

Cl0z Nos MnOa Crzoz 23-

CI3+;
I\Tq+.

OH
\T^^ L'v

N1f;
\ r :3 + ; ,"

cl0 lletathesis 11.1 Reactions + 2H- + HzS 1g) + zn2+ + srco3 (s) C1O'+ 2CrOa 2-

CI1+

(a) zns(s)

(b) sr2+ + co32 (c) lo reacti-on

soe 11,6

54+;

AsOa

C4+ 2cr6+; so,* s6+; 3po,* p5+. As5+;

Mn7+;

COs-

(a) 6+, (b) 6+, (c) 5*,

(d) 1+, (e) 4+, (f) 2-,

(d) g2* + 2OH + Mg (oH) (s) 2 (e) po,*t- * L.2 (a) ec+ I 3H* + Hpo+ -' A9I (s) I1.B 11.7

( s ) 3 * , (h) 5+
(a) 2* t

(b) r-,
(h) 6+ (b) 3f ,

(c) 6+,

/Al

(e) 5*,

(f) 6+,

(s) 5*.
(a) 3*, (s) 3t,

(b) sos2* + 2H+ + Hzo + soz (g) (c) No reaction + N i 2 + + s o + 2 -*


+

(c) 4+,

(d) 5+,

(e) 4+, (f) 5+,

(h) 5+ (b) 6+, (h) 6+,


(c) ti I

(d) sr2+ + 2oH

srso+ (s) + Ni (oH)z (s)

(e) OH + H' + HzO 1.3 ( . ) p b z + * H 2 s( g ) + p b s 1 s + 2 H + ( b ) g a 2 + + s 2 * z n ' * + s o a 2 - - * B a s o +( s ) (c) o reaction (d) Hgzcoals) + 2u- + 2cr -> Hg2cr2(s) + H2o + coz (g) ( e ) F e ( o H )g ( s ) + H 3 p O 4+ F e p o +( s ) + 3 H 2 o L.4 ( a ) 2 N H a ++ S o 4 ' - * (b) No reaction
(c) Cd-'
)+

11.9

(a) 5*,

4*, 6+

(d) 6+,

(e) 6+, (f) 6+,

(s) 5*,
+ zns (s)
Oxidat-En-Reduc 11.10

ti o-n Reac tio_n s reducing the

(a) zn is oxidized from 0 to 2+; Zn is the agentr C12 is reduced from O to l-; C12 is nvili oi -n . rgenc.

u'* + 2oH -> caso,+ s) + 2Hzo ( + 2 N H s( g )

(b) ReCls is reduced, Re goes from 5+ to 4+; ReCl5 is the oxidizing agent. SbC13 is oxidized. Sb goes from 3+ to 5+; SbC13 is the reducing agent. (c) Mg is oxidized from 0 to 2+; Mg is agent. CuC12 is reduced, Cu goes from CuCl2 is the oxidizing agent. the reducing 2+ to 0,

+ S-

a-

- + C d S( s )

(d) Fez (cos) s (s) + 6H+ + 2Fe3t + 3co2 (9) + 3H2o p b 2 + + s o 4 2 + p b s o , +( s ) (e)

(d) No is oxidized, N goes from 2* Eo 4+ NO is the reducing agent. O is reduced from O t o 2 - ; 0 2 i s t h e oxidizi-ng agent. (e) I{O3 is reduced; W goes from 6+ to 0; WO3 is the oxidizing agent. H2 is oxidized, H goes from 0 to l+t I+^ i q J-Lro ralrri ..J agenr..

66

67

It.lI

(a) Clz is reduced from O to 1_; Cl2 is the oxi-dizing agent. NaBr is oxidized, Br goes from _l to O, NaBr is the reducing agent. (b) Zn is oxidized from agent. HCI is reduced, the oxidizing agent. O to 2+; Zn is the reducing H qoes from 1+ to 0; HCI is

1 1 .1 5

(a) (b) (c) (d) (e)

24Ag- * 4AsH3 + 6H2o -> 24Ag + As4o5 2Mn2+ + 5Bio32NO + 4No3 2MNOa f + 14H+ + 2Mno4

24H+ + 7H2o

+ 5Bi3+

+ 4H-r + 3N2O4 +

2H2Q

11H+ + 5HCN + 5I

-> 2Mn2* + BH2o + 5ICN

(c) Fe2O3 is reduced, Fe goes from 3+ to 0; Fe2O3 is the oxidizing agent. A1 is oxidized from O to 3+i A1 is the reduci_ng agent. (d) OF2 is reduced, O goes from 2+ to 0; OFz is the oxidizing agent. H2O is oxidized, O goes from 2- to 0; Hzo is the reducing agent. (e) HgO is both oxidized and reduced and is both reducing agent and oxidizing agent. The Hq is reduced from 2-r to 0 while the O is oxidizd from 2- to O. The reaction is usually called a "decompositj_on" rather than,,oxj-dation_reductionr,, which it is techni-cal1y. I.I2 (a) 2H2O + 4MnOa Cr2o72 + 3C102 -+ 4MnOz + 3C10u + 40H

3zn + l-2H+ + 2H+ + 2ro3 3Se + 2BrO3

2:H2Mooa-> 3zn2t + 2Mo3+ + BH2o + 4cl + S2o32- + 2so+,2Br 2Mn2+ + 3HeO * 2rcl, + H2o

Il-.16

(a) (b) (c) (d) (e)

+ 3H2O + 3H2SeO3 +

5H3ASO3 + 2MnOa Hsroo + 7r

+ 6H+ + 5H3AsOa +

+ 7H+ + 6H2o + 4r.2

12n+ + 3pb3oa + 6Heo -r 6pb2+ + 3pbo2 e r+ C103 + 3H2o -> 4zn2* + + loH+ NH++ + 3H2o + 3H3AsOa+ Br 2zo + 2Se + HSoa -) 6Cl_ + 2HReO + 6H+ + 7H2o + 2cr3+ + 2Mn2+ + 6H+ -> 3rz + cl

1 1. 1 7

(a) (b) (c) (d) (e)

4zn + No3 3H3AsO3 f

BrO3

2H2SeO3 + H2S + H* + 4HzO + 2ReO2 + 3Clz

(b) BH+ * (c) (d) (e) 1.13

+ 3H2s + 2cr3t

+ 3s + 7H2o + lOH+ 4H20

6H2O + p4 + IOHOC1 + 4H3pOa + 10Cf 3cu + 8H- + 2No3 PbOz + 4Hf + pblz * 3cu2f + 2NO +

11.18

(a) 6Fe2+ + cr2o72 (b) (c) 5HNO2 + 2MnOa

+ l4H+ + 6Fe3t

+ H* + 3H20 + 5NO3

+ 12 + 2H2O + Sbao5 + 4No + 2H2o

3 A s 2 S 3 + 5 C - r - O + 9 H 2 O + 6 H g A s O , ++ 9 5 + 5 C 1 * 3N2Ha + 6HeO + 2I + BH+ 3cu2* + 3N2 + 2No + 4H2o + 4H3pOa

(a) 4sb + 4H+ + 4No3

(d) 2IO3 (e) 11.I9

(b) Bnar + 5H2Soa + HzS + Atz + 4Na2SO4 + 4H2O (c) 2IOs + 4H2O+ 5SOz + Iz * 5SOa2- + BH+ (d) 2NF3 + 3A1C13 + Nz r 3Cl2 + 2A1F3 (e) AsaO5 + 4CI2 + IOH2O -+ 4H3Asa + BHCI L.L4 (a) 6Fe2+ + 6H+ + cro3 (b) + 6Fe3r + cl_ + 3H2o

3cu + 2Nog

(a) p,* + IOHOC1 + 6H2O -> lOH+ + lOC1 (b) xeo3 + 6Hr + 9r (c) 3uo2+ + cr2o72 + xe + 3H2O + 3I3 + BH+ *

3uoz2+ + 2cr3r

4H2o

(d) BrO3 (e)


LJ-. U

+ 3!2C2Oa -> Br + 3Te + 4H+ +

+ 6COz + 3H2O 2lzo + 4NO + 3TeO2 + sHgr{'* Br, + L2H2o

3pt r 16H- + 4No3 + lBCl + 3ptC1s2- + 4No + 8H2O (c) cu + 4H+ + soa2- + grr2+ + so2 + 2H2o (d) p + pbo2 + 4H+ + 2Soa2 + 2pbSo+ + 2H2o (e) MnO2 + 4HI + MnI2 + 12 + 2H2O

4NO3

(a) 24H- + 34r (b) Mnoa

+ H9s(roe)z

+ BH+ + 4Mn2+ + I5l2p2oz2+ 4H2O

+ 5MnHzpeoz3-

(c)

3Cs(NH2)2 + 4ero]

+ 3H2O'->3CO(NHz)z + 3SOa2

+ 6H- + 4Br

69

(d) 2rJH* *

5co(No2).t-

llMnof

L4H2o + 5co2+

11.25

(a) gat (b) 6oH

+ 3NO3 + 5OH + 2Nj-2+ *

+ 1 B H 2 O- + B A 1 ( o H ) , *

?I\TH ^

+ 3ONO3 + 11Mn2+ (e) zt+ + 2cNS + 3ro3 + 3IC12 (f) 26H2o * 2Crr3 + 2lCI2 + 42CI LI .26 II .2I (a) (b) (c) BOH + 52- + H2O + 3CN 4eu + BCN + 2OH * 4I2 -> Br 2MnOa + So,*2- + + 4H2O -+ 3CNO _> 2CrO+t* 610r+ 52H+ + 6c1 + Heo _F 2cN + 2soa2

Br2 + 2NiO(OH) + + 2s2- + 3H2o + 2So4'- *

2 H 2 O + 2F,r

(c)

35 + 6OH -+ sosz-

(d) 1O0H + (e) (a) (b) (c) s2pS +

4I2 + S2o32 -+ 4oH

BI-

5H2O

4Ho2

+ so42

+ 4H2O2 -> AHzO + pbSOr+ -t 2CrOq BH2O

2Cr (oH) 3 + 3H2O2 + 4OH 2MnOa

2MnO2 + 2OH + 4OH

* 02 +

2H2O -> 4Au (CN) 2 2H2

(d) Si (e) 17.22

+ 5H2o2 + 6H+ -> 2j/rn2t + 5o2 + B H 2 O (d) Ag2O + H2O2 -> 2Ag + 02 + H2O

+ H2o + SiOg2- + + 3Bro

2cr (oH) g + 4oH

+ 5Hzo +

2croar-

3Br-

/\cids 1.I.27

and Bases;

Basic

Oxides

+ 2AI + 6H2O + 2A1(OH)4 + 3H2 (b) zol- + s2o3 2- + A}CI- + 2Hzo + 2soa + AcI(c) Iz + jCIz + IBOH + 2H3IO52+ 6 H 2 O+ 1 4 C 1 (d) 2Bi(oH)3 + 3sn(oH) +2- * 2Bi + 3sn(oH)62(e) 3NiO2 + 2Fe + 6H2O -f 3Ni (OH) 2 + 2Fe (OH) 3 (a) 5HctO2 + oH ( b ) B M n O , ++ B O H (c) + 3Hzo + +I 4CIo2 + cI +4H2O

(a) 20H

Hydronium is usually considered to be H3O+, though in fact more water molecul-es on the average are associated with it. Tt is impossible for H* to exist independently since there are no core electrons, and so the field around the tiny proton is so intense that the H+ will stick to anything near it.
lM u r r^ ^rr^ + ; r c r ^h u ruu cid: HNO3 , HCl

11.23

I I I L.

te zA

+BMnOa2-+IOa + 2pH3 + 2lpo32 + 3H2

Polyprotic Normal Acid + 2H2o 1L.29 sal-t;

acid:

H2SOa H3 pO4 Na2SOa, KC1 NaHSOa KHC2O4

p+ + 2H2o + 4oH

(d) sbH3 + oH (e) CO(llHr, LL.24

+ 3H2O _+Sb(OH)a *

sal-t:

+ 3OBr

Coz + Nz + 3Br

(a) HwO3 + NaOH -+ NaNOs + H2O (b) 2HNo3 + Mg(oH)z + Mg(Nosrz + 2H2O (c) 3HNO3 + A1 (oH) 3 -> A1 (NOs)e + 3HzO

( a ) 4 l n ( O H )2 + 2 H 2 O + O z + 4 M n ( O H )s (b) 3c1z + 6OH -' 5cI r- clo3 + 3H2o (c) 2HXeO,+ + 2OH + Xe + XeO6a- + 02 + 2H2O Note: There is no unique ans\er to this problem; it depends on an arbitrary choj-ce of the 02 to Xe ratios. For instancef another answer is: 4oH + SHXeoa + 6H2o + 66,z + 3Xeo5+- + 5X"

11.30

(a) KOH + HBr -> KBr + H2O (b) (c) 2KOH + H2SOa + KzSO+ + 2H2O :XOH + H3pO4 + KpO+ + 3H2O

I 1..31

( a ) 3 N a O H + H 3 P O a+ N a 3 p O 4 + 3 H 2 O (b) 2NaOH + H3pOa + NazHpO+ + 2H2O


NaOH + H3POq + NaHzpO+ + H2O

(d) 2as + 6oH (e) 4on+

+ 2Aso33- + 3H, oz + 4so32+ 2H2o

2s2oaz

(c)

70

77

I I , l;l

()

bromic

acid

(b) potassium acid (d) sodium

bromate

Volumetric t-1.3B

Analysis 11.39 11 .41 11.43 (b)


tL

(c)

hydrobromic

nitrite 0. 3B5BM 4 L . 2 B e " M g ( o H )2 69.62 (a) .O44759 NaCl 0.03054M 72.5% H2C2O4 .1101M NaoH

(t,) potassium (f) (h) (i) 11,33 potassium sodium

hydrogen sulfite

sulfate (g) sodium hydrogen carbonate

l-1.40 II .42 LL.44

dihydrogen

phosphate

copper (II)nitrate . B9s%


a+ o+

(a) HCloa
(c) (e) HIO NaHSOs

(b) Hr (d) Hro3 (f ) ca (HCo32 )


IT.46 11.45 (b) 2O.0% Fe (a)

(a) MnOa + BH- + 5Fe-'

+ 5Fe"'

+ Mn-'

+ 4H2O

(S) Cu (woz) z IL.34 (a) H2O + C12O -> 2HCLO

- + 3 N z + 6 H 2 O + 2 B r r 3NzH+ + 2BrO3

(b) 24.0% NzH4

(b) H2O + N2O5 + 2HNO3 (c) SOg f H2O + H2SO4 H2O + HzCO

r1,.47
tl_.48

.341% Fe (a) (b) 12 * 2S2Os2 -> 2f


+ StOe u - -

(d) COz *

.L5B6g tz

( e ) C a O + H 2 O + C a ( O H )z (f) Na2O * H2o + 2NaOH Uquivalent LI.49 Il .35 (a) CIzOz, (f ) zno, 11.36 (b) N2O, (S) Kzo t1.50 (a) Ca(OU)z + CO2 + CaCO3 + H2O (b) COz + (c) (d) znO + 2OH + HzO + CO32 2H+ + Hzo * ,I-'* l.I.5f LL.52 11.54 1 . 15 5 . 11.37 An amphoteric oxide can be dissolved or base. zno in acid (see 11.36c) zrro+2H+*H2o+zn2+ Upon treatment wi_th base: 2OH -> Zn(OH) a2 r.t. 58 57. 86% Fe by either acid I t-56 tL.57 (a) 64.0 g equiv. (a) (c) .1200 N .01s0 M HCl 6.00M H2SOq3.00M' 57.0 ml HzSO'+ .465 N base (a) 90. 0 9 equiv. -l -1 (b) (b) (b) 1 acidic 3 acidic .0750 N H/molecule H,/molecule H3POa 2.00M 11.53 :' HCl 6.00N H2SOal2.0ON' H3PO'+ B.OON 1 (c) soz, (d) B2O3 (e) AleOg Weights_and Normal (c) Solutj-ons I/5, (d) L/6, (e) I/2,

(g) P+Ors + 6H2O+ 4H3pOa (a) L/a, (f) L/4 (b) L/6,

.409 N acid

BaO + SO2 + BaSO3

(e) FeO + 2HC1 + FeClz + H2O

ZrrQ + H2O i

73

CIIAI"]'ER 12 CHEMICAI KINETICS

t2.'l

--r_ Energy
4

tra, r

t I

Reaction

Rates, (a) rate

Rate

Equations

Ea,x
4

L2.t

= k tNol 2 [H2 ] are M-2 se"-l symbolizes [Hz] ,h"re the kr units mole liter-l

AH L _ eactants
l:l.B of the to the concentrations Rates are proportional only if po\er of their coefficients to the reactants Most overall step. occurs in a single the reaction consecutive the sum of several represent reactions (the 'rmechanism"). qtcns step single Note lhat only t\^o moleculest involve usually reactions reactions white most balanced three, occasionally .involve many molecules. because the collision are ineffective Most collisions "damaqe" the too small to i.e-r energy ((Earf, Even if the energy is great molecules. collj.ding with orienLed may not be properly enough the molecules key atoms and i-e., to each other to reactr respect bonds are in the wrong Places. does reaction feasible if a thermodynamically Usually noc succeed, it is because it is much slower than case the decompoIn this side reactions. competing be to bromate and oxygen could of perbromate sition by perbromate oxidation or the very rapid the trouble' By the same token' species present. of other agents' oxidizing probably are very fast perbromates

(b) k units Note:

M, molarity, = k'[No]2

(c) rate

= 2k

L2.2

(a) Rate of form. of C = ktAl tBl (b) k = 1.56 x 1o-2 M-l sec I
(a) Rate of form. of C = klBI 2

L2.3

(b) k = .033 M-r r."-t I2.4 (a) Zero order, k units are = .lOQ M sec r Rate order, M sec-l t:1.9 units of sec-l
-l

(b) First

k must be in

Rate of (c)

disapp.

= .0050 M sec-l k must = 2.5 be in units of M-l 4 x lO M sec-l sec

Second order, Rate of disapp.

12.5

(a) .OOBO sec-r M (c) .2oo M-l sec-r

(b) . O4o sec-l

l.l.10

12.6

(a) Rate(a)rzlnitial (b) Rate = practically

Rate = .l8B zero approach zero asymtotically)


1 , ,, ,

(Actually (c)

[NO] must

Rate(c)rzlnitial

Rate = .074 Rate = 4 times Rate = B times

(d) nate(d)rzrnitial (e) Rate(e),zInitial

Ll-Il

in both reactants (a) The activated complex contains and in by the energy Ea,f, enriched species' a single the to either revert that can easily a configurat,ion or to the Produqts. reactants

74

75

(b) Earf is the minimun energy necessary to raise the reactants into the activated complex configuration, or the minimum energy of collision which can result in product formation. (c) The reaction order s the sum of the dependencies of rate upon concentration, i.e., the sum of the exponent,s of concentrations in the rate equation. (d) A catalyst changes the reaction rate, speeding it up greatly, though very little may be required and it is not consumed in reaction. usually catalyst the

- k - 2[ N o 2 ] k r [ N 2 o s ] k - k 2 [ N o 2 ]- 1 [ N 2 o s ] + - k t h s [ N o ] [ N z g s ] kz [Noz] + kg [No] 12.L4 krk g [Nzos ] [N-o]kzlNozl + kg [No] of appearance of

Rate of dsappearance of A = rate products = (krkg/kz) [A]

12.15 Rate of change of

[o] = o = krloel

- kz[o2] [o] - kg [oe] [o]

(e) Chemisorption is the process in which molecules stick to a surface with considerable energy, comparable to chemical bond energiesChemisorbed species are usually very reactive. (f) The rate determining step is the one step in a mechanisi that goes so slow1y that its rate is essentially the reaction rate. (g) A reaction intermediate is a species which is quite unstable $ith respect to further reaction. It exiss in small concentration during the course of a reaction in which it is being formed and reacted h'ith rapidly. (h) A zero-order reaction proceeds at a rate independent of the concentrations of the reactants.
L2.L2 ICI ICl + H2 + HI + HCl + HI + HCl + Ie slow fast
HI

Uropping the k3 term, kr [Os] = kz [Oz] [O) tol = (krlkz) Iogl/[oz] - kz [oz] [o] + kg [os] [o] ' Rate of disapp. of 03 = kr [o] tvto terms above the first But as stated earlier balance out to zero and rate = ks [og] [o] I rate = ks [oe] $t/kz) logl/lozl = ( l k 3 , / k 2 )l o g J 2 / l o z ] = k [ o g ] ' / lorj

I:r.16

-T--

Net overall reaction is the sum of the above. cancels out; it is the unstable intermediate. L2.L3 Rate of change of [NO]. = O = k r [ N z O s ]

E r =1- 0
Ener

' k2 [No2] [No3] kr [NzOs] = Ir^ =

- ks [No] [Noe]

[rOs] (kz [NOz] + kg [No])

kr [NzOs] kz [Noz] + k3 [No] disapp.

Rate of

of Nzos = kr [N2o5] - kz [Noz][Nos]

eacEanEs

] ] = k1 [N2O5] _ kz [Noe kr INzos kz [Noz] + k3 [No]

76

12.17

Rate of

change of

[No3] = O = kl [NO] tozl - kzlNosj - kg [Nos] [No] k r t N O l [ O z ] = k z [ N o s] = (kt/kz)

Neglecting

the

k3 term: [No3]

tNol [o2]

Rate of

form.

of

NO2 = ks tNO:l [NO] = (krks/kz) 2 tNol [o2 ] order in 02 and second order in NO I2.2O

Many substancesi ca11ed "negative catalysts' are really inhibitors or retarders, i.e., they react with and remove intermediates. But retarders and inhibitors are consumed i-n the reaction. One can conceive of a true negative catalyst in a multistep reaction, if the catalyst enhances the formation of an intermediate not. present othen^/ise in the mechanism which is kinetically (i.e., stable separated by a large energy barrier from product). the final A homogeneous catalyst is present in the same phase as the reactants and is regenerated in that phase as products are formed. A heterogeneous is a catalyst phase, such as a solid separate catalyst for gas reactions. The surface attracts one of the gaseous and holds it in pl-ace until gaseous reactants the other reactant can contact it. In additi-on to facilitating contact between reactants the catalyst, in cases of chemisorption, will actj-vate the adsorbed species.

The above s as required. 12.lB Initiation

first

C l - z -> 2CI

Propagation

Cl + CH+ + HC1 + CHs cHs + Clz + CHsCl + Cl

Terminations

2Cl_+ C f z i

2CH3 + CeHe;

+ Cl + CH3CI

L2.L9

Rate

Equatons

an-d_Temperature
-l MOIC -t

L2.2I

267

KLI

12.22 L2.24

139.3

kJ mole-l

Uncatalyzed

Path

L 2 - 2 3 2 4 8 kJ mole
-t

7.g x 1os M-lsec-1


-3 -1 -l

Ener

CataLyzed Path (usually more conplicated)


I

-l

L2.25 3.O M - s e c 12-27 2.9 12.29 178


-t M
KLJ

L2.26 4.7x L2.28 66BK

10-M

sec

mole

- t-

L2.30

52.3 kJ mole

Ail _*_ eac


A catalyst
rtL>

provides

a path

with

a lower

Ea,

so that

a much larger fraction of collisions are effective and the rate is thereby increased. As shown AH is unaffected by the height of the barrier since we obtain the same products at the same energy leve1 by path. either Both E_r, and ".r, .ru decreased by te same amount of energy by the catalyst, rates are increased by the same factor. so that both

7B

79

CHAPTER 13 CHEMICAL EQUILIBRTUM

( s)
(i)

n independent

of

P, depends ratesf not

only Kt

on Tt

no change.

( h ) Catalysts Addition trations

affect

no change. concenno change.

the equilibrium of A affects but not K (see (g) above); (b) 1eft, (f) (c) Ieft,

Chemical 13.1

Equitibrium, (a) K= (b) K = (c) K = (d) r = [co]2 [o2] [oz]

LeChat-elier' ozl/lcozJz

q Principle

L3.7

(a) right, ( e ) right, (i) right

(d) no effect, (h) right,

no changer

(g) no change,

lcsz] [Hz]u/l"sJtlcrul tcqz/Lco2l towards (c) less moles of (d) left, 13.4 decrease dependent on temperature only and gas (e) left

Proplems Basgg o-n Equilibriun r3.b


)

llonstants l\--_,--l. e ) r3 .0942 a1-jm2

(e) r =
L ) .

61.0 (a) ICI2] = .050M' (b) .042 mole liter-l

(a) left, (b) left, endolhermic

13.10

IPc1s] = .060M

left'

exothermic

t-3.11_ (a) [so2] = .0040, (b) L3.T2


l-J. l_J

13.5

(a) (b)

K must K,

278 (mole/liter)
-f

[oz] at eq- = .0036 I


-t

and Kp are change

7.6 x 10 B-2 x 10

mole liter
-(

-l-

do not

\^/ith pressure.

'
-4

-t

mole liter mole titer

13.6

(a) (b) (c)

[cJ must [C] will expected

decrease increase to for an inert a greater mixture extent of gas. than

1 3 . l _ 4 5.47 x Lo I3. I5 1 3. 1 6

.0684 mole liter lHzl = [Iz] = .319 moles liter Inrl = 2.362 mores fiter-r (a) [Hz] = ltzl = I.35 x Io-3
-1

the reaction volume drives Adding A at constant However if so that to the right [C] increases. pressure A is added at constant [CJ would be but to a less extent decreased by the dilution, of gas. mixture than expected for an inert only rates, affects Catalyst . concentrations equi librium no effecl on

r3.17

mol.es

lrfer

(d) (e)

(b) .0127 moles liter-r


(c) 21.3% 13.18 13.19 .0253 mo].e liter .O3OOmole liter I l

the volume drives Removal of B at constant But so that to the left reaction [C] decreases. pressure the removal of B will at constant volume and [C] will C in a smaller concentrate but not as much as expected for an increaser mixture. inert K decreases

(f)

BO

B1

t ^

C}IAPTER 14

1 3 .2 0

(a) [Hzo] = [co1 = .0335 M, [Co2] = [Hz] = .0665 M (c) 3.e4 (b) 3.e4
(a) ONC1 = (b) Total (c) (d) .436 mo1e. NO = .564 mole, CI2 = .282 mole

.I'}]EORIES OF ACIDS AND BASES

/\cid-Basef4.1

_Concepts provides H+(aq) ions. is A base provides the reaction between

L 5 .

moles = 1.282 mole .340 atm, Pr,^ = .440 atm' NU

(a) An acid oH (aq)


+

P^--^, = ONCI .368 atm

n"r, = .220 atm

ions.
-

Neutralization produce H2O.

H'

and OH

to

L3.22

(a) 2.L9 x lo-10 mole liter-r (b) 6.7L x l0-e


atm

derived cation (b) An acid provides the characteristic the characteristic A base provides from the solvent. the In a neutralization anion of the solvent. and anion combine to form one cation characteristic .or two molecules of solvent. K = .0694 atm p (c) An acid is a proton donor and a base is a proton in aqueous medium is the acceptor. Neutralization acid of water from the conjugate of a proton transfer base of water to form two molecules to the conjugate of water. pair (d) A Lewis aci-d is an acceptor of an efectron with an unshared electron and a base is a molecule pair The neutralican share with an acid. whch it bond of a covalent is the formation zation reaction the base's unshared acid and base utilizing between pair. **n. = ''U' l,l -2 (a) HzO is neither Arrhenius concept. (b) an acid acid nor or a base j-n the base:

L3.23

(a) f4.4 (b) x=

atm-' I.L2x 1O-3 mole liter-l,

L3.24

(a)

.563

(mole /Lter)2 x 10-4 atm-2

(b) K.^ = 8.77 lr'

K = L.'776 (mole/1i-ter)-2 L3-25 13.26 13.27 13.28 3.35 x lo-3 2.44 \ n*", n"r. mole liter-t

10-2 atm atm, PfOt : 50.0 atm,

= 12.63 = PnCt,

= .75 atm atm

H2O can be either

Original Percent L3.29 (a) Kp (b)

Pec15 = 1.00 dissoc. = 75s"

H2o+H2oiH3o-+ottacidl base2 acid2 basel

0,50 atna i u mo f N 2 O 4

- 781 atm of NO2; I.22

a base' since it has two (c) H2o can be either In the adduct. or a Lewis acid-base unshared pairs, the acid part of the adduct reaction autoprotolysis
l f e -r r a l v
\ r!

n r n v# n v \ n u

aof

Shifted

tO

SOme

Other

base,

i.e.,

another water reaetion.

molecule

in

a base

displacement

B3

:O - H
'l

,.+ 1 { - o - I i
l

| ,l .7 + 0 H
l

(a) (a)

NHz , HgAso+,
t-

(b)

Czo+z- ,

(c ) OBr ,

(d) H2Poa (d) HS ,

Lewis oH-

base disPlaces adduct

iadduct I

1-l r new adduct ldisPlaced Ib a s e

1 , 1B .

(b) PH,*r,

( c ) u C z t t s o z,

(e) HPo4I ,l .9 (a) + NHq r

fron

(b) +

H z C z O +,

(c) +

H O C I' F basel +

(d) NHg

14.3

Many reactions

ean be interpreted

as redox,

e-g-,

l4.lo

NaH+NH2+NaNH2+H2 system was interpreted which in the ammonia solvent of the base H- to the base NHz-, is as the leveling betr^teen transfer as an electron also interpreted H(1-) and H(l+) to form H(O). it can be sulfurr of elementary In reactions See agent or a Lewis acid. a reducing considered rniddle of page 364 fox such an example, or below: H -'S:- +': + H -'S Lewis base f,ewis terms system: acid S(2-) and S(O) react to form S(1-). 3r-

(a) HF acidl (b) HNog base2 (c)HF acidl

HzFHF acid2 base2

+ H2No3' HF ? acid2 basel acidl + cN base2 + HcN acid2 i

+ F basel HPo22basel + HCo3 acid2 SO42 base2 + H2S acld2

(d) H2Po4 acidl (e) NzH+ basel (f) Hc2o4 acidl l.l .1,1 See text (a) (b) Hocl +

co32 base2 HSOa acid2 ?

In redox l.4.4 Solvent

NzHsr acidl Czo+zbasel examples

NHaCI + NaNH2 + NaCl + 2NH3 acid base salt solvent

HS base2

Bronsted:

+ NHq + NH2 -> NH3 + NH3 acidl base2 +


T

ans\^ersi other + oH -> ocl -> oH

are:

+ H2o + oH + H3o+

acid2
-> NHg

basel
+ NH3 base

Heo + 02 HCos

Lewi s:

+ NHr*
of

NHz base

(c)

* H2o ? co.'-

adduct acid I4.5

ne\,r+adduct+displaced

(d.) NH3 + CHgNHe? NHr- * CH3NH3+ I t.l2 (a) (b) H z N O H+ H 2 O ? H N O H - + O H


5 -

H+ and base NH3

(a) KOH + HNO3 + KNOg + H2O (b) (c) (d) CaO + 2H2o + Ca (OH) z Clz + H2O + HCI + HOCII l. I i

+ 2NHg - lNH2+ H 2 o + H z + o H + H2o ? HzsOg + oH substance may act as a Bronsted donate or accept a may either

( c ) H (d)

HSo3

2Na + 2H2o '> 2NaoH + H2

( e ) Z n ( O n ) z + 2 N a O H + N a 2 [ Z n ( O H )' + ]

(a) An amphiprotic acid or base, i.e.r proton. (b) Examples are

Br on s,t-ejl- Iowry L4.6 (a)

ept _Co.nc 2-, (b) HAsO+


(c) NO2 ,

HF HSo3 N2Hs+, HC2H3O2

H2AsO3

(d) s

B4

B5

(c) acid: base:


acad:

HF + H2O ? H3o* + FHF + HF ? s z r ' * + ' H>U3 r

r,l" Streng-th

ang Mgl-ecu]_ar

Str_u-cture electroincreases

(in liq.

HF)

..i()

H2o?H3o++sos2-

Since sj-ze is not changing, the greater the negativity the stronger the acid. Strength AsH3 ( H2Se ( HBr toward the right: Size increases effect so acid goj-ng down a group. This is ncreases strength lkewise.

base:
aCLO:

HSO3 + H z o ? H z S o g + o H +

-\ 2.r 1-5 . r , Hzo 7 Nz !tu + Hgo I


f

dominant

base: acid: base:

N2H5

nUI

? NrH.2* + c1

+ -> H C 2 H 3 O 2+ N H 3 Z N t t + ' + C 2 H 3 O 2 HC2H3O2* H2SOa I u2c2n3o2* * nsou

HeS<H2Se<H2Te
'..). (a)

H3POa '

(b) H3AsOa, (c) H2SOa,

(d) HzCOg,

(in anhydrous liq.


L4.L4 When an acid is stronger (cation) of the solvent cation. that HCI+HzO+ H3O +Cl
+ -

H2so

(e) (a)

than it is

ac the characteristic comPletelY converted


r ^n rvv vnf Ye v*__*

(c) sau3

9 -

'

(d)

NOz ,

(:)

so

:O-

the same fate' than H 3 o - s u f f e r Al1 acids stronger "levelled" solventby the so they are to bases whi The same language rs used with regard base (anion) of characteristic than the

@ 'I 'o 9

cl -o-H

1 . .

o ,,
:Cl -O-H

H l '?' ./"s

, O ,
: I-O-H

t . .

' t '"o 9

o blji
l
H

,-f-H

"o

t.. t "

are stronger the solvent .$.r


oz- + H2o+oH H All + H 2 O + H 2 bases stronger

"

+oH + O H than OH

100 Percent (b) 100 Percent are levelled to OH ' HC10+ is stronger than HCl03 since it has a formal charge on central atom. But HIO3 larger is stronger than H5IO5 for the same reason, the fact that the oxidation number is despite greater in the latter.

14.15

that can function t o a substance refers AmPhiProtic substance An amphoteric acid or base' as either into can be brought in water) not soluble (usuallY bY acid o r b Y b a s e either solution (a) Acid (b) sequence: H3o
f

1\

. i .O
H--se--H

I4.

IO

H3POa > HCN > HzO > NHg > CN (d) Yes > H2POa > HzO (b)

Base sequence: Yes' Acid (b) No, sequence:

NH2 (c)

> OH No,

" t " .,9,o

io

H-O-Te-O-H

\x'.'^/ .X ,V:.
..., \ ..
zY' 'Y\

T4.I7
1A 19)

(a)

HSO4 OH (c)

> H C 2 H 3 O 2> H 2 S > H C O 3


9 -

HeSeO+ is a strong acid, having a 2+ formal charge atom. H6TeO6 is weak, having zero on central formal atom. charge on central

(b) Base sequence:


t/4 l o

> COg-

> HS

> C2H3O2

> SO,r

(a)

No,

(b) Yes,

Yes,

(d) No

B6

a1

(g) The Lewis L4.26 Concept (h) t4.29 (a) BY going to (a) The carbon atom is an acid center. accept a pair of can it hYbridization a higher on S to make it SH- has 3 unshared pairs electrons' a Lewis base' Fe acts as the base (b) H+ must be the Lewis acid' are filled 3d, 4s, and 4p orbitats AIl its center. unshared and one can be are the 'pairs but 4 of all goes from dsp3 to d2tp3 used for bonding whe-n re hybridization. (c) Ag+ is an acid with NH is a base; unshared 5 s . a n d 5p orbitals Paar on ntrogen.
!

Electrophilic. Nucleophilic.

The acid

A1C13 displaces displaces I .

CH3

The base OH

@'... H-O:r*-r$=Q:+

@., r. t i'I-O-S=O: -> I I - O - S = O :

t{

I U ' iot I
o "

FI:O:^

L i

" l

:o:
H

\i'*z

empty'
H' is the

(d) This case is isoelectronic acid an Mn(CO)s- is the base'

with

(b);

(b) In the first O, donates a step the base center, In the S. pair to the electrophile of electrons a proton O-, attracts second step the nucleophilQ, reaction. Lewis acid-base in an internal (c) ahe new structure formed by the (step 2) is more stabl-e because all reaction migration formal charges are

point of attack H is the (e) sF is the acid; has been so thoroughlY densitY electron its F- is the F. by the hightY electronegative L4.27

because drained base'

by going to a higher (a) Be is the aci'd centeri Paars' it can accept m o r e e l e c t r o n hYbrdization base' F- is the By acid c e n u e r . (b) Hzo is a base and B is the exPands t o s p 3 . a Pair from o it "".upli"g must be an acid' octet' (c) s atom has an incomplete a base' tra= four unshared Pairs' t'(d) H- has one unshared can accePt acid center; (e) Au is acid centeri
u r> a strong b a s e . p4ir' sp2 to sP a Pair and go from

^ :^

CN

is

a base' ilisplaces H'


f

L4.2a

(a) (b) (c)

Nucleophilic' ElectroPhilic' Nucleophilrc'

The base H the acid

NHz .

ilisPlaces displaces

_ .

Noz .
H2O'

The base Cl

of the base GeS and the (d) Consider GeS2 an adduct is nucleophilic' Then tie aispfacement :s: . acid GeS' the base Ge disPtaces oH The base 02- displaces (e) nucleophilic' Br The acid FeBr3 displaces (f) Electrophilic'

qo

BB

,,,llr"ation CHAPTER 15 IONIC EQUILIBRIUM' PART I t".22 Veak Electrolytes 15.1 I5.3 15.5 15.6 :l.5.7 l-5 x l0 -+
" ? X

of

Water,

PH iog-l :5.o x lo-r3 l

t,-2L

(a) (b)

[H+] = 2.0 x 1o-2 M; toH-l = .040 M; (b) 13.18' (b) l.OB,

[H+] = 2.5 x 1O-r3 u (c) B.78' (c) 10.52, r (d) 7.60 (d) 12-62 M,

(a) O.46t (a) 4.14,

-2

t',-23 t"-24

l0
-5

-3 3.9 x 10 -7 7-3 x 10 (a) 0'082M


-4

2 . 3 x 10

4 (a) 4.7 x l-O M, -3 (c) 1.3 x 10 M (a) 3.5 x 10 -14 -

(b) 2.1 x 10-r

t ' ,- 2 5
(b) 0.112 mol HCIOz M
-?

!1, (b) 4.8 x 10


- l

- 5

11,
M -3
-F,

(c) 1.3 x 10

M,

(d) 4.6 x I0

- "?

4.{

x l0

t"-)-6 l-l M
(b) 1.8e" (b) .72% tHocll = '2O

x lo

15.27 3.r x l0 15.29 7.3 x l0


15.31

15.8
15.9

(a) 3'5 x 10
(a) I'g x 1o

t , , . , 1 8.rt
1 , ' .t 0 l". \2 | , . J l

-3

rt.36
pH : 6.57 to pH = 8.35

2.5 x l_0

-6

15.lo

+ ( a ) [ r i -] = l o c r ] (b) o - 04e"
+

= 8.0 x lO-s;

(a) 9.6 < pH < IO.0, (c) 4.5 < pH < 5.0

(b) 5.5 < PH (

6-0r

lCoHsNHg I
L).L

= 1.2 x l0

-5

.IB M 0. 3I% tH'l = l'5


-4

= o H - l ; [ c o H s N H z ]' 3 0 a 15.13 3.g x lo 15.15 I'0 x 1O-3 t"t

,rlnr()n-Ion

Effect,

Buffers

1 5. 1 4
l5 .l-o

x M

= o'10 M tN;l = o.l3 M; [HNa]

r', r.'l (a) pH t', (a) 3.80

4.35

(b) o.rB%
(b) .46e"

If,.I'

3.9 x l0

L).ro

='-i rH+r I ':^l^.:J'i.:ll::i;:i::""::'ffi"i:::":


-10 M benzoic acrd
l q

r{, 5-5 x 10 rit li) | O.70 M 4.85 5.4I

15.37 15.39 15 .4f

3.2 x l-0 0.71 M

-6

1 (

reacts = 2.7 x l0-5 M; Alt the NHacl lNH,*+l and leaves '10 M oH-' .ii-*'NH3 reacts = 5.4 x lo-s M; All the NHa+ [Nn,*+] '05 M oH-' .IS-*-NH3 and leaves

to

4 .5 x 10-2 t'l

to

produce

1 R

? n

lczHsozl/IHCtsoz] = 6.o/L.O
t '^"' Tt

rrln4

l/

/frtrr

[rln3

...6

tr

91

90

15.45 7.8 x I0

-3 -

tl/\trTER 16

mol
Ii}NIC EQUILIBRIUM, PART II

Polyproticts.46

Acids
i
I l', . ql rrtri I -:-::=.--: i l-r PrOd ]---.--. U C t
- l

= [ H C o 3] = 1 . 2 x 1 0 (a) tcorl 1 -63*' [H*] x 1O-rr' tco?-l = 4'a (b) 3-92 3

lr,.L .3

2.O x t0 2.O x fO

- !' ' "^

16.2 76.4

3.7 x r0

q
M M

_ o

" 2.l- x 10

= 8'5 x 10 L5.4'7 [H-] = tH2Aso;l = 5'6 x ro-8; Hesoi ) 15.48 (a) ts'-l

] M; [HgAsO+ = 0. 29 M = 2 x IO -' M CuCOg (molar than soluble

tasoi-l

solubility, 1.6 x 1O-" l) is J-ess _u M). l-.3 x 10 Ag2CO3 (molar solubility, x l-O-re) is less soluble 1.I x 10-i7 ).
t +

= 4'9 x 1o-2r M '7 (b) [Hs-] = '4 x ]o-8 't


lt,.l

CuS (molar solubilityr 8.9 than Ag2s (molar solubility, 2.O x 10 5.6 x I0
- l

- L' mol Ag2CO3

l-6.8

x 10-16 M L5.4g ts2-l = 4.4


lr,-,)

INi-

1 = 3.3 x l0

- 1-1

-7

15.50

[il+] = 1'9 x lo-3 l'l


t r, , iptation and K as Acids an sp Na- is
r a as.vl v
! s,

Ions

that

function

1 5 . 5 1 B .t B 15.53 8.61 15.55 0.60 M 15.57 4.3 x 10 15.59 5.A2 Titrations (b) 8'46' (b) 5'2Bl

L5.52

2.82

l',-lt)

The final
1 le- h e a
\

conc. of
q f a f
v v

0.16 M and of
.

CI M

is

O.3O M

<a
su

i nn

15.54 4.50 15.56 7.L x 1O-2M 15.58 2.9 x IO 15.60 .o7BM


rr, l.l rr,.I I

lBa''l tNO;l FinalAfter ir I

= O . O 7 OM ; = .15 M

tc2b[ 1 = 2.L x I0

t -

-a

(NOl does not react); ) tr I = 1.6 x I0-4 l

[trta-1 = . OrO (no reaction reaction [sr2+] =.030; 16.13 16. 15

= 9.5 x 1O-+ M = !.2. M

8.3 x 1O-B tNuil = . 21 M minimum

lgid-Base 15.61 L5.62

I r, | ,'r @) L2.L6 (c) I .78 , l(, I t lrt

+ tNn[1

(a) 3'92t @) 9.44,


-6

[NH?] = 3.6 x lO-3 M minimum [NH3] = .18 M Ca(OH)2 will not precipitate.

15.63 6.2 x r0 L5.64 22-00 ml

92

93

Precipitation 16.19 L6.20 L6.2L L6.22 L6-23 t6.24 L6.25

of

Sulfides forms' will form'

IIAPTER I7 I I,I.JMENTS OF CHEMICAL THERMODYNAMICS

No PreciPitate llo PreciPitate CdS will [tl*] [H+]

't'lrrr Firj;t PreciPitate' (minimun) minimun I l.L

Law,

In_terIral_Energyr_

Enth_alpy

= o.z3 = 0.47

x 1o-7 [Pb2+] = 2.3 [H+] = o.7O M, lS2-l x lo-rs lc,rt+l = 3.6 [H-] (a) must exceed tn+l = z.z M'

[Fe M'

x10 ' 'l =

-22

M'

0.30
| 1.2

The fj-rst law essentially states that energy is conserved during a physical or chemical changer but may be transformed. It is usually stated: AE = q - w, where AE is the change in internal energy content of the system, q is the heat absorbed and w is the work done by the system during the change. Enthal-py, H, is defined by t = E + PV. It differs from E by including the PV product of the system (which also has units of energy). function A state has a value dependent only on the pressure, state of the system (i.e., its temperature, vol-ume, composition, etc.) and is independent of how that state was achieved. A1l the state functions in Chapter L'7 are slmbolized by upper case letters. They arez P, T, Y, E, H, G, and S. (a) -1364.3 kJ,/mol

16.26 16.27

0'086

= .30 minimum

(b) [Pb2+]=3x10-7t

Complex Ions l.6.28 ( a ) 2 - 4 x l0-2


- ?

mol,/liter

(b)

( CzHsOH1) + 302 (g) -> 2 c o z G ) + 3 H 2 o ( l )


ahl-l_J()b. J<J

(b) 1'4 (c) 1'9 L6.29 AgCl will

x 1 0 - M
-5

x l

M preciPitate.

(c, (a)

-t t.9 KJlmor -1170 kJlmol + 302 (g) + coz (g) + so2(s) + Nz(s)

not

( b ) c s ( N H z )z ( s )

( + 2 H e O1 )
Since An = o, kJ,/mol kJlmol AH" = A E o = - 1 1 7 0 . 0 k J

/ \
I t .', (a) (b)

-92;2

-3911.8

C e H r z ( 1 ) + 9 0 2 ( 9 ) + 6 C O z( g ) + 6 H z O( f ) AHo = -3919.2 kJ/mol-

(c)

-157.2

kJ/moL kJ

AEo = -5459.55

94

r'
L7.7 L7.g 17.11 AE" = -1405'8 KJ AEo = -726'66 +81.69 kJlmol
u

L7.8 17.10

aE" = -66-57 kJ -LBA-79 kJlmol

| /.2I | /..2 I t...3 | / . ).4 1/.)5

AGo = -48.39 AGo -

kJ;

yes,

spontaneous

-467 kJ

yes,

spontaneous (c) -120.6 J/K

(a) -101,01, +63.6 kJ,/mole

(b) -L2O.6 J/K,

I7.L2

in an increase if it results A change is spontaneous If one observe of the universe' lntropy in the total it \n/ill change is that tir criterion only the "V"tt*, energy decreases' it its Gibbs free sponcaneouufy H and G scales Since there are no absolute for values formation to tabulate convenient Sinceallsubstancesincludingelementshaveabsolute (Such values could valuesformation reactions from the formation calculated entroPies') absolute (a) +103.0 J/K (a) -49.2 First J/K (b) -587'6 (b) -14s9'7 AGo = +267 '4 kJ AGo = -207'0 kJ kJ kJ it is compound

(a) AGo = +38.3 kJ NOT spontaneous. (b) Ac' = -11.8 kJ

Since AG" is Yes, it

positive,

it at

is 300"C.

IS spontaneous spontaneous at at at

11..6

(a) (b)

AGo = +130 kJ Aco = -26kJ AGo = -139.9 AGo = +2I.4 -33.4"C 69.1oC

No, not Yes, Yes,

25oC.

t ? _ l ?

spontaneous spontaneous

1000oC. 25oC. at l200oC.

1 I ..'.7

(a) (b)

entropies, there is nothing to be 9ii":1^?Y..tabulatin be easily


and the l/..:8

No, not

spontaneous

239.7 K or

! I ..'.') 342.2 K or 1 /. r0 I /. ||

L7.I4 17.15 L7.L6


I
I

+18.25 k/mol (a) -128.1 kJ, (d) -166.4 kJ 17.33 205.5 J/K (b) -332 .3 J/K, (c ) -29.1 kJ,

reaction

Second reaction

Secondreactionisspontaneous;firstisnot. HzS and HzO' SOz and H2 react lo produce L7.L7 17.18 L\.Lg AG" = BF3 is (a) -675-5 stable Yes, to it i-s possible' BCl3 will is hydrolyze'

I t . t.,. 2oa J/K I t. t4

is So (diamond) = 2.438 J/K mol. Since S" (graphite) is more ordered 5.694 J/K moL, the diamond crystal (Sovalue lower). -1095.O kJ

hydrolysis; reaction

i / positive' it is

r',

since AGo for the not sPontaneous' kJ

; I l,t,r;. tree yes preparation is possi I i I i r{, ilr

Energy

and_ Equilibrium 17.37 I7.3g L7.4I K


$/

(b) AG" = -41'6 Spontaneous

reactiont

5.12 x 1O-" 1.44 x IOG -55.2 kJ,/mol

0.443 2.O7 +38.5 kJ

preparations L ' 7 . 2 0 ( a ) None of these


because AGo is Since Aci is Positive' Positive (b) less

is
for

spontaneous at 25
NOz than for NO,

| 1 .lr) l/ |.'

is the combustion of NO to NO2 of the decomPosition generally is spontaneous to the elements 96

spontaneous.

But

Aco = +7g.g kJ;

= 1.0 * ,O-tu

ox the nitrogen in all cases.

o?

l l ; rl " f E R Ll.+5

lB

+27 kJ -2A.48 KJ, @) (d) 33.6


',lr,ltrctiOn

L7.44

(b) -42.67

J/K'

(c) -16.75 kJ,

} I,I I(

',IROCHEMISTRY

conduction In metallic j-n electrolytic charge; the charge.

the moving electrons carry moving ions carry conduction

of the atoms about As a metal is heated the vibrati-ons fl-ow and points interfere with el-ectron lattice their (conduction decreases). As a increases resistance ions can is warmed it becomes l-ess viscous, solution move more freely. and its conductance increases.
(a) (b)

Anions

are

attracted occurs at

to the

the

anode.

Oxidation

anode. charged.

The anode is
(d)

positively

Electrons solution) external

(usually in move from some species to the into the anode and thence po\/er supply.

r olytic

Ce11s,

Quantitative

Re]atignFhips

Anode

cu*cu2*+2e

9B

99

,{
(a)

IU.b

cathode Anode

2H2o + 2e 2H2O2+ O 2H2O + 2e 2CI crr2* 2CI

+ Hz *

20f1

r , ,L t a i c

Cells

Electrode

Potentials

+ 4H+ + 4e -> Hz *
zv

(b)

Cathode Anode

-> CL2 + 2e + 2e + Cu

Mg' Anode MgrMg-'+2e


t.L

Ag Cathode
-L

Ag'*e

+Ag

( c ) Cathode Anode

-> CL2 + 2e

5"(ce11) = Eo(cathode) - E"(anode) = .799 - (-2.363)

(d) cathodu

cu2+ + 2e

Anode (inert) 18.7 lB.B 18.9 .5709 lli gio+ + 2H++ 3e

+ cu + 2H2o -> 02 1 4H' + 4e

= + 3 . l - 6 2V

+ Bi + Hzo

'9759 Bi cdAnode cd*cd2++2e-

(a) pb2+ + 2lzo + Pboz(s) + 4n+ + 2e (b) .558 g Pbo2 (c\ 46-6 minutes

Pr, C1-2
Cathode

CLr+2e'+2CL- E ' ( a n ) = 1 . 3 5 9 5- ( - . 4 0 2 9 ) = L . 7 6 2 4v

18.10 18.12 18.13


i

10.06 I Ag 288 min(a) 7Bo c (a) .15659 Ni

L8.11

28.6 min'

E" (ce1l)

= Eo (cath)

(b)

.867 A

lrt..ls

(a) 2.27 v, (c)

(b) snz+ + Mg + Sn + Mg2+, ir @

18.14

(b) 4.29 min I8.16 18.18 28.6 sec 28.379 Cu lrr..26

Sn electrode .587 V,

18.15 333.8 I c 18,17 9.21 liter

(a) (c)

(b) Cu2+ + Ni -> Cu + Ni-2+, i" @

Cu electrode

18.19 1.19 M IB.2O La.2r L8.22 (a) 5.00 liter cu2+l = 0.358 Mr 5.77 hours
(b) tcl-l .893 M = 0.717 M

I u .'7 (a) pr I t, (") | r- (.ql ll cr- 1aq I cr, (s) n. . I


(b) E" =..8240 y (c) The cf2lc1electrode is the cathode.

r , 1r r r ( a ) p t I n r ( s ) l n * ( * s ) l l n r - l 1 a q l " . r ( r )
(b) 1.0652v (c) The anode is u2 lH+ Ii l . r , ) - 1 . 7 8 9 V 18.30 .987 V

l n.

r01

-37

( a ) NoT stable. (b) In +

1 8 . 3 1 - 3 2 0v oxidizi"g L8.32 (a) suitable "s""li ":".t:,:::


left
T

of

the

The but above T l ' . (?) and below Fe-'' arrows form in (reduced APPendix E are o"fY o"""'in 4 r' q+ 2.+ and (cd) ; cd gtt* 1ct'-) t : parenuhesis)

/ \

( c , Yes, and the product wil-l- be In3* + + + 3 + 2ln + ID 2In + 6H -> 2In 3In , 2ln + 3C12 + 2In3* + 6C1

3H2

Pbso+ (Pb)'

(cle) ; (b) cea+ (cd3+); Hocl of part (c) Invert the procedure ca(ca3+)i cr1cr3+) (crz); (d) eu+au3+)t ct 18.33

P b o z ( P b s o + ); (a);

Au, 1au)

.38

( a ) Tl-( b ) T1
+

rs

stable.

( c ) Yes,
r

Tl2T1 + 2H- -> 2Tr- + H2, 3 + + + 3}lzt 2TL + Cl2 + 2TL + 2CI ' 3+ + 3CL2 -> 2'1+ 6C1 (b) YES, (c) NO
?+ "-, +

zn(zn2+'

,-cr"'(Cr2o7 )'

-r.rt , - \, r 't r 3

tq,

3T1t + 2,rr + Tf + 2TL + 6H' + 2Tt2Al

(-) ag+(A9); No (Nzo' *)

(b) rr"' (rr )


(c) Hzs (s) ; ) (d) S(Hzsos 18.34 (a) The reaction will NOTgo as wrtten' * o' nz (s+) t Pb (Pbz+)

. |e ..0

( a ) No,

(a) -1.208 V (b) ti2+ ( c ) ti rs will stable. react with H+ to give Ti2+.

-> 2H+ * (b) H2o2 + 2As+ 1? 3+ + Ag + Fe (.) Rg+ + Fe'(d) No

.,1 |

(a) l.BOB V
(.l]/

Co2+ will

NoT disproportionate. react with H+ to produce co2+.

( c ) co wilt r.r (a) +. 040 (k) 2uOz uoz rr


2l1zo + Sllnoz * 4H r

+ + 3Hzo ( e ) H z S o g+ 2 H 2 S 3 5 18.35 -) Pb2+ + CLz + 2H2o (a) Pbo2 + 2CL + 4H+ (b) No (c) 2NOs + BH+ + 6r
(d) (e) (a) NO 2Mnor+ + 3Mn2+ +

2+ + 4+ + 2H2o + u + 4H -> uoz disproportionate.

+ 3rz + 2No +

HzO

will

( a ) L.397 V ( b ) YES Au' ( c ) Au will


+ will not not react disproportionate. with H+.

18.36

Hg + H92+

-> Hgzz+ lt (a) (b) (c) (d)

-3.396 V No, YES YES, Eut+

(b) No (c) No + (a) 2un3+ -t NIn 3N1n'' h+ ^- 2+ (e) sn'' + sn + 2sn

Eu2+ wiII

not

disproportionate.

wifl

be formed.

103 L02

,ff

Gibbs

Free

EnergY

and einf

Irr.6l

(a) 2.I57 V (b) M9 + Ni2+ * Mg2+ + Ni (c) li is lhe * electrode.

s r B . 4 s ( a ) p b l e u s o(a ) l s o 4 2 - l l e r 2 + l e u
(b) pb2+ * so42 (c) +.233 v (d) -45.0 kr + P b s o +( s )
1 8. 6 2

(a) 2.2407 v (b) (c) zn + C12 * zn is zn2* + 2CI

re.46 (a) Aglasr(s)r-llae+les l (b) es+ + r- ? Agr


(c) -647 v (d) -62.4 kJ
I

negative.

rr.63

(a) -.006 V (b) of the actual conBecause of the departure from the standard. states, the actual centrations is opposite in sign from Eo so the voltage proceeds from reaction in the opposite direction from the Eo. that predicted Spontaneous reaction is: 2 + T - ' P b + 2 H H 2 + P b The cathode is the this case is Pb. positive electrode, which in

t L 8 . 4 7 ( a ) P t l o , t s l l o n - l l u * l o zg I p t
(b) o.B2B V'
, *

-79'9
r r f

kJ

IB .48

( a ) P t l " r t s l l n - l l " ' l o rt s l l e t


(b) 1.229 v, '474.4 kJ ( c ) - I 2 1 . B JK (c) +72.68 JK (c) -72.4
-l

(c)

IB .49 18.50
It,. )I

(a) 0.2943 v ' (a) (a) .547 v, .3935 V, kJ,

(b) -56 .79L kJ, (b) -21I'1 (b) -75.9 (b) '43'5 kJ' kJ'

1 r. r , 4 1il,{,(i

2.o4 M (a) (b) NOT spontaneous. Now reaction IS

18.65

.0239 M

kJ lil.r,/ iu.r,rt

spontaneous.

18.52

(a) -509.5

kJ mole

+1.19 V (a) (b) When t"t2+1 is doubled the (algebraically) increases Cutting [lut2+] in .0089 v. half potential reduction by .0089 V. the voltage by

- l 1 8 . 5 3 -963.1 kJ mol

l.

f,q

(a) 125.2 kJ, (c) Hzoz, cot*, K=7.2x103


- I J

(b) 1.765 v ' s z o e 2 -, C 3 , F 2 K l-8-56 =2'7x10


-14 o r ^ r 1 . 6 x 1 0 l1

lowers

] u .r , r

(a) +.010 V (b) at the end depend on the The concentrations If the volumes relative volumes of solution. are 1.00 M to start, are equal and both solutions then lsn2+l = 1.370 M; [P2+] = 0.63 M. the volume of sn2* greatly while

18.57

K = 9 . 9 x 1 0 " "

18.58 K = 1.25 x LO"

18.59

-.284 V

The Nernst 18.60

Equatlon
f

on the

other

hand if

2+ " R e d u c e [ F e - ' l , r e d u c e P n .2 r t n c r e a s e

exceeds pb2+, then [srr2+] wirl 2+ go to .459 M. lPb I will

hardly

change

104

105

TER 19

If lsrr'*l 18.70 (a) .136 v

vice versa' wilr

[Pb+] "/iU

hardly

change and
NONMETALS, PART I: HYDROGEN AND THE TALOGENS

9o Lo 2'L77 t4'

( b ) p t l H rI t H * t = . o 2 5 l[lH * ] = s ' o o I n 2 l e t
(c) .L75v
18.71 (a) .0163 V (b) Ga (an) + Ga3+ (cat) ( c ) The anode, Gat*, L8.72 is i.e., the ? Ga3+ (an) + Ga (cat) Ga in contact with .3oo M

ly(lrogen l',-l (CHq), living water (H2O), hydrocarbons matter. H2 can be obtained from water by reduction with C, hydrocarbons, iron; or by el-ectrolysis of vater.
(a) (b)

2Na + 2H2O -> 2NaOH + H2 3Fe + 4H2O -> Fe3O4 + AHz Similar reactions can be written products are FeO and Fe2O3. in which the

the

negative

electrode'
+

The H consumes H2Soa' reaction The discharge SOa with the o in PbO2 to form water while combine As H2SOa is goes io forn PbSO+ on each electrode' is reduced' removed the densitY

(c) (d)

zn +

2H+ *

rn'*

* n, + Hz

Zn + 2OH

-> ZnOz2

( e ) C + H 2 o + C o + H 2 (f) cH4 | H2o + co * 3Hz

( g ) C a H 2+ 2 H 2 O + C a ( O H ) 2 + 2 H 2 l'r. ] (a) Hz r (b) (c) (d) 2Na + 2NaH

He + Ca + CaH2 Hz + c12 + 2Hcl 3H2 + Nz + 2NHs H2 + H2o + 2Cu

(e) Cuzo t (f)

co + 2H2 + cHsoH

( g ) W O + 3 H z + W + 3 H e O (a) Saltlike hydrides are ionic crystals, hard solids + w r LL L ' a- y r r ** , . y. . r a n d c o n d u c t e l e c t r i c i t y i L when molten. The hydrogen is a negative ion.
..i

(b)

Interstitial hydrides resemble the parent metal in structure and properti-es. The hydrogen is present as single neutral atoms in interstices in the metal lattice. Conplex hydrJ-des are salts. The negative metal atom (e.9. B) surrounded a central hydrogen atoms (e.9. BH4-). ion by has

(c)

106

tr(d) all the nonare formed with hydrides Covalent or gases liquids simple and are usually metals (e.g- HCl); but with B, C, and Si complicated like (e.g' hydrocarbons can result structures CsHrs). 19.5 H2 is the London force' the least Since Hz exhibits aside from Het and is not ststance most volatile in liguids' very soluble because of their are very effective These solids in the form of atoms' hydrogen to dissolve ability with other to react thus making them very available adsorbed on the surface. substances (a) 19.8 .5039 H2 lift, le.L4 (a) CaF2(s) + resO4(1) + r*(n) + CaSoa(s) (d) (e) 6Cr* Cr2O72- + I4H+ -> 2Cr3* + 7H2O + 3C1z OK for Br2 and 12 but not for F2 since Eo (red) of F2 is more positive than that of any of the oxidizing agents above.

2t{F(r) 4F(b) (c) (d) 2Nac1 (1) 2Bx "Itttt,

H2 * F2
2Na + c12 2Cl+ Br2 * NaHSOa + 3Na2SO+ * H 2 S O +* for H2O reaction)

19.6

+ C12 +

5NaHSO3 + 2NaIO3 + Iz

(b) . e58s
(b) 28.89 g of 13.33 tines its Air own $/eight (e) (a) (b) (c)
-l

(See answer

key

in

text

an ionic

(a) 2.OL6 g of (c) 26.87 g of 8-92 g of of

PBrs + 3H2O + 3HBr + H3PO3 H2 t Cl'2 -) 2HCI ZnCL2

li
j L l

19.9

(a)

( b ) 3 2. 4 3 g o f z n

Zn + C12'+

(c) 58.87 I

2P+3CL2+2PC1s 23+Cl-2+SzC1z 2C!2 -> 2HCl + SC12

1 9 .r 0

(a) (c) (d)

-270 kJ mole -44


f

bl -

'244

-l

kJ mole

(d)

- l

kJ mole

( e ) HzS * vs

-270(a); HzOr -241'8 A H - o t s : Hf", -269 vs -244 (b) N H 3 - 4 6 . I 9 v s - 4 4 ( c )

co+c12+coclz

(s)
(h) (i) r'r- f6 (a) (b) (c)

SO2 + CL2 -> SO2C12 3I + CL2 -> 2Cl + 13

ll
i

1 0 l l

A"" = -Bo5.B kJ mole

-t

H2O + CL2 -> HOC1 + HC1 4Hr + sio2 + siFr+ (g) + 2H2C-

energy indicati Result is very close to NaCl latlice in arrangement and size are similar lattices that the The larger value for NaH is consistent wth the smaller size of H- compared to C1-'
L9.L2 (a) 26.09 g of H2 (b) 289'8 liters of H2

NaCOa + 2HF + 2NaF * COe + H2O HF + KF + KHF2; contains CaO + 2ItE -> CaF2 * H2O key for some alternatives and some ionic FHF ion

(d)

The Halogens

(See answer reactions)

1 9 .t 3

(a) 2CL (b) (c)

* MnO2 + 4 H

2CT IOCI

2 + +Mn + 2H2O + CL2 + 2 + + 2f12O + CL2 + PbOz + 4 H ' + P b -+l6Hi+2Mn t**Bnro+5c12

nr

<-

t1

+F 'ur-

(a) (H bonded) (b)

+HF i

+ 2MnOa

l0B

109

tw*
Eq(a), but in very concentrated HF is a weak acid, reaction (b) removes F- and drives reaction solutons, At moderate and 1ow concentrations' (a) to the right. a and it behaves like (b) is unimportant reaction nonnal weak acid' lg.l8onemustuseconcentratedH2Soatodriveoutthe vo}atilehaloacid,andconcentratedH2Soaisastrong The to Br2' Br-' agent to oxidize enough oxidizing gas' in the product of FIBr and Br2 is a mixture result t9.24 19.25 l').26 The above CI2 + 2Br two Ag salts '> 2CL -> 2CI dilute * Is doe not redissolve brown in NHs. + Bxz 1i9ht Is *

CI2 + 3I Also very

deep brown starch + blue complex

28.2 F 886 g of L323 g of C12 CI2

19.19
l

(a) FeCl2' ( b ) RbCl,

FeIII GrI

more covalent

than GrII

FeIT

more metallic

than

( c ) BeF2, F more electronegative compounds are more ionic. Lg.2O of XX 3 is an AB3E2 moleculet two trigonal Pairs' PYramidal a T-shaped molecule' giving

than

Cl.

so its

bithe 5 trigonal pairs are unshared, one unshared; pairs'

XX s j-s ABsE; 6 octagfonal pairs' square PYramid.al molecule '


I

XXt

, pentagonal

bipyramid, 2Na + C12 " le --) e* * -c t r . electr,

no unshared

Lg.2L

(a) 2NaCr
(b) (c) (d) (e)

electr)

2NaCl + 2H2o 2NaCl + 2H2o

^, CL2 + H2 + 2NaOH H2 + Nacl ^-- | Jflz -fl2 f

+ Naocl + NaClOg (s)

electr. 6NaCl + 3H2o < raclOg + H2O HCf ' (b)


+

rqil-

electr. (c) (f)

, il \,d^ t^ r A u v4 effect),

L9.22
I

(a)

CIz '

HF (u bonding clz

(d) Fz,
' L

(e) Cle I

1q.23

ca-' Ag* AgCl Ag-

+ 2F + cl

+ CaFz (s) -> AgCl (s) white *

+ 2NH3 -> Ag (NH3) ,* + Br -> AgBr (s) -> A g I ( s )

c1

cream color

Ag+ + -

yellow

110

111

W
( e ) 2PbS + 3O2 + 2PbO + 2SOz CHAPTER 20 11: THE NONMETALST PART THE GROUP VT A ELEMENTS reactions which as Pb3O4. produce also oxides of pb such higher

20.5 Oxygen 2O.L silicate in H2O' in carbonate and Free oxygen in air' and Get oz by liquifaction rocksf in living matter' (minor) by of airt also fractional-i"iirr"tion water' of electrolYsis
(a) (b) 4H9o -> ztlg2c- + 02 2HgO '> ztg + 02 also 2Na2O2 + 2Na2O + 02 heating 2+ 2HzO + 4OH + Oz 2O2
or aqueousr

(a) C H 4 + O z + C + 2 H 2 O (b) 2CHa+3O2+2CO+AH.O

ll

( c ) CH+ + 2O2 -> CQz + 2Hza 20.6 .. IA oxides are hard high*me1ting VIA oxides are gases, liquids, are covalent. IA oxides react bases, VIA oxides give acids. (a) Oe (b) oz ionic compounds; or low melting, and hrith water to give

2O.2

,'.O.7

@l
olr

Oz2-

olr

o*

(c) (d)
lii

2NaNOg 2KC1O3

-+ 2NaNOz + Oz -> 2KCL + 3oz 2H2o + 02

+
Ttr

t
Tftr

+ + +
Tftr TiJr

++ t+
,JIt lTk

(e)

ztzo *9SE*

t+
1 T f

t+
i

++
1T
a-t-

++
?T

++ ++
1I 1T

i,
IL

2O.3

(a) r*02+9' (b) (c) 2ta * 02 + Na2O2 4Li + 02 + 2L2O

f+
BC*22 unpr.e
also 2Hg + oz + 2llgo
O2 +

^
BO=f;l unpr.e

rJ
BO=1 0 unpr.e O2

l l i

(d) (e)

2Mg + oz + 2ugo '+ 2lgzo' 4Hg + oz

(f)Ba*02+BaO2 (g) C + Qz + COz' also -> 2CO 2C + Oz


d*

O*

(h) S+Oz+SOz 25 + 3Oz + 2SOg (i) (j) 2O.4 (a) 2cueo t oz + 4cuo

t
(minor Product)
Trt
T*

+
Tt?t

+
'lTr<

t+
1I

+ 5Oz + P4Ol 0 Pq + 3Oz + P4Oe P+

,IT

t+

,II

r+ ++
U

+J
1T

+ BCOz + 10H2O 2CqHro + 13Oz + 6HeO + SOz (b) CsHr 23 + 9O2 + 5COz -> 6COz + BHzO (c) 2CgHsO + 9Oz -> 2ZnO + 2SOz (d) 2ZnS + 3Oe

++

eo:2L I unpr.e

BO=I; 1 unpr.e

TL2

113

20.9

(a) (b) (c)

Pb *

H2O2 + SO4

2-

2 C r ( O H )3 + 4 o H 2MnOa

+ PbSO+ + 2 H z O 2+ BHzO + 3 H 2 O 2 + 2CrO4 + 2H


. L

r0.16

(a) C2H22O1r(s) + H2SOa(1) + fZC(s)

+ 11HzO(1)

+ H 2 S O a( a q ) (b) NaNO3 s) ( ( + H 2 S O + 1 ) + N a H S O qs ) ( ( + HNO3 9)

-> 2Mn + 5HzOz + 6H-

5Qz + B H 2 O

(d) AgzO t

H2o2 '+ 2Ag + Oz + H 2 O

( c ) C u ( s ) + 2 H 2 S o a( 1 ) + c u s o u (d) zn(s)

20.10

* S O z( S ) + 2 H z o ( I ) (.q) - + Z n S O a( a q ) + H 2 ( g ) + H z S O +( a q )

6o
./ \,'"
Bo=L4 , Type ABrE molecule' bent shaPe (a) -285.9 kJ mole-l I of of with

@..

( e ) z n S ( s ) + H 2 S O a( a q ) + Z n S O 4( a q ) + H z S ( g ) (f) .10.17 Fe203 (s) ( + 3 H 2 S O + a q ) - + F e z ( S O + )s ( a q ) + 3 H 2 O Fes + 2H+ -> Fe2+ + H2s SOz + H2O + HzSOs

e,,o/o\,
one unshared pair'

(a) s + Fe + Fesr (b) S + Oz + SOz,

(c) Na2SO3 aq) + S (s) + Na2S2O3 aq) ( ( 2O.II H2o H2o ozone are more exothermc (d) S + Oz + SOz, N-OH(.q) 2SO2 t 02 + fgQ.,

(b) -333.3 kJ mole Generally than with reactions oxygen'

+ S O 3 ( 9 ) + a H S O s( a q ) 2 S o 2* 0 2 + f g Q r ,

(e) S+oz+Soz

i
20.I2

SO3 + H2SOa -> HzSzOt None of instance .LB the above answers i-s unique. ans\^er key in text. See for

2gg.5 kJ mole-l

Sulfur 20.13

Selenium,

4q-fellel!.1$

(a) 2 S O 2 + O z + 2 S O s (b)

but its of Sg melt to a mobile fluid The crystals in increase increases with further viscosity open and form Sx chains' as Ss rings temperature off' boil fragments various At very high temperatures So, Sq S2 ParticularlY' Hot water is PumPed underground which is forced uP w i t h t h e a i d and melts the sul-fur of comPressed air'

SO2 + CL2 -> SO2C12 ( c ) s o z ( g ) + H 2 o( 1 ) - > H 2 S O 3( a q )


2 +Cl +3H2O

3 s o z( g ) + C 1 0 3 + 6 O H + 3 S O q ( e ) S o z( s ) + o H , ^ : , - > H S o g ( a q )
(d)
tq9,

2O.L4

(f ) so2 (s) + so3


\ l 3 /

2-

(aq) + H2O + 2HSOg


AB2E2 tlpe, AB3E type, (3 resonance ABa type, bent trigonal forms) tetrahedron pyramid

,^.,O.- a -'d.O i
(b)n:O:

20.15 (a) S+02+SOz also minor + S


2-

: : : : : :
:O: V

amounts:

2 5 + 3 O 2 + 2 S O g 2 - .

s/"

S = O:

s (s)
S (S)

( a q ) + s 2 (aq,) 22+ SzOs * SO3

I nr

(c)

F e + S - ) F e S S + 2F2 -+ SF+, S *
/f\

. . 1
3F2 -+ 5Pa

, 4 ,O

23 + CL2 '> SzCIz S + 4 H N O 3+ S O z ( g ) + 4NO2 + 2H2O

,o ,
r14

( 6resonance
115

forms)

(s)

(d)

,B,O

AB4 tlE)e,

tetrahedron

)o.22

( a ) 2 H C 1( a q ) ( b ) 2 H C 1( a q )

Na2So3 (aq) + 2NaCl (aq)

+ Soe (g)

+ H2o

,o=l=0,
(6 resonance forms) (e)

+ Na2S (aq) + 2NaCI (aq)

+ HzS(g) + Soz (g)

Na2S2O3 (_q)

+ 2 H C 1( a q ) - ' 2 N a C l ( a q )

+ H2O + S(s)

Ihe answer key gives

the above in ionic

form.

o
(f)

ll ll t - s - S - il - s - t n " \7 " l[ ll
forms i.

two tetrahedra (each like part d) joined corner to corner '

.to.23

(a) 45 + 6oH -> 2s2- * s2o32- + 3HzO (b) s2o32- + 2H+-> s + soz(s) + H2o
SO2 + S+Oo- , SOg
2-

manv resonance i\.

.'.0.24

+ SzOs-

?-

- - s - - s - - n . - i l " 1 1
Il u

il l

l ll

T\^ro tetrahedra part c) like a corner.

(each sharing

.'.0.25 r - 3 - r /\
F F

Note the expand.ed octet in SF4, which. is permissible for S but noc for O (only 2s, 2p orbitals, maximum of 4).

O:

O:

'.0.26

: i
ll

l
f

l
l

- 6 - - o - - s - - H
. .

' l tl

l
O:

O:

"

Two tetrahedra part c) joined to corner.

(like corner

Per-acids are polymers formed. by removing H2O from peroxy-acids the simple acid. have O-O bonds. (a) Add eb2+ la1so many other colored, very insoluble metal H- to (b) form foul smalling form to sharp metal-s) sulfide. to form a dark Or add

't.2'1

H2S gas. snelling BaSo+. SO2 gas. Other metals,

I
2O.2O

Add H* to Add 8.2*

(a) cHsc(NHz)s * H2o + cHsc(NHz)o + HzS (b) (c) SO2 + H2O + HeSOs SOr + H2O + HzSO+ + 6 H 2 O + 2 A 1 ( O H )3 + 3 H z S ( g ) 'tr-28

(c) e.9.,

precipitate used.

pb2+ also

(d) Atesa (e) (f)

(d) Add acid to form and S(s) which forms (a) (b)

H2S 03 (and eventually SO2 gas) a rnlky colloidal precipitate.

SeO + H2O + HzSeO

H2O2 + HOSO2CI + HCI + HOSO2OOH HeOe + 2HOSO2CI + 2HCI + HOSO2O the above reactions are: OSO2OH

TeOg + H2O + HzTeO+ (S) HzO + HzSOs -> HzOz + H2SOa (h) -> 2HzSO+ HzO + H2S2O7 -> 2H2O + 2SO2 3O2 + 2H2S -> 2HzO + 2TeOz 3O2 + 2HzTe 2Pbs+3o2+2Pbo+2so2 (a1so get
(d)

Structurally

20.2L

(a) (b) (c)

(a) no-G\+i)-E
\Y--l

,,

-o" + Hcl + no-o-8-ou


,,

o
i

(b) Ho - -G\*fu-4r\+lcr)- s - oH+ 2Hc1


of Pb)

higher

oxides

l \/ o

\1,--

2Na2So3*02+2Na2So4

+ HO - S - O -'O - S - OH

o i

tl o o l

l o
I16

t
LL7

l o

r-

ifl
(b) bent, pyramid, (g) (c) (e) trigonal planar' .IIAPTER 21 . I ' I I I iN O N M E T A L S ,P A R T I I I : or octahedron 'l.l as Z The elements become increasing more metalfic also stability of negative oxidation states increases, Multiple decreases. bonding bet\n/een atoms occurs only is the only one which for nitrogen, and nitrogen carrnot expand its octet. The hydrides become less basic rnith increasing Z. increasingly N2*3H2+2NHs N2 + 6Li + 2LisN (alsorother metals. ) temperature Group IA and Group IIA THEGROUPVAELEMENTS

20.29

(a) bent, (d) (f) trigonal

tetrahedron' tetrahedron (i)

tetrahedron, shape,

distorted (h)

,,saw-horse,,

octahedron,

20.30

O":iYt:-i' (a) HTe is a weaker base than.Hs (Same reasonang as Hur 4u larger' much
(b)

is
HI. ,,

so

(HO)2SO2 versus. ComPare t h e m i n t h e f o r m s : lone oxYgens on f,ne ( H O ) e , T e ' H2SOa has more atomcentral

N2 r
I

02 -> 2No high

(arc)

N2tCaC2+CaNCN+C
I

N2 is

so unreactive

L i

high energy, there is.

because -, 94L kJ mole


- I

the

N=N bond has such the strongest

about

bond

crops, e.9., Certain bacteria which return crops exhaust it. Really
I I

alfalfa, nitroqen

contain to the

fixing nitrogen soil. Other

pure

N2 at

STP' 1.250 g liter

g liter-l

"Air

nitrogrr",

L.257

is hydrogen bonding between the In both cases there on N' but there amine hydrogens and the unshared pair in ethylene-diamine are twice as many opportunities can and more extended and complex H bonded clusters result. of structure Pa is most reactive because the units is more permeable to are monomeric, the structure and most it has the highest vapor pressure, reagent, strained important the 6OaPPP bond angle is greatly th9 Black phosphorus, and lowers the bond strength. pollmerj-c is least reactive. most complex and highly

llB

119

ff*

- = "-4'''
forms are noE' the polymeric Only Pa is soluble' is an electrical crystal' molecular P,+, a strictly and black is insulator; tt i" a poorer insulatori the covalent within plesumaly the besc t""''"I"t graPhite' bonded Planes as n 2L.7 + (a) P+Oo + 6HzO HsPOs+ 2HzO BN has each B at the center of a The diamond-like To make a modelt of N, and vice-versa. tetrahedron cubic ce11 of B atoms, subdivide make a face centered sub every other B sub cubes and put a N into it into cube. (Haber process). An AB3E Prepared from the elements pyramid. Very a trigonal shaped like type molecule, Pure NH3 polar. Acts as a base in water solution. (compared to point high boiling has an unusually hydrogen because of extensive Group VA hydrides) other bonding. (a) + 2H2o (b) (c) (d) (e) 2NHs (aq) NH3 (ag) + Ag + + Ag (NH3) 2 +

P a O 5 ,+ B O H + 4 H P O 3 PaO6 + H+ (b) no reaction

+ no reaction Sb+Oe + 6HzO +4SbOz +2HzO SbaO5+4OH -> 4sbocl(s) sbao6 + 4H+ + AcL

+ H* +t NH,**

( c ) Bi2O3 + H2O + no reaction + no reaction Bi2O3 + NaOH -> 2BiOCl (s) + uzo Bi2O3 + 2H+ + zCL + 2Bi3+ + 3H2o rr"o sizoa + 6H+ acid) (very concentrated

NH3 (aq) + H2o + co2 -+ NH4' + Hco3 4NH3(s) + 3Oz hetsl Pt'h"lt 2Nz (g) + 6H2o 4No(g) + 6H2o

4 N H 3( g ) + 5 0 2 ( s )

( f ) N H 3( 9 ) + H C I ( s ) + N H + C I ( s ) t"tl (s) 2NHs(g) + 2v(s) zvll + 3H2 (h) 2 N H s( 1 ) + 2 N a ( s ) + 2 N a + + 2 N H 2 Note: Na+ and NHz dissofve in + Hz (9) NH3(1)

N z * 3H2 + 2nt
pf

4 N H 3 + 5Oz :; BN is:

4NO + 6H2O

2NO + 02 -> 2NO2 2]r.g GraPhite-like 3NO2 + H2O + 2HNOa + NO 2NO * 0 2 a s i n third reaction, * etc. 4H2O

I I \--y'u-n,/"\N/
N

'

( a ) 3Cu + BH+ + 2NO3 etc. r \^Iith benzene-like resonance (alternatively, (b) (c) Azn r lon+
(>,,

3Cu2* + 2NO + conc. HNO3)

get + No3

NO2 in

l 1

aB\r/u\* i l
R

/tr-B-

'> 4zn2+ + NH4+ + 3H2O + 2 N 2 O s( s )

P , * O ro , - \

( + 4 H N O s( 1 ) + 4 H P O 3 s )
NH+NOg Ca(Nos)2 +

\n/"u*/
l

(d) (e)

NH3 + HNO3 + ca(oH)2 +

2HNO3 +

2H2O

ionic:

ca(oH)z(s) + znl + c^2+ + 2H2o

L20

I2I

I'ti

2I.I4

(a)

NeOr NO N2O3, NO2,

N2O4 N2O5

'1.17
H - O tl P -

(b)

NH+Nog N2 + 02

httl tt!

n - H
II tl H P

,"ro

+ N2o burn NHg on Pt)

II

tl il t "
f,

- H

Zono (or

H -

P -
I

- o H

l
I

"

NO+NOz+NzOg 2tio+oz+2NOz 2No2 -+ tgu P + O r o + 4 H N O 3+ 4 H P O s + 2 N 2 O 5 (c) . ' = N = ' O'^' , = , * s : N = t l - "er ' ^ lij = i: o @ (minor form) | . lB The acj-d hydrogens (a) (b) (c) are only those

I
I

I
I

attached

to

oxygens.

PaOl s + 2H2O + 4HPO3 3 P a O 1 + I O H 2 O - + 4 H 5 P 3 0 e e PaOls + 4H2O + 2HaP2O7 4Hrpo,*

(d) PaOle + 6H2o I

*: * -,, - = [ :: = f' b = , = B - i "


";'\ @ "
o '.o/
cr'Y..
i
a !'

, = - R = + :, = - R - ' o U o:dr '


o

The above formulas (a) infinite represent polymer, or cyclic pollmer, (b) tri_mer, (c) dimer, and (d) monomer. AIl possi_bilities are represented by H P O , for which the reaction would be: n+2 n 3n+1' f 4l. L9 n+2 P+or o * -tH2o + Hn+zPnon+r

@ z.'b.: many resonance forms I N -N\

\ d'.
@./2" N'

(a) LigN + 3H2O -+ 3L1OH + NH3 ( b ) A l N + 3 H 2 O + A l ( O H )3 + N H 3

'h'.

' @N

pr'

.. o "

-.g:tt

many resonance

forms

(c) Ca3P2 + 6H2O+ 3Ca(OH)z + 2PHa (d) (e) CaNCN + 3H2O -+ CaCO3 + 2NH3 NC13 + 3H2O + NHs + 3HOCIPC13 + 3H2O + H3PO + 3HCL

2I.I5

(a) N H a N O 3 + N z O + 2 H 2 O (b) N H 4 N O 2 + N z + 2 H z O

(f)

(g) PC15 + 4H2O + HsPOq + 5HCt (h) HqPzOz + H2O + 2H3PO'+ (i) 3NaN3 -+ NagN + 4N2 + H2O 2H2o 1 'o 3NO2 + H2O + 2HNO3 + NO

/^\
\ e /

Pb(NO3)z + PbO + 2NO2 + ZO2 N a N O 3 + N a N O 2+ Z C z 2Nal{3 -} 2Na + 3N2, also 2NH3 + OCl + NzH+ + Cl

(d) (e) zL. ro

(a) H2ll2O2 = HzO + N2O (b) (c) 2HNO2 2HNOg = HzO + N2O3 = HzO + N2O5

(a) (b) (c)

5Hz + 2H+ + 2Noz + 2NHgOH+ + N H I + F+ 2 H F + N F s + 3 H z

( d ) 2 H a P 2 O 7= A H z O + P 4 O 1 e

r22

1,23

{${-

( e ) 4 H e P g O e= 6 H z O + 3 P a O q (f) 4HgPOg = 6HzO i P4O5 As2o5

| .24

( a ) Na3As + 3HzO + AsH3 + 3NaOH (b) A s 2 O 5 + 3 H e O + 2 H e A s O + ( c ) l4g3Bi2 + 6H2O + 2BiHs + 3Mg(OH)2 (d) A s 4 O 5 + 6 H 2 O + HsAsOa

(g) 2HgAso+ = 3Hzo t

2 r . 2 L( a ) ' = - , O
( b ) , = - ,

. * -O : - = ' *

( e ) 2SbC13 + 3H2O + SbzO + 6HCl (f)


D ul . + T l 4 "+ ! I J

D-

'

t v
,=-,

" o

**

a\" \./
,

Y : O - N = O :

F!. t

/ \7

.-

(c)

, = t

<->

l d ) - N = :

o
I

, =

N = N

t \ - ! \

: = N -

< - )

@
.^ \l

trans | .26 ( a ) NHs ( b ) NHz + ( c ) PClr+ (d) PC15 AB3E AB2E2 AB* ABs ABs trigonal bent tetrahedron octahedron trigonal

cis pyramid

\7

:O: A

1 .. \7 /
.. \y

l ^ , , t7 Q
/
l

a v

tl i l o :

( e ) H - o - { = o , " l
:O:6

+ - +H - o - N - o :

.. \? -e)
" t l

" \-,,
+ - +" = f

o:

( e ) SbCl5 (f) Sb(OH)5

bipyramid hydrogens)

( f ) H - = N = :

- N = N :

(ignoring the ABe octahedron

@ o
2r.22 ( a ) l t + + H 2 N N H 2+ H z N N H g * , (b) (") (d) u+ + H2NoH + H:NoHu+ + No2

o @
2H+ + HzNNHz + HgNNH

1.27

( a ) Reduction of SiOe to form

phosphate rods a CaSiO3 slag.

with

carbon

using

+ HNoz 2HNo2 + Hzo + No + No2

( b ) Direct combination of the at moderate temperatures,

elements under pressure using a solid catalyst. a),

H+ + NH3 + NH+* + Hzo + co2

(e) 2H+ + co32 2I .23

( c ) Treat phosphate rock with sulfuric acid. Alternatively (part make P4 from phosphate rock burn P4 to,P4O19 dnd add water to form H3POa. (d) Make NH3 by Haber process, burn it catalytically to NO, combine with 02 and H2O (problem 27.12) IINO3. ( e ) React (f) H2SOa v'j-th phosphate carbon. rock, C a 3 ( P O 4 )2 .

(a) 4As + 3O2 + 2As2O3 ( b ) P , + + 3 O z + P + O o, P ' * t (c) (d) 2PCla t Oz + 2POC1g 5O2 -> P+Oro

to

Oz + 2NO + 2NOz

Reduce As203 with

(e) 2Sb2S3 + 9O2 + 2SbzO + 6502

124

L25

r
2L.28 (a) (b) + NH+ + OH
H 4 T

+ NHs (g) + H2O

(]HAPTER 22 ,IHE NONMETALS I'ART IV: bond to r'rbon and Silicon CARBON, SILICON, BORON, AND THE NOBLE GASES

+ 3H2O + PH3(g) + 3HzPOz 3+ 6HzO + 4AsO ( c ) As4O5 + l2OH t 5H2O + 2NaSb(oH)s (d) Sb2O5 + 2Na+ + 2OH 3OH (a) each P forms a single Pa tetrahedron, a n unshared Pair' other 3 P's and has

2L.29

.1

rt forms a nelative ion (Ca- and Cz2-); has a strong tendency toward catenation; and forms a gaseous oxide.
Catenation can occur easily because the C-C bond is very strong; few bonds to C are stronger. (a) In diamond each C is surrounded tetrahedrally by (c) In SiC, Si & C al-ternate C atoms. places in the (b) In graphite diamond structure. all C's are sp2 and form a continuous plane; there are only weak (London) forces betv/een planes.

.2 pair.
(d)
L

a fourth but attach As in comPleting position, in outward O's around each P.


P4O5I

oxygen to each P of a tetrahedron

1l

.4

Truly

ionic
l-

carbides
,-

contain

metallic

cations

and

either C' or C2anions. Covalent carbides, like SiC, are covalent netlrork crystals, very hard and high melting. Interstitial carbides contain carbon atoms wi-thin a host metal lattice greatly without changing the lattice or the metallic character; usually harden the metal. (a) HCN(g) hydrogen cyanide (b) HCN(aq) hydrocyanic acid (c) KCN potassium cyanide (d) KOCN potassium cyanate (e) KSCN potassium thi-ocyanate (f) Fe(CO)s iron pentacarbonyl, or more formally, pentacarbonyliron (O) (g) NaHCOs sodium hydrogen carbonate (h) NazCOs sodium carbonate

(a) Co(g) + c12(S) + cocr2(9) (b) Co(s) + S(g) + cos(g) (c) 2Co(g) + oz(g) -> 2Coz(g) ( d ) C o ( g ) + F e o ( s ) + F e ( 1 ) . + c o z( S ) (e) 4co(9) + Ni (s) + ni(cou 1s
L26

L27

(a) ".".

(") (")

+ 2HzO(1) + Ca(OH)2 (s) + Cz[z(g) * 12HrO(1) -+ 3cH+ (g) r 4 A 1 ( O H )s ( s )

':.L2

HgBOg* H2O ? g.O* * Represent the above H-(.e)

H2BO3 structurally: + B(oH)r*

(b) Al+Cg

B(oH)g + H2o i

22.4

(a)

:N=N:

(b)O,c=o,@

. l ; 11 3 .

(a)

4Mg + B2O3 + 3MgO + MgBz, or 3Mg + B2O3 + 3Mgo + 28 (limited lS)

(c)
(d)

Q"=*,
: = c = ,O ** O, - c = N,
:'d:c='S:
(b) (c) (d)

2BBra + 3Hz '> 29 + 6HBr (g) 2e + N2 + 23 Mg + 28 + MgB2 +BF4 and 2 8 ( O H )

(e) (f)

(e)BF3*F (f)

B2O3 + H2O +.2HBOz' B2O3 *

(s)
:uI
jg

rii,
I
u
^

( h )z r :
\ 7 "
a l.:\"

3H2O + 2HsBOg or '

"

l
I

\.1-:

:'1

.1-1 .

3 resonance forms

(9) e(oH)3 + oH + B(oH)a htt! (h) B(oH), HBo2 + H2o (i) 2B(oH)s 5 B2o3 + 3H2o

22.9

(a) caco3(s) + sioz (s) + casio3 (t) + coz (g)

(j) .l).L4

2LiH + B2H6 + 2LiBH+

(b) caco3(s)

""*i
haaf

cao(s) + coz(g)
ca2t + H2o + coz(g) -'970V -'580V the net

+ 2 + + HCO3 ( c ) C a C O ( s ) + H ' + C a caco3 ,^, + ts,, OCN + H2O * 2e 2H+ *

3 atoms by means of 2 electrons A 3 center bond jolns 2 atoms bond which joins in contrast to a conventional by means of 2 electrons.

.'.a..L5 B+Hro -> CN + 2OH

PbO 'l- H2O + 2e Yes, if voltage Pbo + cN

+ Pb + 2OH

'l'lre Noble Gases

is subtracted the upper equation - '580 = +'390V is -(-.97O) -> Pb + ocN

(a) 'i - {F
/L\ \P/ -L : . ' r \ r. . . 2. / : . ' Y \::

AB2E3 (trigonal AB4E2

linear bipyramidal square planar pairs)

-/..\
i1/

(octahedral

pairs)

22.IT

The B atom bonds' Two BH2 groups have conventional H atoms w using bridging via two bonds are joined bonds a ! e The conventional BHB bondsare 3 center bonds li-e l-n in the same plane but the bridging all the four bonds to B so that angles at right a plane tetrahedralare roughlY

(c)
tl
tl

AB3E

trigonal

pyramid pairs for I bonds)

(tetrahedral

128

L29

4r(d)

'ii -

,
Xe

AB5E

sguare

CHAPTER 23 PYramid METALS AND METALLURGY

(octahedral

Pairs)

.F

F..

I'he l4etallic (e)


..

Bond

, tl
I I

ABu

tetrahedron are many other waYS to

: O = X e

=o:

(There

tl
l l

draw the Lewis structure

usi

bonds, + formal charge single charge on on Xe, and - formal lead to same resul 0, b\rt all for the shape.)

(photons of all Visible light of all wavelengths energies) are absorbed since an almost continuous band of leuels are available; the re-radiation of these photons gives the metal its luster. (thermal The slightest energy increase energy) raises into electrons empty orbitals which pervade the whole crystal. These highly mobile electrons conduct charge (i.e., current). or momentum (i.e., heat) vr! readily in which compared to other solids the movement of ions is necessary for conduction of either electricity or heat. The extension of the molecular orbital ttrroughout the periodmetal crystal is a consequence of the perfect potential--i.e-, iclty of the electric the perfectly regular arrangement At high temperature of the atoms. this regular arrangenent is disturbed by vibrations of points the atoms about their lattj-ce diminish"ing the mobility of the electrons. In an intrinsic semir (electrons conductor the number of charge carriers or incr'eases holes) exponentially \^rith increasing temperature and the conductivity increases directly with the number of carriers. for highly Note that doped semiconductors the concentration of permanent carriers far exceeds the intrinsic carriers and the temperature characteristics of conductivity revert to metal1ic. Conductors have a conduction band which is only partially filled. with electrons. Insulators and semiconductors filled have completely conduction bands, above which is a forbidden Above energy zone. this is an empty band of orbitals. In semiconductors the forbidden zone is narrow enough so that a few electrons can be promoted thermally up to the empty (See fig. band where they are conduction electrons. 23.3, p. 589).

(r)

O :-bt '
\

,
]l '/ Xe

-'bl '

A86 octahedron
many' many resonance formE

ll 22.L7

../ o:\ ..9: Lt o oq.


(a) xe (g), + re (g) + xeF2 (s) (b) xe(g) (c) xe(g) + 2TzG) + X e F q( s ) + 3 F 2 ( g ) + X e F e( s ) H2o(t) + 2 H F( 9 ) + x e o F + ( l ) + 6Hr (S) + xog (s)

(d) XeFe (s) t

( e ) X e F e ( s ) + 3 H z O( I ) 22.L8 i

The noble gases above Kr have too high an electroto react energy) (too high an ionization negativity Only oxygen and non-metal' with another readily have a high enough electronegativity,to fluorine szp6 from the completed el-ectron density extract gases and form covalent ^vn !n ia r rqr r . iEo n r of the noble s Y s r bonds.

22.Lg

(a) (b)

6,tn2+ + 5Xeo3 + gHzO + 5xe + 6Mnoa [MnOq ] = -L2O M' 3'599 of XeO3

+ 1BH+

22.20

of the London force depends on the diffuseness Both Hz and' He have two electrons cloud. electron bound to one positive but in He they are tightly are nuclei the two attracting while in Hz center; more easily in an elliptical' resulting separated cloud' polarizabler electron

130

I31

23.4

to atomic orbitals atoms contribute Neighboring !l In accordance orbitals' molecular encompassing th' for each a"o" ttrat there is one m'o' the rule of share millions of atoms in a crystal millions energy rangie' calle a timited all within m.o.'s, "band" because the m.o.'s must be so close in an energy band subtends Each the band. within A band mad at the a.o. energy' roughly centered cover a higher energy a.o. will from a higher from a lowetr separated range and may be completely zonet an energy range in which band by a forbidden there are no m.o-ts at all'

(b) (c)

Ba2+.

sr2+,

and pbz+ form

very

insoluble

sulfates.

salts Na' & Mg-' form very soluble exception A possible in sea water. in neutral than sea water Co32-. and NiS are very most of the

with the anions is M9CO3 but HCO3

COz is

rather (d)

PbSr Bi2S3,

insoluble.

,l'r,Llurgy may produce the metal reduces weight, Roasting yield a mixture (e.9., from its sulfide), directly puriamenable to chemical more read.ily of oxides fication. sulfj-des 2ZnS + 3O2 + 2ZnO + 2SO2 C U S + O z + C u + S O 2 carbonates PbCO3 +'PbO + CO2

Physical

Properties,

Occurrence

of

Metals

23.5

of electric metals are good conductors Physically hard and high nel and usually tough, mechanically they are Chemically very high boiling. invariably the coinagt (excepting agents good reducing usually metals) .

3.O

outer 23.7 23.8

P electrons.

process in which (a) Froth floatation is a physical at particles in the crude ore segregate the desired and can be skimmed off bubbles of the air the surface mud below the the useless leaving the froth, with surface. liquid form silver zinc extracts (b) Parkes process. iquid freezes to the top where it lead and floats liquid off. and can be lifted Pt (c) (d) M9CO3 HgS KCl BaSOa ZrSiOa LaPOa (f) r,iquation. heated to just the pure metal (e) Mond process. Van Arkel Ni(impure) + Aco(g) + ti(co),*(S)

2.4 cm native
oxide carbonate sulfide halide sulfate silicate phosphate

metal

process.

zr(impure) zrI4(g)

+ 2fzlg) + 2Iz(g)

-> Zrl4 (9) then, Krolt process.

+ zr(s)

TiCla(9)

+ 2Mg(1) + Ti(s) + 2utgcLz (L)

An ore containing above the metal's poured off'

is metal a native point and melting

z3.Y

Au3+, have such high reduction (a) The ions, e.g-, in the materials that any reducing potentials with them to reduce them have reacted environment to the metallic state.

by is purified An impure ingot (g) zone refning. length, to pass down its molten section a thin causing in the molten portionthe impurities concentrating (h) Thermite Applied process: also to Cr2Oi other + 2Al + AlzOg + 2Cr .9., Fe2O3 n luoOt-

oxides.

132

133

23-L2

(a) ore contains (b)

dug out of the i-s a solid amount of exploitable an the unwanted part of

ground which a metal' the ore' a group lA a hydrat'o'

\-t

i uf

^^^l --i *.>>9rvr9

to

remove

A1 from

Fe:

A 1 2 O 3( s ) + 2 o H ( a q ) Neutrali-ze, A I ( O H )q + and then

+ 3H2o '> 2AI (oH) '+ (aq) calcine to remove H2O:

Gangue is

containing (c) Aln is a mixed sulfate usually and aluminum as cations; rnetaf (d) An amalgam s if t"i=(or .ri." (e) Flux to react (f) Flux as a hot

+ A1 (oH) 3 (s) + H2o A1203,_, + 3H2O(g)


t5,

of a metal in mer a solution may be solid or you prefer); carbonate) oxj-de' added in smol

2AI (oH) g (s)


Dissolve anode: Al2O3 C , . f c t

in

cryolyte

and electrolyze

(or i-s 4n oxide with an imPuritY

+ 20-

,1 !

+ C O z( g ) + 4 e

yields plus impurity to and cools liquid

which separ slag' solid' a glassy lr|

cathode: Produce

3e

+ At-"' + al (1) CaCO3

(9) Smelting in operation liquids. 23.13

refl chemical is a high temperature and slags are presenc metals which

Ca(OH)2 from

CaCO3 + COz(g)

+ CaO water suspension

CaO + H2O + Ca(OH)2,

Fe2O3 + 3CO + 2Fe + 3Co2 CaO+SiO2+CaSiO3 Caco3 -> cao + Coz C + 0 2 + C o z CO2+C-t2CO and the open rtr In both the basic oxygen process of ti:::: the main step is the oxidation process mt more imPuriticl ,Tn addition' addition' ln carbon with oxygen. The basic oxygen process ll removed in the slag. into the melt from abo pure 02 is forced as fast puddJ'o a shallow The open- hearth is very slow as hot air above it' with the metal reacts

Mg (OH) 2 from sea water Precipitate .-+ 2+ Mg'- + ca(oH) z + Mi (oH)e + ca' Form MgC12, dry it, and electrolyze

Mg(OH)z + 2HCL +MgC1z + 2H2O anode cathode, | | 2CI + CLz + 2e + Mg can only be negative) examples fmportant melt.

Mg2* + 2e

(Eo very Very active metals of the made by electrolysis are Mg, AI. Lir Nar Car K.

r "

1 q

In some cases it is not agent; in some cases it the metal.

enough reduc a Powerful in or alloYs vlf dissolves

Eo such as Ag, Cu, Au, and' Metals with positive Eo such as Ni and negative with only slightly others of aqueous by electrolysis Cr can be obtained solutions. '() (a) CuCog is treated hrith aqueous H2SO4 to form (b) CUS can be roasted to Cu. In aqueous CuSO4. in both cases pure Cu is obtained by electrolysis;
/h\
\v /

23.L6

(a) UO3 + 2AL + U + AI2O3 (b) (c) (d) 3V2o5 + l0A1 + 6V + 5Al2o3 -> 2Ta + 5Na2o Tazos + lONa ThO2 + 2Ca + Th + 2cao 'I

#1- a

imnrrra

CU
C+

iS

the

anOde.

cu

(impure) + cu-' + 2e at anode 2+ + 2e + Cu (pure) at cathode Cu-

( e ) W O g + 2 A 1+ W + A 1 2 O 3

'fhe anode voltage to form Ag' is insufficient 3+ under Au so any Ag or Au in the Cu deposits rnode as a sLudge.

-|.

or the

135

23.22

4Ag(s)

+ 8CN

+ Oz(g)

2 H 2 O- + 4 A g ( C N ) z

+ 4OH the

.'t.27

(a) Ca * H2 + ga, (b) (c) (d) 3Ca * N2 + Ca3N2 2Ca * 02 + lgg Ca + C12 -> CaC12

Aqueous cyanide in the presence of air brings silver into from rhich it solution, can by electrolysis.

23.23

(a)

Pbco3 -> Pbo + CO2

roasting

( e ) C a + S + C a S
on

Pbo+c+Pb+co
pbo + CO + pb + CO2

1
I

r - d rct - e - -r- - - i-

(f) (g) (h) (i)

oca r

p4 +

2Ca3p2

f (b) 2PbS + 302 + 2Pbo + 2Soz 2PbO+PbS+3Pb+SO2 (also reactions involving sulfate, see ansr^rer
I l.28 The Representative 23.24 (a) (b) (c) (d) Metals

Ca + 2C + CaC2 Ca + 2H2O + Ca (OH) z * Hz ca + 2NH3 -> ca(NH) z * Hz would require

2Na + Hz + 2NaH Na t N2 + no reaction

less investment of energy to form c+ Ca rather than Ca-', b u t C a C 1 2 has so much greater Iattice energy than CaCl that it more than compensates for the difference in ionization energy. T\ro reaSonS for the large lattice energy of CaCl2 are (a) greater I chargie, and (b) Casmaller ion. size of cu'n, compared to

It

2Na * 02 + Na2O2 2Na + Cle + 2NaCl

hypothetical

(e) 2Na*S+NazS (f) I2wa * P4 + 4Na3e

(g) 2Na + 2C'> Na2C2 (h) (i) 23.25 2Na + 2H2O + 2NaOH + H2 2la + 2NH3 + 2NaNHe + Hz

2+ Be differs dratica11y from the rest of the group in having very small size so that elements lik o*]rgun and halogen remain covalently bonded rather than ionizing. (a) 2AI + 3C12 + A12Ct5 (9) (note: A1C13 dimerizes in gas phase)

potential Ionization reveals that it from Cs than from remove an electron the free gaseous metal atoms. But in the ionizing of environment tendency of Cs because of the large over that released when the very small Li+ ion \4rater molecules. Because of its verv small size the

is
Li

easier to
when

(b) (c) (d) (e) (f) l= lL

4A1 + 3Oz -+ 2ALzAs 2A1 + 3s + At2S3 2AL + Nz + 2AlN 2A1 + 2OH + 6H2O + 2AI (OH) + + 3H2 reacts of a + 3He

an aqueous Li is hydration is surroundEd

2AI + 6H+ + 2Al3+

23.26

Li+

ion

resembles u92+ ldiagonal


J

relationship)

and con
c form Ll

The surface appears to be ',passiver,, that is very slowly. The passivity may be the result tough impervious film of metal oxide.

to Na' forms a rather and hydroxide. Also and its normal oxide

insoluble Li reacts is Li2O.

fluoride, with N2 to

136
L5 I

23.32

Elements on the

will ion to

behave ionic

similarlY radius is

if the

the

ratio

of

Transition (a) 2fe

Metals

and

Inner-transition

Metals

same'

A13+

+ 3Cl2 + 2FeC1g, Cr same

size of about 1.5 times the and Be are Both Al charge. chforides ' covalent 23.33

B e 2 + a n d l ' 5 t i m e s th and both amPhoteric

Zn + CL2 -> ZnCLz (b) 4Fe + 3O2 + 2FezOg' also Cr same

3Fe + 2o2 + Fe3O4

so extensivelY B o t h A I 3 + a n d s 2 - hydrolYze in neutral Lo .tY extent is Present neither 1+ - --^--2t Al"'+H2oZAloH s' In +H2o?HS soluti-on is +oH where to the S 2can exist the --+ +H

that solu (c) (d) (e) ampho solu

2Zn+Oz+2ZnO Fe + S + FeS, Cr, Zn same no reaction steam only

2Cr * N2 + 2CrN' reactions with

others hot

very

basic

3Fe + 4H2O -> Fe3O4 + 4H2 2Cr+3H2O+Cr2O3+3H2 Zn * H2O -+ ZnO + H2 + 2 + Fe + 2H' + Fe * Hz, others Fe + OH + no reaction + 6H2o -> 2cr(oH).t2H2O+ Zn(OH) +2* 3H, + Hz

alwninrun

converted

Al (oH) q sulfur is

In acid tied

where Al3+ can exist ver! weak H2S' 23.34 (a) (b) Sn+2CL2+SnCl4

uP as tha (f) (g)

same

Sn+02+SnOz -> SnS2 ( c ) Sn + 25 -> sn"' (d) sn + 2H' ^ J

2Cr + 6oH Zn + + H2 zOH +

(e)

Sn + OH or

2H2O -> S n ( o H ) g * H z 2-> SnO2 + Hz Sn + 2OH * + 3SnOz + 4NO + ZHzO + Pb (OH) o 2-

\r,

3sn + Arf + 4Nos

metals are much more usefuL structurThe transition speaking. chemically a1ly, but are less metallic of in their salts, capable They are usually colored found combined and frequently many oxidation states, ion. with oxygen in the negative (which is the rule Zr and Hf are very close in size larger for periods 5 & 6 beyond Hf) and considerably period (Genera}ly 5 elements are much than Ti. 4.) than period larger CrO + 2H' -> Cr-' + H2O

J.54

/4^' \ P b O z( s ) + \ ,
(b) (c) (d)

2H2O + 2OH (aq)

( 3PbOzs) + p b s O 4 + O ( S )
-> 2PbO + Oz (g) i ( 2PbOzs) PbO + H2O 'r OH -' Pb(OH)e +

(e)
l.

(f)

SnS + S'- + no reaction 2 - + ^ n^ 5 2 s 3 SnSz+S +PbCIg _ 2 SnF++2F +SnFe

)-

CrO + OH + no reaction cr2o3+6H++2cr3++3H2o Cr2Os + 2OH + 3H2O -> 2Cr (OH) + 2CrO3 + H2O -> H2Ct2O7

(s) P b C l z + C I
l
L

(h)

CrO3 * H2O + H2CrO4 or CrO3 * 2OH -> CrO'+2

i- H2O

l3B

139

r23.40
have The noble metals associated potentials' they lack the tYPical an electron.

unusually high ionizatfon with their small size. of easy trait metallic

t"t't",R24 COMPOUNDS

23.4L

(a) +0.0592 V

(b) +0'770 V

!t,'t ure I

of

Complexes number is the nrn"nber of atoms sphere, i.e., the. number of central metal atom. in the atoms

(c) rhere would be no rtay to know that +0'788 V rrHgtil The use of the correct Eo measurement'
"1.0

that would be M" would give a result l'f cellg' For concentration Eo. as the correct for the cell A calculati-on Eo is not involved"1'0 Mrl (a) and based on a in part described rrHg+rr with I electron exc of concentration

(a) Coordination first coordination connected to the (b) the

A ligand is an atom, ion, or molecule attached to central metal atom via an unshared electron pair.

v,rould give +0-0592 V as the result, measurement would be +0.0296 v' (a) 2La + 3Clz + 2laCls (b) 4r,a -t- 3O2 + 2lazOg (c) 2La + 35 + La2S3 (d) 2r,a + Nz + 2LaN

and the ao

23.42

(c) A chelate is a coordination complex formed by a bidentate or polydentate ligand, i.e., a ligand. with tvo or more coordinated atoms in the same molecule or ron. (d) Enantiomorphs are two isomers which are alike cxcept that one is the mirror image of the other. (e) An inert sociation or (f) its complex has a very ligand exchange. slow rate of dis-

i
ll

(e) 2r,a + 6H2O + 2I (OH) 3 + 3H2 (f) 2 f a + 6H- -> 2 L a - '


+ 3H2

The low spin state of a complex electrons paired as possible. (b) Au 3+, (c) v o,

has as many of

23.43
properties differ also' from

(a) co 3+, (c) the chemical Co 2+

(d) co 3+,

elements differ The inner-transition bf to the number of f electe5' respect with in chenlo involved not generally are electrons betut'A is not much difference so there bonding elements.

metals form complexes most readily, also ohhers whose ions are smal_l and highly charged.. The rrost common arrangements of ligands around the central rrrct.I are octahedral (2 co, tetrahedral , linear ,rdinate), and square planar. I.JII3, a base, ,ru acid, rr,tt.ively via uses its unshared orbital the pair on the silver to bond to Alterin aqueous Ag+,

'l'ransition

an empty

Ag+. ion

one can consider

,;,rlution to be ag(u2O)2+. Then the reaction is a rrtrcleophilic substitution or base displacement r().rction, in which the base NH3 displaces the base
il,O.

r, -= 1.26 x 1O-34

24.6'K=I.35x10-32

140

141

23.40

have The noble metals associated potentials' they lack the typical an electron. (a) +0.0592 V

high ionization unusually Thuli small size. with their of easy loss trait metallic

| ilAr"tER 24 COMPOTJNDS

23.4L

(b) +0-770 v

|,l-.jture .|

of

Complexes number is the nrnber of atoms sphere, i.e.. the. number of metal central atom. in the atoms

+0.788 V iE (c) there would be no l^tay to know that t'Hg*" " The use of Eo measurement. the correct "1.0 M" would give a result that would be regarded cellst For concentration Eo. as the correct for the cell A calculation E" is not involved. (a) and tased on a "l.O M" in part described "Hg*" with exchanged I electron of concentration and the actual would give +0.0592 V as the result, would be +O.0296 V. measurement

(a) Coordination first coordination connected to the (b) the

A ligand is an atom, ion, or molecule attached to central metal aton via an unshared electron pair.

23.42

(a) 2La + 3Cl2 + 2LaC1s (b) (c) (d) (e) (f) 4La + 3o2 + 2Lazos 2I + 35 + La2S3

(c) A chelate is a coordination complex formed by a bidentate or polydentate ligand, i.e., a ligand with t\n/o or more coordinated atoms in the same molecule or ion(d) Enantiomorphs are t\^7o isomers which are alike that one is the mirror except image of the other. (e) An inert or sociation (f) its r (a) (e) t complex has a very ligand exchange. slo\/ rate of dj-s-

2La + N2 + 2 2r,a + 6H2O -' 2Ia (OH) 3 + 3H2 2La + 6H+ + 2La3+ + 3H2

The low spin -state of a complex paired electrons as possible. co 3+, Co 2+ (b) Au 3+. (c) v 0,

has as many of

23.43

with from each other differ metals The transition and since d to the number of d electrons' respect in chemical are very much involved electrons also. differ properties the chemical

(d) co 3+.

from each elements differ The inner-transition but f number of f electrons' with respect to the involved in chemical are not generally electrons between bonding so there is not much clifference elements.

Transition metals form complexes most readily, also others whose ions are sma1l and highly charged. The most counon arrangements of ligands around the central (2 cometal are octahedral, tetrahedral, linear ordj-nate), and square planar. NH3, a base, an acid, natively via uses its unshared orbital the pair on the silver to bond to Ag+,

an empty

Ag+. ion in

Alteraqueous

one can consider

to be Ag(H2O)2+. solution Then the reaction is a nucleophilic substitution or base displacement reaction, in which the base NH3 displaces the base H2O. -3+ K = I . 2 6 x l O

24.6'K=1.35x10-32

L40

L4t

724.7 ( a ) Kz [Rh (Hzo) cl s ] ( c ) N a 3 . [ R e o 2( C N ) + ] ( e ) K+[Ni (CN) +]

( b ) l c o ( N H g )+ ( s o +])N o g (d) lco (wHg z (en) e] Clz ) ( f ) K +[ N i ( c N ) 6]

(a) (b) (c ) (d)

[ P t ( N H s) , * ( o H ) ( s o 4 ) ] o H t P d ( d i p y ) ( N C S )2 l [Co (NHg +Clz ] cl. H2O ) [ c r ( N H 3 ) e] l c r ( C z o + ) z ]

(s)
24.a (a) (c)

lCu(NHg)r+ls[CrC1e]e

V (co) e

(b) zn [Ptcl6 ]

l c o C l ( N o z ) ( t l H s )+ l z S o +
(e) [ P t ( N H 3 )r + ] t P t c l g ( N H r )

Note in compound (d) that if Cr is 3+ in the cation and 1+ in the anion, ions are +1 then the original and -1 and the ions in the isomer are +3 and -3. First then consider a1l possible coordination examine each for stereoisomerism. isomers are (a = NH3): isomers,

( d ) N a 3 [ A g ( S z O s) e ]
tf\

\,
(h) 24.9 (a) (b) (c) (d) (e)
/5\
\ ! /

I P t C l + ( N H s )z ]

(S) Ks [AuBr6,]

l N i ( N H 3 )o l s [ C o ( N o z ) e ] z (O) potassiun tetracyanonickelate ( II ) potassium tetracyanonickelate errate ( III) ammonium pentachloroaquof (II) tetrachloroplatinate tetraamminecopper (III) chloride nitritopentaammineiridium (IIr) tetracyanonickelate hexaamminecobalt

Coordination

[Pta4] [PtClo]r and sixfour(nor

IPta3C1] [PtaCls], [PtaaC12] [PtCl4]. ( (Itr'l coordinate coordi-nate The first any optical For anion, Pt is Pt is

[Pta3C13][PtaCI3], In alf Pt(IV) Pt(II) of these

the

and is and is

the octahedral; square planar.

24.Lo

(a) (b) (c) (d) (e) (f)

(o ) ni tro syltricarbonylcobalt latinum ( IV ) diammine te trachlorop (O) potassium tetracyanoplatinate (II) (tII) hexanitrocobaltate hexaamminecobalt (I) sodium dicyanoaurate Bi s (e thylene di amine ) chloro thi oc yanatocobaLt chloride

two above isomers).

can have no stereoisomers there is only

[Pta3C13] lPtacl3] but two cations:

one possible

c1
Isomerism 24.IL (a) (c) of Complexes (b) Fe [Fe (cN) s ] (d) KzFe Ire (cN) s ] For anion, other KFe [Fe (cN) e] cuz [Fe (cN) o]

c1
a one possible C-l's and the isomers. cis or trans ai

Pta4cl2 PtCl4 there is only but two c a t i o n s , o n e w i t . h c i s trans. the last four have optical

None of

24.L2

(a) A is B i s (b)

I C o ( N H s) s ( I i z o ) C l B r ] B r . H 2 o [ C o ( N H s) s ( H z o ) z c l ] B r z isomers

(a)

isomers, There are tttro geometric where a = NHs, t = NCS-

hydrate

a A

1 1

(a) (b) (c) (d) In

l c o ( N H 3 ) s ( S o q )] N o a (co) s (NCS) ] [rvrn [Pt(NH3)gcl] [Pt(NH3)cl3] [ c o ( e n ) z ( H z o )B r ] B r 2 ' H 2 o (c) both Pt are 2+ in both isomers. I43

c
cis

L42

(b)

There are two NOz = n. Set NH3 = a and (aII a are cis) or facial isomers, geometric edge (two of the a a r e t r a n s ) a

n a " n reLar
(c) Set is NH3 = a. an optical

n
If both isomer a's and Clrs are cis

edgr
t The other is

mlrror
trans BrCl

c1
(f) CalI only C2or*2- = oxt NH3 = . one isomer If NH3 are trans,

mirror
have either ats or Cl's trans.

a If NH3 are cis, there is a d, I pair

c1
(d) Only one

( e ) If Cl and Br are cis i . e . r a d ' 1 P a l r .

there

is

an optical

nLrror

'I

L44

r
(g) There is a d, 1 pair

Noe
I I
I

r,l_+ng_jAjsmpsxee
.'O has no unpaired electrons. [Ni (Cl)q]must have 2 unpaired. lNicl4l'[Fe (CN) e] of CN in is the is d3t-

I
I miffor

.lI 24.L7 No optical and neither


rL^ LJIE ---^ SiItIE .

low

spin,

as deduced series,

from

the

posi-tion

isomers are possible for square p1 for tetrahedral if any two ligands
LEE NHc = A.

spectrochemical high spin.

whereas for

[Fe F6] e(Il-l),

The configurations
?-

(a) The three possibilities are trans " vJ I a - Cl, or trans a - Br. (b) the two possibill are trans Cl's or cis Cl's. (c) Same as (b). (d) Only one isomer. See drawinqs on p . 7 5 6 o f

ron are: [FeF5]-

[Fe(cN)s]

2 4 . 1 8 The dipole

moment of the trans isomer is zero the Pt - Cl bond dipoles are at 180", and likewl the Pt - N bond dipoles. In the cis isomer the Pt - N bond dipoles are at 90o and have a resul opposite in direction but NOT EeUAL to the resul of the Pt - Cl bond dipoles, so there is a net molecular dipole momenE. If either no optical 1 pair. the NO 's or isomer, but Cl's are trans for the al1-cis

+ + + + +

(a) The Cn complex will be low spi-n because A exceeds P, while the H2O complex is high spin. (b) [Fe (cn) e]
+-

l F e ( H 2 O )Gl

24.19

there can there is I

t t t

++
(a)

++

t+

++

The pairing energy must 1ie between 250 and 460 kJ mol and a value of 335 has been reported. p 643 that series Note in the spectrochemical 2than H2O, induces even less of a splitting CzOg 3+ 3so if 1" high spin then [Mn(ceo+) s] [ M n ( H z o )o l (b)

mirror

must

also

be

high

spin.

I46

I47

24.24

tco(NH3)61''

co(NHr)61"'

++

++

+ lrzott
P=270kJ Ao .P

1
++ t+ ++
a o t P

(a) cr3+ d3, same for strong or weak field ligands (b) Ni2+ d8, same for strong or weak

+ t

+ +

l,,ou, I
P=2lOkJ

l+ ++ r+
(c) li2* d8, square planar only occurs with strong field ligands
,1u2 -'r2 T

d 7 ,

d " ,

t* II

dxy d.z2

(a)
4. )

Zn-' ,

Cl

,10

(b)

ct2*,

d4 High spin

Low spin

High spin

Low spin

++++ t+ ++ ++++ ++ ++ +* r+
(c) t+ Ni-'

+tl
(d) co2+ d7, as in onJ-y strong f i e l d ligands
(c)

t+ dxz, dyz
as above f

+ +++ +
(d) Ni"' , Low spin

+ +L_
dL1

,i8 High spin

Low spin + +

High spin

t+ t+ ++
(e) co2+ d7, complexes in weak field
u+ a

+ _
+ t + + +

++++++ ++ t+ ++
(e) I"k 2+ , d5 High sPin

t+ r+ ++
(f) Fe"'

t+1+
, o

all 4th

tetrahedraL peiod are

+ t+ ++

,5
( r ) K n d r m u s t (high fj.eld) spin dj-amagnetic r+ be low if

Low spin

Low spin

High spin

t+ +v t+ r + + ++

+ + + + +f
.,L ?

t + + + ++
(h) Iow spin ,
o

^ + , +

+ + +
, d" High sPin

(S) Ir' d ' , m u s t be low spin (high field) if ONLY one unpaired ti3+ light

(e) Low spin

High sprn

is dr. only one electron, and the hv of absorbed is the measure of Ao. The red-violet transmitted

+
(i)

+
/,L

+
,

+ +
o High

+ + _
(i)
<T

+ + _
, d" High sPin

color is due to absorption of li_ght in the mdd.le of the spectrum, i.e., green. A stronger field ligand causes absorption more toward the vj-oLet end Leavinq the red transmitted.

Low spin

spin

Low spin

I49

' ,.s

t ( a) t?!r ' t 2!7- a * t f

4 2"-

t ,l ,t '

66r-,, 2gtt

* -

66rn 3}ot,

* .

O _14

t"l 1!r * rlo * l"


Radioactivity

(d) - r 9" * r]]* * r]lc"


f,o 13
4

l',.6 (a) t3lr" * 22"lna"z * ^He

,, . (D'
(d)
25.8

r14-, - 114^. 0 * _le 4-7^g 48uq


- rl e

25.L

Vithin the nucleus there must be very strong forces, operating within a very short range 3cm) (\' 2 x 1O-r . otherwise it would not be Pos
for so many positively charged particles, the

(c) * i3* i" . ll""


)',.J ;l',.9 2.6 MeV

+ llFe + llMn zo 23

9.40 MeV

to coexist

in such close proximity.

rt

is Postu

from the exchange of the force originates that One can make a rough analogy betr^reen nucleons. by the being held together molecule hydrogen between the two nuclei. of electrons 25.2 of (a) luclide combination means a particular "isotop6 practically synonlmous with and protonsr and isotope to nuclei tO' is applied except nuclide particle (b) A nucleon of is a nuclear atoms. (c) The crl a proton or neutron. about lu, i.e., matter of fissionable mass is the least quantity exceeds captured the number of neutrons wherein a chain so that reactions, number of fission (d) A thermal neutron has kinetic can occur. ltg of the magnitude RT per mole (for T = 29BK). to that of the average speed is comparable (e) Fission is a at room temperature. molecule splits wherein a large nucleus reaction nuclear from 4O-60% of the ones (ranging 2 much smaller (t along with a few free neutrons in weight), (f) Fusion is the joining of tvo 3 per fission) (g) A Tra one. into a larger small nuclides than 92. element has an atomic number larger

Energy must be 6.34 - 6.12 = 0.22 MeV. Some decays, those giving alphas of only 6.12 MeV, result in an -SOa - 2L7 excited nucleus. This decays very shortly tll"t thereafter to the ground state releasing the .22 MeV of excitation as a y ray.

l'r.10

6.56 parallel

6.23 = 0.33 MeV y rays. The explanation is to problem 25.9 but involves the excited

' -Jlnn. state or 2 r e


.,''.11 19.99999u . " ,. 1 3 3.52 MeV 25.L2 25.I4 25.L6 11.02166u 3.42 MeV 26.98672u

=",.15 30.98066u !',.L7


-",.18 ,!r'.I9 :" -2O

Electron
Must

c a p t u r e m u s t be the process.
capture,

be electron

50.9448 u The nucleus captures the closest electron, namely a (from one ls electron. Vhen a higher energy electron of the outer orbitals) falls into the empty ls orbital its loss in energy i-s emitted as an x-ray.

25.3

t"t

2ffat*2rrar"i * |""

* t"r llar*fl*s f"


. E A

t"r rflet * rl7ro,* |"u

* tur1l!a"*1!!"s l" {c)-fe * tl?"n* r!!na *-1" turllua* i;Ms (d)-ie . i3* * llct

t-c t'r of Rlrdioactive "'.2L .".23 .003659 2BB days

Decay 2 5 . 2 2 ' . o O 7 7 4g 25.24 151 6.40 hr. I

* llNi o l2nc." * l.
150

-al

-1

(a)

.00624 min
-l

(b)

111 nin

Nuclear

Reactions

25.26 25.27 25.29 2 5 .3 l

(a)

.2666 yx

(b)

2.60 Yr. 2900 Yrs. old

255 min 1.55 x lo-64s (a) I.557 x 1 0 r 7 atoms

25.28

25.30 0.a27 c

25.3st.l !!4. + fn * !1"' n v tnt ]!n + fn * |"" * 1r"i

t.r lfcr + fn* i" . if'


(d) ]"i * in * l" * 7n""

( b ) 1 . 5 x 1 0 - g

Di si 4tegrati,ojl

!_eri e s 4

25.32

I 54 *rur n l * IqJ t
of,

;ru
-_L

-ot 150_

'I (n

-llov
oo I qn .,bo o+

- >0 +

rJvmL b)

tA

0 + 150^. -e -,uo Ol

l-46 _ -z ^5m o

t"r ll!t" + lH+ zf" * rflr * ttr 13c" |r"'' l" * j!s. * (s) '3ltn * aru" lr, + '33*

l46e_
6 ,q ?? 232h 90-"

4u^ * 142n^ + 2"" 60."228-^ BB^o 2\6Dn


4

'37u (h) * l3*.* a|"+i33*


ls.3o t"l 13e l^" * N"* 1l* n {rl !r,i + ;n * 1". * l"
(c) ]"i * ln * " * luu tfc n lrtr* ti* * tal t

228^^ 89"" 2L6^t


tJJ

22Brh 90^" 2 L 2R^ . 1


tJJ

)24 ^^}(a

220^-}(n o 208-.
P h

)1 ? -llPo
q

t5z

25.34

-iimp
9 3 )rq 89^" 209^.

J 5_

ZJ J--

grPa
..1 zL_

g2u
111

22911^ 90-" 2 L 3_ . 83IJA

^^tr JW

tsr

Af

2L3_
52

6t

.r)

* <"tenlv"l" * |" * l!t. * trr 13"' l* * lH* l:A=


A q A . 1 A p . {s) j]sc + ;He '+ lH + ;;r

2 0 9B^ . ]
UJ

c-'r + fu * zf" * llc. !]v


;3 ,',.37t"t 'lZu, (b)'3t^, o 2anl"cr,) -)1'7N p , (d (e)"n}u, trl 233u=,rur2f,lv,( h ) - : : c f
?qn
J

L5

153

7
25.3A
? q - :4: c f v

u is

t1: 6 N t

,1 -> *o*

. 133*,

26
CHEMISTRY

"X"

slmbol

for element #105

Nuc]eaI 25.39 25.4O

Fission 7.59 MeV

and Fusion

(a)

H
F ^

, v n z - v B 2 - v n 3

(a) 180.0 MeV released (b) . oB18e" (b)

cHs

C2H5 CH3

25.4I

(a) 5.5 MeV released (b) .1es%


+

cHg - CH - c H - c H - c H 3 85
q 51

2? I I lR - I iqu 2s.42 (a) + ln + 3in + *llce Y U U 5 A

(c)
a

CH

un3 C H e - C = C H z CH I

I
I

(b) -Ilu
/ <a

+ ln + 3ln + lln U U 3/

th

+ r3B^^
f f

UB2

(d)
^ r a =

aH-

CH3

(c) 'llu

oq g1r, * ln * - O " * 134*" + 1 1 z r 0" 52-" 4U

u n - u

C - C H 2 - C H 2 - C H 3

(e)

c1
I
I I I

ur-

CH -

CH3

c1
(f)

cH3 (s)

oz*A[
wJ(h)

?"t
Noz Noz

L54

15

---Tl

25.25 25.26 25.27

(a) .00624 *ir,-r (a) .2666 yx-r 255 min

(b) (b)

1 1 1 -m i n 2.60 yr . 2900 yrs. 0.827 ci old

Nuclear

Reactions

25.3st"l 3i". * f'. * !lu. * y


(b) t3" n 1r,* 1"" * ]"i

25,2A 25.30

25.29 r.55 x to-64u 25.31 (a) 1.557 x l0r7 atoms ( b ) I . 5 x l O s 9

t"r ]fcr* ],'* i" . if'


(d) ]"i * in * " * f,""

Di si ntegrati
5.3 rq -^Er oal

e g4 !_e_-Lis
+ ^He + z n
-r

l5n --DY oo lqn

tlSoo * * f.
tqo -;;"u

(e)ti!t" * fH* zf"* il!'


1 *" R "T1^

* !^. rct 23oc" * |" * l!'.


(s) '31*n |ru * 1r,+ '33* *

'' Il4]^r +
bf

l . + - l l oc a +
, * l42n^

146 ^ -> 4 + o )He

1"^ -it* -

ir.

60-.2287.^ B9^'" 216_.


^_AE

(h)"nZui3*.* a|,'* 3!* *


15.36 (a) t3" * |n" * |" * 1l*

2s.33

): - 1:) r h
YV

- I2l 8 ^a 2 n
'J

22grh 90".. 2L2_.


^-lja

zzl_

2L2D^
at+

zu_ ^-Kn ticr

zlo^ ^,PO t +

208^,
z ) - 2 1 r n 233p^

ol !r.i * f,,* 1""* l" (c)]"i * i" * "* !""


(d) '2" ,lH * l]x * y

25.34

er^*
225n^ gg^" 22Lr_ Bi'-

233rr 92" 2L7 . ^t )_ A E (

229ah 90^" 2L3_.


^.IJr J

))q 99

(e)f,lr"* ln* "* ?3'" (r)ll"" * lH* ln * l3o"


tsr lls" n |nu* !" * llrt

29?no
t3

'9?"t
UJ

(h);1r . fH* zf" * Zi"' ,-37(a)""2u,(b)'tt^, o 2f,lcr,( d ) (e)'32u,rrI 2i3"=,(s) ."r2", ( h )


L52
I f J

-;;Np, ) :c - :\(r f
Y

, ?a (5 . 3

249.,F * r5n o .1 not . g8"t 7ru "x'r is slzmbol for

260-IO5^, #I05

CHAPTER 26 ORGANIC CHEMISTRY

element

Nuclear 25.39 25.40

Fi-ssion 7.59 MeV

and Fusion

26.7

(a)

./"nr-CH2-CH3

- ^
H (b) cHg CzHs CHg I I - CH - C H - C H - C H 3 cHs

(a) tBO.0 MeV released (b) .0818?

25.4L

(a) 5.5 MeV released (b) .L95"6 (c)

* 2s.42 t"l 23r!rvfr' * :f" * t33""

un3

I
J+ 1 ? a ff
I

* * u 2llru 10" tl" ' 3?*

Ln2 (d)

CH

C H z - C = C H e CH CH3

* o 2llru f'' * s|"* '12'"

o-o - v

u = f

I C - C H 2 - C H 2 - C H 3
cl
I I I

(e)
vr 3

CH -

CH3

ua

(f)

cHg (s)

o,n-ffrf xo,
tV!o,oz
(h)

",

rMl t ( ) t ( ) l

\/V
L54

1 C I J

26.2 26.3

See structures Draw the correctly. (a) right

p.

757. inplied and then name them from the


L

structures

di

(4\

cl

- C

c1 I I
CI

CHs - CHz - CH : a smaller index 2-pentene

CH - CHs, by counting nunber is requi-red.

I cl-

c - c - c - c l
cl-

(b)

cH2 - cH - CH3r the


CHs

longest

chain

is

four

CHs

tri
2-mel-hvl hrrf.ane

(5)

cl

( c )

C H 3 - C =

C - CH3

c limpossible, carbon from 5 bonds on left.

I c _
I c1
I

C1

c _ c

C1 I
C l

C1 I
I

C]

C ] - C - C - C

- c

I cHs
(d) CH3 CH -

C = CH,

cHs
2 - r J - ma '{v l re l f -l-'rr1 * -l -*E.yne

from the right by counting the index numbers are smal

C1 l c _ c _ c _ c l _ I
I

cl
l t r

ct
l t t

cl
l l t

C1 l{,.6 CH2 = CH CH2 CHs

(e) CHs -

CHs CH = C - CHg, impossible, carbon from

?H, ?'t'
CH2 -

5 bonds on 3fd left.

'

CH2

n,
(f) CHs CH - CH2 - CHz - CHs, longest chain six carbons. I 3-methylhexane c z Hs ia

^ - cHs
H

cHs

I
C H 2 = C - C H 3

cHs

cHe CHz-CH-CH3

26.4

(a) alkenet
(c) alkadienes

Crrnr.
"nnrn_, Crr"r'

(b)

alkvnes '

C H n 2n-2

^ - cHs
/ and t
H in

(d) cycloalkanes Formulas chlorine;

(Same scheme a s

2 6 . 5 chiral-

carbons*)

26.5

below show only the carbon skeleton for clarity. hydrogen was omitted

c1
butanes (6 structures) C1 - c CI I
C?t -

cl
( m o n o 2 ) C l - C - C - C I C - C - C

156

r57

c1 I
I

Br
c l - c - c - c - c - c l cl

n t - a - r - - r - t - C r.1

r
T 26.9
(a)

I
I

c - c * - c * - c
a

t t

cl

r^
Br

vt'
Br
Br-_Br

clpropanes

I - i
t l

two

(3 s t r u c t u r e s ) C l - C - C - C
(1

Noz Q:,
(b) three

c l - c - c - c - c l

I
I

c1
26.8
(a)

Br
OH OH

Br

Br

r\" \?
(b)

l'-

A
OH I

[*o, werpn' i
$u'
(c) one only

r
Vu
*oz
r

\lu'
OH

Br
../,-*\ OH

,Au'l l( )
\:l't
OH

rAr u'

t ( ) t

sr

VBr

rAr "lV,
OH

t t( I

\ lt

Noz

Br

c1

e'frn'

\l

o,.,.c),.
OH

c1

Br

t ( ) t () t
CI

/ \ / \

lsB

^ral.'OO crr'
\A\l
6.I4 26.LL
Z O . L

(c) Cracking is a process in which a long degraded into shorter molecules thermally other substance involved in the reaction. (d) A conjugated double bonds. (e) In without usually system has alternating

molecule without and

is any

single

an addition reaction two molecules are joined any other reactant involved, or product via one or more double bonds. reaction attached some atom or group to a carbon atom.

(f) In a substitution replaces the hydrogen (a)

x - C = C - X Each of connect (a)

these

four

atoms are the

linea:r bond must

the carbons which form groups. to t\nro unlike QH NO CHs C z Hs

double

CHg

r - = c-\

CHs (b) C Hs

,/

\ 'C . = / C , / \
H (c) CHs

YEs (trans shown)


CH3 CzHs

r6.15
\TA

(a)

cHs

\
u

.
!

/ \
H

./
C = C

,CH2-CH3

* H z

va

CHs (d)

vrtz

vLL.

\
H H C = C - C H z - C H 3 *

C H g- C H z H z + C H a - C H

Hoooc \ \ 'C , / = C

,cooH yeS (cis shown)

(b)

cHs -

II tl
vrr J v.r

,/
H
z6 . r5

further
containing a double

CHs - CH = CH - CHz - C H 3 t C H s - C H z - C H 2 - C H 2 - cHs

H2 ->

(a) An olefin bond. (b) the the the

is

a hydrocarbon

is An homologous series differing same function nrmber of CHz grouPs in molecufe.

of compounds wl a series onlY ln from one another portron the hydrocarbon

(c) HBr+

CHr-CHz. \c=
CH3-CH2

cHz

l-Tf.

Btr I
vLtz v erJ

CH 3-CH2

160

161

(d)

HBr + CH3 - C = C H + C H s I
I

cHz Br

.16.19

l-T{^

-}

l-T{^] v a

a-Rr^ L z v

a-T-I^

I
I

Br 26.16
(a) Br2 + CHa -> HBr + CH3Br takes plaoO .'.6.2O

Substitution on an alkane is difficult but can be facilitated by a free radical mechanism. An aromatic ring is most readily attacked by a Lewis acid, usually a positive ion. After it joins the ring by using a pair of "double bond,' electrons, the aromaticity is restored by splitting out H+ from the carbon which was attacked. (a) (b) (c) (d) (e) (f) (S) (h) o- and p-nitrophenol o- and p-bromophenol m-dinitrobenzene m-bromonitrobenzene m-nitrobenzoic acid o- and p-bromonitrobenzene o- and p-ni_trotoluene o- and p-xylene

A series of substitutions like the above leading to CH2Br2 CHBr3, and CBr4. (b) CHs CH = CH - CH3 *

Br2 + CHg - CHBr - CHBr

ci,
( c ) C H - C = C - C H 3 + +2P.12 CH3-CBr2 -CBr2 cHs
(d)

i\Lcoholsr

''@
'6.2L (a)

Ethers,

Carbonyl

Compounds,

Br, *

tTl
(b)
arTf ^ -

_ C - H
U

tl tl

tl C

26.I7

(a) (b) (c)

CH2 = CH2 + Br2 + CH2Br - CH2Br CH=CH+2HBr+CHg-CHBr2

CHz - CHs

same as

(c)

(d) cH3 - O - C H 2 - C H 3 (ej Un3 - O - C - C H 3 tl


ll

-. v.. CHz \ .

-^.<r /
H2SO , CHz + , , ^ -- ; H2O ^,,/

CH2-CH2

""rr
CH2 g

u n - u n

./"r,

(f)

OH 26.L8 (a) the


+^

In an unslnnetrical addition the hydrogen adds E0 carbon with the more hydrogens already attached
i +

(b) The hydrogen, as H-, adds first, to form a carbocation. The more stable carbocation has the + charge on the more substituted carbon, so the H atom goes to the less substituted carbon, i.e., the carbon wj-th the more hydrogens.

(s)

f^

I o -N a+'
cHs

C H s - C H z - C H z - C H - C H 2- C _ O H

il ll

L62

163

(h)

lt cHs - c cHs
(i)

il

26.25
vL! z

Hydrogen bonding leads forces in alcoho1s. (a) (b) Grignard reagent,

to

very

strong

intermoLecular

vLtz

vr

26.26

C4HeMgBr (n-butyl cyanide)

NaBr and cyanobutane C+HgCN

OH

(c)

Br

and n-butyl C+HgOH

alcohol

(I-butanol)

Br

(d)

NaBr and ethyl

n-butyl

ether

CaHeOC2H5 26.22 (a) (b) (c) (d) (e) (f) (g) (h) (i) /-i' \ \J Four 4-methyl-1-Pentanol 4-methyl-2-Pentanol 2-methYl-2-Pentanol 2-methyl-3-pentanone ether isoProPYl ethyl 4-methylpentanal methyl PhenYl ketone cyclohexanone benzoate methyl *-- iu +-obenZOic aCid Ir rv alcohols and three ethers CHs (d, 1 pair) OH C H e - C - C H 3 (e)
tl il

26.27

(a) (b) (c)

Hz and C3H7O

Na+

HzO and CH3 - CH = CHz H2O and CH3 - CH - O - CH - CHs

CHs (d) o

CHe

H2O and CeHs - C - O - CH - CH3

tl

26.23

cHs
tl

CaHeOH

C2H5 - CH OH

Cr-'

and CH3

26.28

C H - C H - C H 2 O H

cHs C H g - O - C H - C H s

cHe

(a*f) Molecules with double or triple:bonds are cleaved to the resultant acids or ketones; aLcohol-s and aldehydes are oxidized to the corresponding acids. (a) (b) (c) (d) (e) (f) (9) (h) (i) acetic acid and butanoic acid (acetone) acetic acid and propanone 2 molecules of propanoic acid propanoic acid propanone (acetone) propanone p-nitrotoluene is oxidized to p-nitrobenzoic acid product too mild to cleave, is diol, 2, 3-hexanediol under mild. conditions reaction stops at aldehyde, propanal

I cHs

CzHs-O-C2H5

C H s - O - C H z 26.24 propyl formate (a) methyl propanoate; propyl methyl isopropyl (b) methyl etheri (c) propanal (d) 2-butanone (e) 2-pentanol, 3-Pentanol (f) methyl proPanoate Many others are Possible.

ether

t64

165

26.29

(a) l-pentanol (b) 2-pentanol (c) pentane (d) first 2-pentene' (a) (b) seo and ethyl ethane

(c) then pentane (d)

H 2S O a 2 CH3CH2OH+. 7 medrum temp. from

HzO +

(CHgCHz)z O

26.30

bromide

(a) cHc=cHe reduction. wTffi-rr-+ strong ox'

u^.H^ un3Ln3

and MgBr2 ( e ) cHgcHzoH acid (f) 2-methylproPanoate alcohol

(c) MaBr and propanoic 26.3I (a) (b) (c) (d) 26.32

> cH3 cooH

C H g C H z O H+ H B r + H z O + C H 3 C H 2 B T from (f) CH3-CH2Br*NaCN + CHs-CNz-CN+NaBr OH base + from (b) CH3CHO HCN+ C H s - C H - ur\ --_+ hydrolysis

(CHg)z CItCOO-la+ sodium (cHs) z cH-oH' isopropyl

( s)
(h)

ceHz Coo Na- sodium

butanoate ethanol

CaHz Coo Na+ and Ho-CH2-CH3, (b)

(a) propanol (c ) 3-methyl-3-pentanol

3-hexanol

oHo

OH

26.33

(a) ( C H g) z C H M g B r or (b) ( C H g) z C O + CH3CH2MgBT+ CHg CH3 Mg Br +


* CH3 CHO +

oH CHs
CHg-CH-CH-CHs

cHs-ri-c-o-Na+

strong+ :id

cHa - cH - cooH

t t

/ -." ( C H s) z C H C H O OH

26.35

+ CH3CH2CH2OH HBr + CH3CHzCH2BT + CH3CH2CH2BT Mg + CHsCHzCH2M9BT

I C I

similarly CHz - CHs


url3Ur12(-r12rjr1

prepare
-->

(CH3) 2CHI49Br
L.n3\-n2Lnu

mr_Id ox.

CH

similarly (a)

prepare

CH3COCH3 (acetone) hydrolysis)

(c)
vr J vttz

^
I
l

C H g C H z C H o+ C H g C H z C H z B T ) ( a f t e r
CH gCHeCHOHCHzCHzCHg

I
I

M g B r + C - H - )

CH3CH2CH-CH2-OH

I I
CHa

I
I

(b)
(CHg)zCH Mg Br + CH3COCH3 + CH3 CH -

oH
C -

cHg

26.34

(a) cnecnzoH

H'sol high temp. *ild o";

Hzo + cH2 = glt


(c) OH

CHg

CHs

I I

CH3

(b) cHacHzoH avoid

cH3cHo (dist.irt

the product

to

C3HTM9Br + CH3COCH3+ C s H z - C - C H 3
. un3

further

oxidation)

;,,

L66

L67

l
(d) (CH3)2CHt{gBr * CH3CHzCHO CH3-CH2-CH-CH-CH3 +
OH CH3
zo.Jo

(e)

t l

H2 SOa --+ CHgCHzCH2OH high temp.

CH3CH = CHz

CH3CH = CHe + HBr + CH3CHBTCHg cH3cHBrCH3 + NaOH + CH3CHOHCH3 NaBr * 26-37 (a) The alkyl halide ionizes to a slight, extent to give a halide ion and a carbocation which is a powerful Lewis aci.d. that displaces a proton from water. (b) A Lewis the OH- ion base from such as a bromide the alcohol. ion displaces Secondary (C2H5) 2NH Tertiary 26.40 (a)
26.39 Primary CaHeNH2

NO^
L

amines (CH3 ) 2CH CH2NH2 (CHg ) g CNHz

CH3CH2-CH-NHz

I
I

cHe
amines: C3H7 NH CH3

(c) HCN ionizes to a slight extent and the CN i o n , a strong Lewis base, joins the carbon of the group. The excess electron is then local_ized on th oxygen which becomes negatively charged and a stronE base and picks up the hydrogen (1eft ion from the ionization of HCN).

(CHs) z CHNHCH3 (CHg)z NC2H5 amine' others

amine ethyl

methyl

Ami-nes 26.38

Amides (a)

(b) one HzNCHzCooH isomer is 2-hydroxyacetamide C2H5NO2. is ntroethane HOCHzCONHz. Another (c) N-methyl acetamide CHg I NH - CHs

o
26.41
(a) ethylamine CeHsNHz i.e., p-aminotoluene' or

I
H
(b) (c) (d) CH3NHC2H5 H2NC5H12NH2

(b) p-toluidi-ne,

p-methylaniline

HeN{(
c - NHz o

V/
(c) (d)
=na{-mia T]^ -

)F",
amine

HBr and methyl

butyl

tl l t
C -.NHZ

168

L69

(e) (f) (g) (h)

HN+er and CHgCHzCOOH diethylammonium base hydrolYsis bromide Yields (czHs)e NHe* Br CHgCHzCOOH water' Cl and

Pol)mers 26"47 (a) Addition pollzmers are formed when the double bond opens up and joins one molecule to another. Poly (vinyl chloride) is an example:
- C H - C H 2 - C H -

neutralization ethylamine

yields C2H5NH2

26.42

to amines they Applied directly atoms attached they count to alcohols to the carbon directly

the number of carbon count Applied to the nitrogen. the carbon atoms attached the -OH group. which carries

cl

cl

CHz-CH-CH2-

c1

26.43

(A) CHCHBTCH3 * NH3 + CHS - CHCH3 + HBT


I I

(b) CoBolymers are formed \,/hen two different unsaturated compounds add. Butadiene-styrene rubber is an example:

NHz (b) (c) (in CzHsCOOH + NH3 + CzHsCOO NH+r + C z H s C O O C H 3 N H 3 + C z H s C O N H z+ C H 3 O H hydrolysis to

-CH? - CH = CH - CHn - CH - CH^t ' " l | |

aqueous medium one gets + CzHsCOO NH+ + CHgOH)

i i repeat unit

lt \2

)l

I i

26.44

CH3CH2OH2 H3o*, cH3cooH.H2o CHaCHeOH CHsCHzNHz r

t,

(c) Cond.ensation polymers are formed by condensatj-on reactions, including such compounds as polyesters and polyamides. Nylon 66 is an example of a polyamide:

26.45 26.46

CH3CH2O OH CH3CH2NH2 CH3COO r CHTCHzOH (a) The conjugate base of phenol (see structures top by resonance is of stabilized highly page 695) '

- lN r cI - c r * H g - c - N aoEtz
I I
26.4A

l l l

Nl'C

l : I
I

i l i c4Hg- c - N -

repeat unit

in on the nitrogen (b) The unshared pair of electrons because it is withdrawn is "less available" aniline by the resonanc somewhat as suggested the ring into forms:

lI",

@N*,

fis**"' \ r ' V
L70

ll"t
26.49

In a Friedel-Crafts reaction the A1C13 is a Lewis acid whi-ch adds a halide and generates a reactive polymerization carbocation. In addition AlC13 is a Lewis acid which adds to an olefin localizing a negative charge on the AL and generating a carbocation at the other end of the mo1ecu1e, which is propagated at the end of the growing chain. (a) Ethylene, (b) tetrafluoroethene or ethene (c) styrene (phenylethylene, or vinyl benzene) (d) vinyl (or cyanoethylene, cyanide or a-crylonitrile) (e) adipic (see problem acid and 1, 6-diamino-hexane (f) isoprene (or'methylbutadiene) 26.4'7c) (9) phenol and formaldehyde (h) terephthalic acid ' glycol. and the ethylene

L7T

--5
26.50
(a) (b) (c)

cHz cF2
CH

c 7 d z - c H 2 + c H-2 cH2CF, - CF, r- CF, - cF2L L t -

c H 2 - f Ht

',A''ll t( )t
\7r
(d) - c}lz - c H l
C N I

Amino Acids, 27.L

proteins

t l

cH^-cn+ ' l l
cNl

cH. o

(e)

See 26. 47 (c)

H z N - H - { !O H i
- cH2 f c H r C = C H - C H o -

cH. o
H T /

n - H - - oH -' H2o +
H

(f) 1-.",- C = C H I cH3


I

J",

(e)

OH

OH

H 2 N - - b - u - c H - b - o H the
H

f", I

"r fl

dlpeptide.

cHz+A

I \cfz /
I ---., f, fl nf lo-.,",-o-'{O>bio-c2lt4-ob<(
', . u
\ t -

repea of the above reaction peptide (shown on page 7Sg).

gives

the

tri-

(h)
t

\\__Z + r

\_ ))-,/ -

27'2
H2N

CHr-H-r:r+ "':

fl - b - o H ,,*, ' " r + - c H _ [fl - o " H' fr |


\ -

cH3-cH-cH3o

\ o + l l H3N- cH - [ - o, cH3-CH_CH3 .uI


27.3 There is no chiral carbon is no optical isomerism. Primary: the sequence of

r "i H2N- cH - c - ocHr_u_cH,


atom in glycine so there

amino acid

resid.ues. of the

Secondary: chain

the local structural_ (e.9., alpha helj.x).

confj-guration

L72

r73

Tertiary: the overall how it is folded.

configuration

of

the

chain;

(e)

how several (not connected by chains Quaternary: covalent bonds) are bonded to form one functj-onal unit.

cH -

I c I
H

zHs
OH vs. its mirror imaqe

t.5

Secondary structure depends primarily on hydrogen bonding. Tertiary structure, in addition to hydrogen bonding, uses salt formation + (R-NH3 OOC-R'), disulfide (R-S-S-R' ) , links and van der Waals forces.
/ i r ! ' '

27.LO

The chiral (a)

carbon H

is

shown by an asterisk. (b) HO-CHzCOOH none possible

CHg-C*-COOH

I
I I

f . o

1,ooo,ooo,/l2o

8330 residues (c)

UN

t.7

From A and B there are four possible peptides AA, AB, BA, and BB. Note that AB is NOT the same as BA. For tripeptides, you can fill each of these slots in two ways: 2 x 2 x 2 = B possible tripeptides. In the general caser loll can fill each of y slots in x V ways gj-ving x' possibilities. The + sign means that right handed rotation is observed for the compound., whereas D slmbolizes absolute configuration in accordance with the convention shown at the top of page 713.

CHs - CCIBr

- CH

none possible

(d)

c1

I C H s - C * - H I
Br Carbohydrates 27.LL

(e)

OH C H e -c

I* -

CN

C zH s

.B

the

A carbohydrate is a polyhydroxy alcohol, with ether, aldehyde, or ketone groups, formula CH2O or close to it.

frequently with empi-rical

(a) CH3 - CHz - CHz - CH3 vs.

CHs - CH - CH3 I I
cHs CH3 - CH - C3H7

27.L2

(b) CzHs -

A five or six carbon carbohydrate is a monosaccharide, also ca11ed a sugar. An ether formed from two sugars is a disaccharide. A polyether formed from a sinqle type of sugar (glucose) is a polysaccharide. CHz OH The middle carbon is chiral; a d and 1 pair exists.

CH - C2H5 CHs

vs.

27.L3

CHa vs. C2H5OH 27.L4

I
I

OH
-u I
I

( c ) C H e - O - C H 3 (d) tt

CHz OH

c1 ./"
1 I

*CH-CHO I I
OH
e vrrz

OH

2 chiral

carbons,

4 J-Somers

* cf I
I

I t

OH

OH

o o H

i i

l
one chiral carbon, 2 isomers

L74

L75

'ats i7.15

and Oils C1TCOOCH2 Cr zCOOCH C1 TCOOCH2 3 C 1T C O O +3NaOH->


aLtt r ^z v t
t t T t\q

2 7 . 2 2 Call

T H l
l l v tLz I I

the two complementary chalns X and Y. They separate and X generates partner a complementary Yr which must be identical_to Y, while Y generates a partner complementary X' rrhich must be identical to X. The two new double helixes XYr and YXl, are identical to the original XY. in the sequence of nucleotides

OH

OH

OH 27.23 A mutation is a mistake in a DNA chain.

7 -L6

C 3 H 6 = C s H h = C 3 H 4 = CaHrs COOCH2 C 3 H 6 = C 3 H a = C 3 H 4 = CeHls COOCH C3H5 = CsH4 = CaH+ = C r z H a5 C O O C H 2 Cr zHe 5COOCH C r z H g5 C O O C H 2


u8nlS LL JLH2

+ 9H2 ->

27.24

90,OOO/L2O = 750 amino acid residues. 3 bases to code one amino acid, so 3 x = 765,000 molecular !'t. RNA. (a) (b) (c) complement complement of of A-c-A U-C-U is is U-C-U A-G-A

It requires 75O x 340

27.25

on DNA is T-C-T

opposite'strand

7-L7

I + CTTHz3COOCH I Ct
7.lB

Cr THz3COOCHz 5 4 / 0 2 + 39CO2 + 37HzO

Enz)mes and Metabolism 27.26 Turnover nrnnber is the number of one enzlme molecule performs its times per catalytic minute function.

I Hz 3COOCH2 from 27.2'7 whch oils are derived are

The fatty aeids unsaturated. Ibtal of

l.Lg

18 carbons;

stearic

Competit.ive inhibj-tors form a complex wi-th enzyme in (as does the subequilibrium with the free species strate). Non-competitive inhibitors form permanent (non-reversible) complexes with the enzlme. If the product of an enzlmatic reaction is a mildly "turn-off" competitive inhibitor it will the enzyme it from producing and prevent more product than is needed. is used when The term "feedback" inhibition product it is not the primary itself but some product subsequent which competes with the substrate for the enz!me and turns off production of the primary product. The enzyme forms a loose complex with the substrate which causes a particular bond to become d.istorted and made prone to react with minimal energy of activation. They are very specific because onl-y the provided fit correct substrate into the template will
l-rrr'l-lra
v af Irv

acid. 27.28

rcleic 7.2O

Acids An alpha helix is a protein conformation; a single strand held in place by intrachain hydrogen bonding (approximately paral1e1 to the axis). A double helix is a DNA conformationi a double strand held in place by interchain (approximately hydrogen bonding perpendicular to the axis). Not identical but complementary. has A the other has T, and where has C. tr{here one chain one has G the other

27.29

'.2I

L76

r77

30

Catabolic building

means breaking down, up or synthesizing.

and anabolic

means

31

Suppose a substance A is to be converted to substance C which has a higher free energy than A. It can be accomplished if the reaction A + B -) C + U 4 > 4 negative AG; such a reaction is ca11ed a coupled reaction.
L ^ ^

32

The substance B in the preceding example j_s a "high_ energy" molecule. Its spontaneous reaction to low energ:y products can be coupled to a desirable reaction which otherwise would be thermodynamically impossible. An example is ATp; in which case "D" represents ADp plus inorganic phosphate.

Lt

You might also like